1235
Set Theory A set is an unordered collection of objects without repetitions.

Finite Math Lecture Slides

Embed Size (px)

DESCRIPTION

Slides for a Freshman level course in Finite Math: Sets, Combinatorics, Probability, Matrix Algebra, Linear Programming, and Markov Chains.

Citation preview

Page 1: Finite Math Lecture Slides

Set Theory

A set is an unordered collection of objects without repetitions.

Example{a, b, c} = {b, c , a}

We usually use capital letters to denote sets.

A = {a, b, c}B = {b, c, d , e}

Sets are the nouns of Set Theory, they’re what it’s about.

Page 2: Finite Math Lecture Slides

Set Theory

A set is an unordered collection of objects without repetitions.

Example{a, b, c} = {b, c , a}

We usually use capital letters to denote sets.

A = {a, b, c}B = {b, c, d , e}

Sets are the nouns of Set Theory, they’re what it’s about.

Page 3: Finite Math Lecture Slides

Set Theory

A set is an unordered collection of objects without repetitions.

Example{a, b, c} = {b, c , a}

We usually use capital letters to denote sets.

A = {a, b, c}B = {b, c, d , e}

Sets are the nouns of Set Theory, they’re what it’s about.

Page 4: Finite Math Lecture Slides

Set Theory

A set is an unordered collection of objects without repetitions.

Example{a, b, c} = {b, c , a}

We usually use capital letters to denote sets.

A = {a, b, c}B = {b, c, d , e}

Sets are the nouns of Set Theory, they’re what it’s about.

Page 5: Finite Math Lecture Slides

Relations

In mathematics a relation is like a verb:

it “relates” two things in a way that can be either true or false.

In high school algebra the common relations are =, <, >.

In Set Theory the most common relations are =, ⊂, ⊃, ∈.

Page 6: Finite Math Lecture Slides

Relations

In mathematics a relation is like a verb:

it “relates” two things in a way that can be either true or false.

In high school algebra the common relations are =, <, >.

In Set Theory the most common relations are =, ⊂, ⊃, ∈.

Page 7: Finite Math Lecture Slides

Relations

In mathematics a relation is like a verb:

it “relates” two things in a way that can be either true or false.

In high school algebra the common relations are =, <, >.

In Set Theory the most common relations are =, ⊂, ⊃, ∈.

Page 8: Finite Math Lecture Slides

The expressionA ⊂ B

means that every object (or element) of A is also in B.

Example:A = {a, b, c}B = {a, b, c , d , e}A ⊂ BB ⊃ AB 6⊂ A

Also,A ⊂ AB ⊂ B,

so ⊂ is analagous to ≤.

Page 9: Finite Math Lecture Slides

The expressionA ⊂ B

means that every object (or element) of A is also in B.

Example:A = {a, b, c}B = {a, b, c , d , e}A ⊂ BB ⊃ AB 6⊂ A

Also,A ⊂ AB ⊂ B,

so ⊂ is analagous to ≤.

Page 10: Finite Math Lecture Slides

The expressiona ∈ A

means that the object a is contained in A.

Example:A = {a, b, c}a ∈ Ab ∈ Ad 6∈ A

Page 11: Finite Math Lecture Slides

The expressiona ∈ A

means that the object a is contained in A.

Example:A = {a, b, c}a ∈ Ab ∈ Ad 6∈ A

Page 12: Finite Math Lecture Slides

Operations

An operation is something you do to objects to get new ones.

In algebra the common operations are +, −, ·, ÷, etc.

In Set Theory the common operations are ∪, ∩, ′, and ×.

Page 13: Finite Math Lecture Slides

Operations

An operation is something you do to objects to get new ones.

In algebra the common operations are +, −, ·, ÷, etc.

In Set Theory the common operations are ∪, ∩, ′, and ×.

Page 14: Finite Math Lecture Slides

The expressionA ∪ B

means “combine all elements of A with those of B”.

Example:A = {a, b, c}B = {b, c, d , e}

A ∪ B = {a, b, c , d , e}

Loosely, ∪ means “or”.

Page 15: Finite Math Lecture Slides

The expressionA ∪ B

means “combine all elements of A with those of B”.

Example:A = {a, b, c}B = {b, c, d , e}

A ∪ B = {a, b, c , d , e}

Loosely, ∪ means “or”.

Page 16: Finite Math Lecture Slides

The expressionA ∪ B

means “combine all elements of A with those of B”.

Example:A = {a, b, c}B = {b, c, d , e}

A ∪ B = {a, b, c , d , e}

Loosely, ∪ means “or”.

Page 17: Finite Math Lecture Slides

The expressionA ∩ B

means “take only those elements of A which are also in B”.

Example:A = {a, b, c}B = {b, c , d , e}

A ∩ B = {b, c}

Loosely, ∩ means “and”.

But the English word “and” sometimes means “or”!

Page 18: Finite Math Lecture Slides

The expressionA ∩ B

means “take only those elements of A which are also in B”.

Example:A = {a, b, c}B = {b, c , d , e}

A ∩ B = {b, c}

Loosely, ∩ means “and”.

But the English word “and” sometimes means “or”!

Page 19: Finite Math Lecture Slides

The expressionA ∩ B

means “take only those elements of A which are also in B”.

Example:A = {a, b, c}B = {b, c , d , e}

A ∩ B = {b, c}

Loosely, ∩ means “and”.

But the English word “and” sometimes means “or”!

Page 20: Finite Math Lecture Slides

U

The letter U is used to denote the universe,

the set of every object that matters in a problem.

Example: If a problem is about English letters, the universe mightbe

U = {a, b, c , d , e, f , g , h, i , j , k , l ,m, n, o, p, q, r , s, t, u, v ,w , x , y , z}.

But if it’s about Greek letters the universe would be

U = {α, β, γ, δ, ε, ζ, η, θ, ι, κ, λ, µ, ν, ξ, o, π, ρ, σ, τ, υ, φ, χ, ψ, ω}.

If it’s about rolling dice the universe might be

U = {1, 2, 3, 4, 5, 6}.

Page 21: Finite Math Lecture Slides

U

The letter U is used to denote the universe,

the set of every object that matters in a problem.

Example: If a problem is about English letters, the universe mightbe

U = {a, b, c , d , e, f , g , h, i , j , k , l ,m, n, o, p, q, r , s, t, u, v ,w , x , y , z}.

But if it’s about Greek letters the universe would be

U = {α, β, γ, δ, ε, ζ, η, θ, ι, κ, λ, µ, ν, ξ, o, π, ρ, σ, τ, υ, φ, χ, ψ, ω}.

If it’s about rolling dice the universe might be

U = {1, 2, 3, 4, 5, 6}.

Page 22: Finite Math Lecture Slides

The expressionA′

is read “A complement” and denotes the objects of U not in A.

Example:U = {1, 2, 3, 4, 5, 6}E = {2, 4, 6}

E ′ = {1, 3, 5}

Loosely, ′ means “not”.

Page 23: Finite Math Lecture Slides

The expressionA′

is read “A complement” and denotes the objects of U not in A.

Example:U = {1, 2, 3, 4, 5, 6}E = {2, 4, 6}

E ′ = {1, 3, 5}

Loosely, ′ means “not”.

Page 24: Finite Math Lecture Slides

The expressionA′

is read “A complement” and denotes the objects of U not in A.

Example:U = {1, 2, 3, 4, 5, 6}E = {2, 4, 6}

E ′ = {1, 3, 5}

Loosely, ′ means “not”.

Page 25: Finite Math Lecture Slides

Example: Suppose that

U = all animals W = whalesF = fish M = mammalsA = aquatic animals S = seals.

Translate the following English phrases into Set Theory.

1. Whales are aquatic mammals.

2. Whales are not fish.

1.W = A ∩MW ⊂ A ∩MW = A ∪MW ⊂ A ∪M

Page 26: Finite Math Lecture Slides

Example: Suppose that

U = all animals W = whalesF = fish M = mammalsA = aquatic animals S = seals.

Translate the following English phrases into Set Theory.

1. Whales are aquatic mammals.

2. Whales are not fish.

1.W = A ∩MW ⊂ A ∩MW = A ∪MW ⊂ A ∪M

Page 27: Finite Math Lecture Slides

Example: Suppose that

U = all animals W = whalesF = fish M = mammalsA = aquatic animals S = seals.

Translate the following English phrases into Set Theory.

1. Whales are aquatic mammals.

2. Whales are not fish.

1.Solution:

W ⊂ A ∩M

Page 28: Finite Math Lecture Slides

Example: Suppose that

U = all animals W = whalesF = fish M = mammalsA = aquatic animals S = seals.

Translate the following English phrases into Set Theory.

1. Whales are aquatic mammals.

2. Whales are not fish.

2.W 6= FW 6⊂ FW ⊂ F ′

W ∩ F = ∅ = {}

Page 29: Finite Math Lecture Slides

Example: Suppose that

U = all animals W = whalesF = fish M = mammalsA = aquatic animals S = seals.

Translate the following English phrases into Set Theory.

1. Whales are aquatic mammals.

2. Whales are not fish.

2.W 6= FW 6⊂ FW ⊂ F ′

W ∩ F = ∅ = {}

Page 30: Finite Math Lecture Slides

Example: Suppose that

U = all animals W = whalesF = fish M = mammalsA = aquatic animals S = seals.

Translate the following English phrases into Set Theory.

1. Whales are aquatic mammals.

2. Whales are not fish.

2.Solution:

W ⊂ F ′

W ∩ F = ∅

Page 31: Finite Math Lecture Slides

Venn Diagrams

A Venn diagram is a visual representation of sets.

Page 32: Finite Math Lecture Slides
Page 33: Finite Math Lecture Slides

How can we describe the shaded region?

(A′ ∩ B ′) ∪ (A ∩ B ′) ∪ (A′ ∩ B)

(A ∩ B)′

Page 34: Finite Math Lecture Slides

How can we describe the shaded region?

(A′ ∩ B ′) ∪ (A ∩ B ′) ∪ (A′ ∩ B)

(A ∩ B)′

Page 35: Finite Math Lecture Slides
Page 36: Finite Math Lecture Slides
Page 37: Finite Math Lecture Slides

How can we describe the shaded region?

(A ∩ B ′ ∩ C ) ∪ (A ∩ B ∩ C ) ∪ (A′ ∩ B ∩ C )

(A ∪ B) ∩ C

Page 38: Finite Math Lecture Slides

How can we describe the shaded region?

(A ∩ B ′ ∩ C ) ∪ (A ∩ B ∩ C ) ∪ (A′ ∩ B ∩ C )

(A ∪ B) ∩ C

Page 39: Finite Math Lecture Slides

Using Venn Diagrams to Count

Venn diagrams can be helpful in some counting problems.

We’ll use the notation n[A] to denote the number of elements in A.

Example:A = {a, r , y , p, j}

n[A] = 5

There are two important principles:

AP The addition principle, also called the partitionprinciple;

CP the complement principle.

Page 40: Finite Math Lecture Slides

Disjoint

Sets are disjoint if they don’t overlap:

A ∩ B = {} = ∅.

Example:

disjoint: {a, d , t, p}, {b, f , o, v , z}not disjoint: {a, d , t, p}, {b, f , p, v , z}

Page 41: Finite Math Lecture Slides

APThe addition principle, or partition principle, says that the size ofthe union of two disjoint sets is the sum of the sizes of each one.

If A ∩ B = ∅ then n[A ∪ B] = n[A] + n[B].

Page 42: Finite Math Lecture Slides

APThe addition principle, or partition principle, says that the size ofthe union of two disjoint sets is the sum of the sizes of each one.

If A ∩ B = ∅ then n[A ∪ B] = n[A] + n[B].

Page 43: Finite Math Lecture Slides

APThe addition principle, or partition principle, says that the size ofthe union of two disjoint sets is the sum of the sizes of each one.

If A ∩ B = ∅ then n[A ∪ B] = n[A] + n[B].

Page 44: Finite Math Lecture Slides

Example: What is n[A]?

n[A] = 4 + 3 = 7

Page 45: Finite Math Lecture Slides

Example: What is n[A]?

n[A] = 4 + 3 = 7

Page 46: Finite Math Lecture Slides

Example: What is n[A ∪ B]?

n[A ∪ B] = 3 + 5 + 12 = 20

Page 47: Finite Math Lecture Slides

Example: What is n[A ∪ B]?

n[A ∪ B] = 3 + 5 + 12 = 20

Page 48: Finite Math Lecture Slides

CP

The complement principle is based on the observation thatU = A ∪ A′.

n[A] + n[A′] = n[U]

n[A] = n[U]− n[A′]n[A′] = n[U]− n[A].

So if you know n[U] and either n[A] or n[A′] you can figure out theother.

Page 49: Finite Math Lecture Slides

CP

The complement principle is based on the observation thatU = A ∪ A′.

n[A] + n[A′] = n[U]n[A] = n[U]− n[A′]n[A′] = n[U]− n[A].

So if you know n[U] and either n[A] or n[A′] you can figure out theother.

Page 50: Finite Math Lecture Slides

CP

The complement principle is based on the observation thatU = A ∪ A′.

n[A] + n[A′] = n[U]n[A] = n[U]− n[A′]n[A′] = n[U]− n[A].

So if you know n[U] and either n[A] or n[A′] you can figure out theother.

Page 51: Finite Math Lecture Slides

Example: What is n[A]?

n[A] = 15− (5 + 7) = 3

Page 52: Finite Math Lecture Slides

Example: What is n[A]?

n[A] = 15− (5 + 7) = 3

Page 53: Finite Math Lecture Slides

Example: What is n[A′ ∩ B ′ ∩ C ′]?

n[A] = 40− (3 + 5 + 6 + 12) = 14

Page 54: Finite Math Lecture Slides

Example: What is n[A′ ∩ B ′ ∩ C ′]?

n[A] = 40− (3 + 5 + 6 + 12) = 14

Page 55: Finite Math Lecture Slides

Play VennDoku athttp://mypage.iu.edu/˜dabrowsa/FiniteJS.html!

Page 56: Finite Math Lecture Slides

Example: There are 100 members of Beta Epsilon Rho fraternity,of whom 60 are business majors and 50 are psych majors; 20 aremajoring in both business and psych. How many members of thefraternity are majoring in neither business nor psych?

Venn Diagram

Page 57: Finite Math Lecture Slides

Example: There are 100 members of Beta Epsilon Rho fraternity,of whom 60 are business majors and 50 are psych majors; 20 aremajoring in both business and psych. How many members of thefraternity are majoring in neither business nor psych?

Venn Diagram

Page 58: Finite Math Lecture Slides
Page 59: Finite Math Lecture Slides
Page 60: Finite Math Lecture Slides
Page 61: Finite Math Lecture Slides
Page 62: Finite Math Lecture Slides

Example: The 100 members of Beta Epsilon Rho fraternity areordering pizza. Pepperoni is requested by 80 members, andBrussels sprouts by 15; 12 members want neither pepperoni norBrussels sprouts. How many want both pepperoni and Brusselssprouts?

Venn Diagram

Page 63: Finite Math Lecture Slides

Example: The 100 members of Beta Epsilon Rho fraternity areordering pizza. Pepperoni is requested by 80 members, andBrussels sprouts by 15; 12 members want neither pepperoni norBrussels sprouts. How many want both pepperoni and Brusselssprouts?

Venn Diagram

Page 64: Finite Math Lecture Slides

Note that 12+? = 100, so ? = 88.

Page 65: Finite Math Lecture Slides

Now 80+? = 88, so ? = 8.

Page 66: Finite Math Lecture Slides

Now work across.

Page 67: Finite Math Lecture Slides

Answer: 7 people like pizza with pepperoni and Brussels sprouts.

Page 68: Finite Math Lecture Slides

Alternate Method

n[B] + n[P] = n[B ∪ P] + n[B ∩ P]15 + 80 = 88 +

= 7

Page 69: Finite Math Lecture Slides

Alternate Method

n[B] + n[P] = n[B ∪ P] + n[B ∩ P]

15 + 80 = 88 += 7

Page 70: Finite Math Lecture Slides

Alternate Method

n[B] + n[P] = n[B ∪ P] + n[B ∩ P]15 + 80 = 88 +

= 7

Page 71: Finite Math Lecture Slides

Alternate Method

n[B] + n[P] = n[B ∪ P] + n[B ∩ P]15 + 80 = 88 +

= 7

Page 72: Finite Math Lecture Slides

Inclusion-Exclusion Formula

In general,

n[A] + n[B] = n[A ∪ B] + n[A ∩ B]n[A ∪ B] = n[A] + n[B]− n[A ∩ B]n[A ∩ B] = n[A] + n[B]− n[A ∪ B].

Page 73: Finite Math Lecture Slides

Example: According to a survey of the members of an investmentclub, 150 own stock in Alcoa, 180 own stock in Boeing, 100 ownstock in Coke, 70 own stock in Alcoa and Boeing, 30 own stock inAlcoa and Coke, 50 own stock in Boeing and Coke, 20 own stockin all three companies, and 60 own stock in none of these ofcompanies.

1. How many members were surveyed?

2. How many of those surveyed do not own stock in Coke?

3. How many own stock in Boeing but not Alcoa?

4. How many own stock in Alcoa or Boeing and in Coke?

5. How many own stock in Alcoa or in Boeing and Coke?

6. How many own stock in Alcoa or Boeing but not in Coke?

7. How many own stock in exactly 1 company?

8. How many own stock in exactly 2 companies?

Page 74: Finite Math Lecture Slides

Example: According to a survey of the members of an investmentclub, 150 own stock in Alcoa, 180 own stock in Boeing, 100 ownstock in Coke, 70 own stock in Alcoa and Boeing, 30 own stock inAlcoa and Coke, 50 own stock in Boeing and Coke, 20 own stockin all three companies, and 60 own stock in none of these ofcompanies.

Venn Diagram

Page 75: Finite Math Lecture Slides

Work from the middle out.

Page 76: Finite Math Lecture Slides

Keep going.

Page 77: Finite Math Lecture Slides

Finish up.

Page 78: Finite Math Lecture Slides

Now we can start answering the questions.

Page 79: Finite Math Lecture Slides

How many members were surveyed?360

Page 80: Finite Math Lecture Slides

How many of those surveyed do not own stock in Coke?CP: 360− 100 = 260

Page 81: Finite Math Lecture Slides

How many own stock in Boeing but not Alcoa?n[B ∩ A′] = 80 + 30 = 110

Page 82: Finite Math Lecture Slides

How many own stock in Alcoa or Boeing and in Coke?n[(A ∪ B) ∩ C ] = 10 + 20 + 30 = 60

Page 83: Finite Math Lecture Slides

How many own stock in Alcoa or in Boeing and Coke?n[A ∪ (B ∩ C )] = 150 + 30 = 180

Page 84: Finite Math Lecture Slides

How many own stock in Alcoa or Boeing but not in Coke?n[(A ∪ B) ∩ C ′] = 70 + 50 + 80 = 200

Page 85: Finite Math Lecture Slides

How many own stock in exactly 1 company?70 + 80 + 40 = 190

Page 86: Finite Math Lecture Slides

How many own stock in exactly 2 companies?

10 + 30 + 50 = 90

Page 87: Finite Math Lecture Slides

How many own stock in exactly 2 companies?10 + 30 + 50 = 90

Page 88: Finite Math Lecture Slides

Example: Of the 360 members of the Investment Club, 180 ownstock in Alcoa (A), 180 own stock in Boeing (B), 120 own stock inCoke (C), 100 own stock in Alcoa and Boeing, 90 own stock inAlcoa and Coke, 70 own stock in Boeing and Coke, and 80 ownstock in none of these companies.

1. How many members own stock in all three companies?

2. How many members own stock in exactly one of Boeing andCoke?

3. How many own stock in Coke or in Boeing and Alcoa?

4. How many do not own stock in Boeing and Coke?

5. How many do not own stock in Alcoa or Coke?

Page 89: Finite Math Lecture Slides

Example: Of the 360 members of the Investment Club, 180 ownstock in Alcoa (A), 180 own stock in Boeing (B), 120 own stock inCoke (C), 100 own stock in Alcoa and Boeing, 90 own stock inAlcoa and Coke, 70 own stock in Boeing and Coke, and 80 ownstock in none of these companies.

Page 90: Finite Math Lecture Slides

Use the Inclusion-Exclusion formula on A ∪ B.

n[A ∪ B] = n[A] + n[B]− n[A ∩ B]= 180 + 180− 100 = 260

Page 91: Finite Math Lecture Slides

Now360 = 260 + 80 + ?

? = 20.

Page 92: Finite Math Lecture Slides

Now focus on C :

120 = 20 + 70 + ?? = 30.

Page 93: Finite Math Lecture Slides

Now focus on A ∩ C :

90 = 30 + ?? = 60.

Page 94: Finite Math Lecture Slides

Now that we have the center it’s easy to work from the middle out.

Page 95: Finite Math Lecture Slides

How many members own stock in all three companies? 60

Page 96: Finite Math Lecture Slides

How many members own stock in all three companies?

60

Page 97: Finite Math Lecture Slides

How many members own stock in all three companies? 60

Page 98: Finite Math Lecture Slides

How many members own stock in exactly one of Boeing and Coke?

40 + 70 + 30 + 20 = 160

Page 99: Finite Math Lecture Slides

How many members own stock in exactly one of Boeing and Coke?40 + 70 + 30 + 20 = 160

Page 100: Finite Math Lecture Slides

How many own stock in Coke or in Boeing and Alcoa?

120 + 40 = 160

Page 101: Finite Math Lecture Slides

How many own stock in Coke or in Boeing and Alcoa?120 + 40 = 160

Page 102: Finite Math Lecture Slides

How many do not own stock in Boeing and Coke?

CP: 360− 70 = 290

Page 103: Finite Math Lecture Slides

How many do not own stock in Boeing and Coke?CP: 360− 70 = 290

Page 104: Finite Math Lecture Slides

How many do not own stock in Alcoa or Coke?

CP: 360− 210 = 150

Page 105: Finite Math Lecture Slides

How many do not own stock in Alcoa or Coke?CP: 360− 210 = 150

Page 106: Finite Math Lecture Slides

Partitions

A partition of a set A isa division of A up into nonoverlapping pieces.

Example: IfA = {1, 2, 3, 4, 5, 6}

thenA1 = {1, 6}A2 = {2, 3, 5}A3 = {4}

is a partition of A.

Of course the AP (PP) automatically applies:

n[A] = n[A1] + n[A2] + n[A3].

Page 107: Finite Math Lecture Slides

Example: Suppose that n[A] = 120 and A is partitioned into 4subsets W , X , Y , and Z . Suppose further that W has 12elements, X and Y are the same size, and Z is twice as large as X .Find the sizes of X , Y , and Z .

A partition can always be diagrammed as a pizza.

Fill in the sizes using a variable if necessary.

Page 108: Finite Math Lecture Slides

Example: Suppose that n[A] = 120 and A is partitioned into 4subsets W , X , Y , and Z . Suppose further that W has 12elements, X and Y are the same size, and Z is twice as large as X .Find the sizes of X , Y , and Z .

A partition can always be diagrammed as a pizza.

Fill in the sizes using a variable if necessary.

Page 109: Finite Math Lecture Slides

Example: Suppose that n[A] = 120 and A is partitioned into 4subsets W , X , Y , and Z . Suppose further that W has 12elements, X and Y are the same size, and Z is twice as large as X .Find the sizes of X , Y , and Z .

A partition can always be diagrammed as a pizza.

Fill in the sizes using a variable if necessary.

Page 110: Finite Math Lecture Slides

Example: Suppose that n[A] = 120 and A is partitioned into 4subsets W , X , Y , and Z . Suppose further that W has 12elements, X and Y are the same size, and Z is twice as large as X .Find the sizes of X , Y , and Z .

X is the smallest piece, start with that and call its size x .

Y has the same size, and Zhas size 2x .

Now use the AP:

120 = 12 + x + x + 2x

108 = 4x

27 = x .

Page 111: Finite Math Lecture Slides

Example: Suppose that n[A] = 120 and A is partitioned into 4subsets W , X , Y , and Z . Suppose further that W has 12elements, X and Y are the same size, and Z is twice as large as X .Find the sizes of X , Y , and Z .

X is the smallest piece, start with that and call its size x .

Y has the same size, and Zhas size 2x .

Now use the AP:

120 = 12 + x + x + 2x

108 = 4x

27 = x .

Page 112: Finite Math Lecture Slides

Example: Suppose that n[A] = 120 and A is partitioned into 4subsets W , X , Y , and Z . Suppose further that W has 12elements, X and Y are the same size, and Z is twice as large as X .Find the sizes of X , Y , and Z .

X is the smallest piece, start with that and call its size x .

Y has the same size, and Zhas size 2x .

Now use the AP:

120 = 12 + x + x + 2x

108 = 4x

27 = x .

Page 113: Finite Math Lecture Slides

Example: Suppose that n[A] = 120 and A is partitioned into 4subsets W , X , Y , and Z . Suppose further that W has 12elements, X and Y are the same size, and Z is twice as large as X .Find the sizes of X , Y , and Z .

Now we can determine the other sizes.

x = 27

n[X ] = 27

n[Y ] = 27

n[Z ] = 54

Page 114: Finite Math Lecture Slides

Outcomes: What Matters?

Example 1: A sale table at a jewelry store has 5 bracelets, 4necklaces, and 8 pairs of earrings, all different and all at the sameprice. Shelly selects 3 items to purchase. How many outcomes arepossible?

Example 2: A sale table has 6 red scarves, 4 blue scarves, 6 greenscarves, and 6 purple scarves. Emily buys one of the red scarves,Kristen buys one of the blue scarves, Sara buys one of the greenscarves, and Laura buys one of the purple scarves. How manyoutcomes are possible?

Example 3: 6 IU students and 5 Purdue students go to the SummerIntern Job Fair in Indianapolis. Bank One, Neimann Marcus,Conseco, and the Lilly company each get one summer intern fromthis group. How many outcomes are possible?

Example 4: A sale catalogue lists 6 rock CD’s and 4 country CD’s.Each of Kim, Stacey, and Lynn picks out her favorite CD from thislist. How many outcomes are possible?

Page 115: Finite Math Lecture Slides

All the examples examples involve selecting some objects.

The details involve:

1. What objects are you choosing?(Domain)

2. Do different objects have different qualities that matter?(Qualities)

3. Do you do different things with different selected objects?(Roles)

4. Can the same object be selected multiple times?(Replacement)

Page 116: Finite Math Lecture Slides

All the examples examples involve selecting some objects.

The details involve:

1. What objects are you choosing?(Domain)

2. Do different objects have different qualities that matter?(Qualities)

3. Do you do different things with different selected objects?(Roles)

4. Can the same object be selected multiple times?(Replacement)

Page 117: Finite Math Lecture Slides

All the examples examples involve selecting some objects.

The details involve:

1. What objects are you choosing?(Domain)

2. Do different objects have different qualities that matter?(Qualities)

3. Do you do different things with different selected objects?(Roles)

4. Can the same object be selected multiple times?(Replacement)

Page 118: Finite Math Lecture Slides

All the examples examples involve selecting some objects.

The details involve:

1. What objects are you choosing?(Domain)

2. Do different objects have different qualities that matter?(Qualities)

3. Do you do different things with different selected objects?(Roles)

4. Can the same object be selected multiple times?(Replacement)

Page 119: Finite Math Lecture Slides

All the examples examples involve selecting some objects.

The details involve:

1. What objects are you choosing?(Domain)

2. Do different objects have different qualities that matter?(Qualities)

3. Do you do different things with different selected objects?(Roles)

4. Can the same object be selected multiple times?(Replacement)

Page 120: Finite Math Lecture Slides

All the examples examples involve selecting some objects.

The details involve:

1. What objects are you choosing?(Domain)

2. Do different objects have different qualities that matter?(Qualities)

3. Do you do different things with different selected objects?(Roles)

4. Can the same object be selected multiple times?(Replacement)

Page 121: Finite Math Lecture Slides

All the examples examples involve selecting some objects.

The details involve:

1. What objects are you choosing?(Domain)

2. Do different objects have different qualities that matter?(Qualities)

3. Do you do different things with different selected objects?(Roles)

4. Can the same object be selected multiple times?(Replacement)

Page 122: Finite Math Lecture Slides

All the examples examples involve selecting some objects.

The details involve:

1. What objects are you choosing?(Domain)

2. Do different objects have different qualities that matter?(Qualities)

3. Do you do different things with different selected objects?(Roles)

4. Can the same object be selected multiple times?(Replacement)

Page 123: Finite Math Lecture Slides

All the examples examples involve selecting some objects.

The details involve:

1. What objects are you choosing?(Domain)

2. Do different objects have different qualities that matter?(Qualities)

3. Do you do different things with different selected objects?(Roles)

4. Can the same object be selected multiple times?(Replacement)

Page 124: Finite Math Lecture Slides

Example: A sale table at a jewelry store has 5 bracelets, 4necklaces, and 8 pairs of earrings, all different and all at the sameprice. Gladys selects 3 items to purchase. How many outcomes arepossible?

Analysis:

1. Domain: 17 items of jewelry.

2. Qualities: none that matter.

3. Roles: none.

4. Replacement: no.

Page 125: Finite Math Lecture Slides

Example: A sale table at a jewelry store has 5 bracelets, 4necklaces, and 8 pairs of earrings, all different and all at the sameprice. Gladys selects 3 items to purchase. How many outcomes arepossible?

Analysis:

1. Domain: 17 items of jewelry.

2. Qualities: none that matter.

3. Roles: none.

4. Replacement: no.

Page 126: Finite Math Lecture Slides

Example: A sale table at a jewelry store has 5 bracelets, 4necklaces, and 8 pairs of earrings, all different and all at the sameprice. Gladys selects 3 items to purchase. How many outcomes arepossible?

Analysis:

1. Domain: 17 items of jewelry.

2. Qualities: none that matter.

3. Roles: none.

4. Replacement: no.

Page 127: Finite Math Lecture Slides

Example: A sale table at a jewelry store has 5 bracelets, 4necklaces, and 8 pairs of earrings, all different and all at the sameprice. Gladys selects 3 items to purchase. How many outcomes arepossible?

Analysis:

1. Domain: 17 items of jewelry.

2. Qualities: none that matter.

3. Roles: none.

4. Replacement: no.

Page 128: Finite Math Lecture Slides

Example: A sale table at a jewelry store has 5 bracelets, 4necklaces, and 8 pairs of earrings, all different and all at the sameprice. Gladys selects 3 items to purchase. How many outcomes arepossible?

Analysis:

1. Domain: 17 items of jewelry.

2. Qualities: none that matter.

3. Roles: none.

4. Replacement: no.

Page 129: Finite Math Lecture Slides

Example: A sale table at a jewelry store has 5 bracelets, 4necklaces, and 8 pairs of earrings, all different and all at the sameprice. Gladys selects 3 items to purchase. How many outcomes arepossible?

Analysis:

1. Domain: 17 items of jewelry.

2. Qualities: none that matter.

3. Roles: none.

4. Replacement: no.

Page 130: Finite Math Lecture Slides

Example: A sale table at a jewelry store has 5 bracelets, 4necklaces, and 8 pairs of earrings, all different and all at the sameprice. Gladys selects 3 items to purchase. How many outcomes arepossible?

Analysis:

1. Domain: 17 items of jewelry.

2. Qualities: none that matter.

3. Roles: none.

4. Replacement: no.

Page 131: Finite Math Lecture Slides

Example: A sale table at a jewelry store has 5 bracelets, 4necklaces, and 8 pairs of earrings, all different and all at the sameprice. Gladys selects 3 items to purchase. How many outcomes arepossible?

Analysis:

1. Domain: 17 items of jewelry.

2. Qualities: none that matter.

3. Roles: none.

4. Replacement: no.

Page 132: Finite Math Lecture Slides

Example: A sale table at a jewelry store has 5 bracelets, 4necklaces, and 8 pairs of earrings, all different and all at the sameprice. Gladys selects 3 items to purchase. How many outcomes arepossible?

Analysis:

1. Domain: 17 items of jewelry.

2. Qualities: none that matter.

3. Roles: none.

4. Replacement: no.

Page 133: Finite Math Lecture Slides

Example: A sale table has 6 red scarves, 4 blue scarves, 6 greenscarves, and 6 purple scarves. Emily buys one of the red scarves,Kristen buys one of the blue scarves, Sara buys one of the greenscarves, and Laura buys one of the purple scarves. How manyoutcomes are possible?

Analysis:

1. Domain: 22 scarves.

2. Qualities: red, blue, green, and purple.

3. Roles: Emily, Kristen, Sara, and Laura.

4. Replacement: no.

Page 134: Finite Math Lecture Slides

Example: A sale table has 6 red scarves, 4 blue scarves, 6 greenscarves, and 6 purple scarves. Emily buys one of the red scarves,Kristen buys one of the blue scarves, Sara buys one of the greenscarves, and Laura buys one of the purple scarves. How manyoutcomes are possible?

Analysis:

1. Domain: 22 scarves.

2. Qualities: red, blue, green, and purple.

3. Roles: Emily, Kristen, Sara, and Laura.

4. Replacement: no.

Page 135: Finite Math Lecture Slides

Example: A sale table has 6 red scarves, 4 blue scarves, 6 greenscarves, and 6 purple scarves. Emily buys one of the red scarves,Kristen buys one of the blue scarves, Sara buys one of the greenscarves, and Laura buys one of the purple scarves. How manyoutcomes are possible?

Analysis:

1. Domain: 22 scarves.

2. Qualities: red, blue, green, and purple.

3. Roles: Emily, Kristen, Sara, and Laura.

4. Replacement: no.

Page 136: Finite Math Lecture Slides

Example: A sale table has 6 red scarves, 4 blue scarves, 6 greenscarves, and 6 purple scarves. Emily buys one of the red scarves,Kristen buys one of the blue scarves, Sara buys one of the greenscarves, and Laura buys one of the purple scarves. How manyoutcomes are possible?

Analysis:

1. Domain: 22 scarves.

2. Qualities: red, blue, green, and purple.

3. Roles: Emily, Kristen, Sara, and Laura.

4. Replacement: no.

Page 137: Finite Math Lecture Slides

Example: A sale table has 6 red scarves, 4 blue scarves, 6 greenscarves, and 6 purple scarves. Emily buys one of the red scarves,Kristen buys one of the blue scarves, Sara buys one of the greenscarves, and Laura buys one of the purple scarves. How manyoutcomes are possible?

Analysis:

1. Domain: 22 scarves.

2. Qualities: red, blue, green, and purple.

3. Roles: Emily, Kristen, Sara, and Laura.

4. Replacement: no.

Page 138: Finite Math Lecture Slides

Example: A sale table has 6 red scarves, 4 blue scarves, 6 greenscarves, and 6 purple scarves. Emily buys one of the red scarves,Kristen buys one of the blue scarves, Sara buys one of the greenscarves, and Laura buys one of the purple scarves. How manyoutcomes are possible?

Analysis:

1. Domain: 22 scarves.

2. Qualities: red, blue, green, and purple.

3. Roles: Emily, Kristen, Sara, and Laura.

4. Replacement: no.

Page 139: Finite Math Lecture Slides

Example: A sale table has 6 red scarves, 4 blue scarves, 6 greenscarves, and 6 purple scarves. Emily buys one of the red scarves,Kristen buys one of the blue scarves, Sara buys one of the greenscarves, and Laura buys one of the purple scarves. How manyoutcomes are possible?

Analysis:

1. Domain: 22 scarves.

2. Qualities: red, blue, green, and purple.

3. Roles: Emily, Kristen, Sara, and Laura.

4. Replacement: no.

Page 140: Finite Math Lecture Slides

Example: A sale table has 6 red scarves, 4 blue scarves, 6 greenscarves, and 6 purple scarves. Emily buys one of the red scarves,Kristen buys one of the blue scarves, Sara buys one of the greenscarves, and Laura buys one of the purple scarves. How manyoutcomes are possible?

Analysis:

1. Domain: 22 scarves.

2. Qualities: red, blue, green, and purple.

3. Roles: Emily, Kristen, Sara, and Laura.

4. Replacement: no.

Page 141: Finite Math Lecture Slides

Example: A sale table has 6 red scarves, 4 blue scarves, 6 greenscarves, and 6 purple scarves. Emily buys one of the red scarves,Kristen buys one of the blue scarves, Sara buys one of the greenscarves, and Laura buys one of the purple scarves. How manyoutcomes are possible?

Analysis:

1. Domain: 22 scarves.

2. Qualities: red, blue, green, and purple.

3. Roles: Emily, Kristen, Sara, and Laura.

4. Replacement: no.

Page 142: Finite Math Lecture Slides

Example: 6 IU students and 5 Purdue students go to the SummerIntern Job Fair in Indianapolis. Bank One, Neimann Marcus,Conseco, and the Lilly company each get one summer intern fromthis group. How many outcomes are possible?

Analysis:

1. Domain: 11 students.

2. Qualities: none that matter.

3. Roles: Bank One, Neimann Marcus, Conseco, and Lilly.

4. Replacement: no.

Page 143: Finite Math Lecture Slides

Example: 6 IU students and 5 Purdue students go to the SummerIntern Job Fair in Indianapolis. Bank One, Neimann Marcus,Conseco, and the Lilly company each get one summer intern fromthis group. How many outcomes are possible?

Analysis:

1. Domain: 11 students.

2. Qualities: none that matter.

3. Roles: Bank One, Neimann Marcus, Conseco, and Lilly.

4. Replacement: no.

Page 144: Finite Math Lecture Slides

Example: 6 IU students and 5 Purdue students go to the SummerIntern Job Fair in Indianapolis. Bank One, Neimann Marcus,Conseco, and the Lilly company each get one summer intern fromthis group. How many outcomes are possible?

Analysis:

1. Domain: 11 students.

2. Qualities: none that matter.

3. Roles: Bank One, Neimann Marcus, Conseco, and Lilly.

4. Replacement: no.

Page 145: Finite Math Lecture Slides

Example: 6 IU students and 5 Purdue students go to the SummerIntern Job Fair in Indianapolis. Bank One, Neimann Marcus,Conseco, and the Lilly company each get one summer intern fromthis group. How many outcomes are possible?

Analysis:

1. Domain: 11 students.

2. Qualities: none that matter.

3. Roles: Bank One, Neimann Marcus, Conseco, and Lilly.

4. Replacement: no.

Page 146: Finite Math Lecture Slides

Example: 6 IU students and 5 Purdue students go to the SummerIntern Job Fair in Indianapolis. Bank One, Neimann Marcus,Conseco, and the Lilly company each get one summer intern fromthis group. How many outcomes are possible?

Analysis:

1. Domain: 11 students.

2. Qualities: none that matter.

3. Roles: Bank One, Neimann Marcus, Conseco, and Lilly.

4. Replacement: no.

Page 147: Finite Math Lecture Slides

Example: 6 IU students and 5 Purdue students go to the SummerIntern Job Fair in Indianapolis. Bank One, Neimann Marcus,Conseco, and the Lilly company each get one summer intern fromthis group. How many outcomes are possible?

Analysis:

1. Domain: 11 students.

2. Qualities: none that matter.

3. Roles: Bank One, Neimann Marcus, Conseco, and Lilly.

4. Replacement: no.

Page 148: Finite Math Lecture Slides

Example: 6 IU students and 5 Purdue students go to the SummerIntern Job Fair in Indianapolis. Bank One, Neimann Marcus,Conseco, and the Lilly company each get one summer intern fromthis group. How many outcomes are possible?

Analysis:

1. Domain: 11 students.

2. Qualities: none that matter.

3. Roles: Bank One, Neimann Marcus, Conseco, and Lilly.

4. Replacement: no.

Page 149: Finite Math Lecture Slides

Example: 6 IU students and 5 Purdue students go to the SummerIntern Job Fair in Indianapolis. Bank One, Neimann Marcus,Conseco, and the Lilly company each get one summer intern fromthis group. How many outcomes are possible?

Analysis:

1. Domain: 11 students.

2. Qualities: none that matter.

3. Roles: Bank One, Neimann Marcus, Conseco, and Lilly.

4. Replacement: no.

Page 150: Finite Math Lecture Slides

Example: 6 IU students and 5 Purdue students go to the SummerIntern Job Fair in Indianapolis. Bank One, Neimann Marcus,Conseco, and the Lilly company each get one summer intern fromthis group. How many outcomes are possible?

Analysis:

1. Domain: 11 students.

2. Qualities: none that matter.

3. Roles: Bank One, Neimann Marcus, Conseco, and Lilly.

4. Replacement: no.

Page 151: Finite Math Lecture Slides

Example: A sale catalogue lists 6 rock CD’s and 4 country CD’s.Each of Kim, Stacey, and Lynn picks out her favorite CD from thislist. How many outcomes are possible?

Analysis:

1. Domain: 10 CDs.

2. Qualities: none that matter.

3. Roles: Kim, Stacey, and Lynn.

4. Replacement: yes.

Page 152: Finite Math Lecture Slides

Example: A sale catalogue lists 6 rock CD’s and 4 country CD’s.Each of Kim, Stacey, and Lynn picks out her favorite CD from thislist. How many outcomes are possible?

Analysis:

1. Domain: 10 CDs.

2. Qualities: none that matter.

3. Roles: Kim, Stacey, and Lynn.

4. Replacement: yes.

Page 153: Finite Math Lecture Slides

Example: A sale catalogue lists 6 rock CD’s and 4 country CD’s.Each of Kim, Stacey, and Lynn picks out her favorite CD from thislist. How many outcomes are possible?

Analysis:

1. Domain: 10 CDs.

2. Qualities: none that matter.

3. Roles: Kim, Stacey, and Lynn.

4. Replacement: yes.

Page 154: Finite Math Lecture Slides

Example: A sale catalogue lists 6 rock CD’s and 4 country CD’s.Each of Kim, Stacey, and Lynn picks out her favorite CD from thislist. How many outcomes are possible?

Analysis:

1. Domain: 10 CDs.

2. Qualities: none that matter.

3. Roles: Kim, Stacey, and Lynn.

4. Replacement: yes.

Page 155: Finite Math Lecture Slides

Example: A sale catalogue lists 6 rock CD’s and 4 country CD’s.Each of Kim, Stacey, and Lynn picks out her favorite CD from thislist. How many outcomes are possible?

Analysis:

1. Domain: 10 CDs.

2. Qualities: none that matter.

3. Roles: Kim, Stacey, and Lynn.

4. Replacement: yes.

Page 156: Finite Math Lecture Slides

Example: A sale catalogue lists 6 rock CD’s and 4 country CD’s.Each of Kim, Stacey, and Lynn picks out her favorite CD from thislist. How many outcomes are possible?

Analysis:

1. Domain: 10 CDs.

2. Qualities: none that matter.

3. Roles: Kim, Stacey, and Lynn.

4. Replacement: yes.

Page 157: Finite Math Lecture Slides

Example: A sale catalogue lists 6 rock CD’s and 4 country CD’s.Each of Kim, Stacey, and Lynn picks out her favorite CD from thislist. How many outcomes are possible?

Analysis:

1. Domain: 10 CDs.

2. Qualities: none that matter.

3. Roles: Kim, Stacey, and Lynn.

4. Replacement: yes.

Page 158: Finite Math Lecture Slides

Example: A sale catalogue lists 6 rock CD’s and 4 country CD’s.Each of Kim, Stacey, and Lynn picks out her favorite CD from thislist. How many outcomes are possible?

Analysis:

1. Domain: 10 CDs.

2. Qualities: none that matter.

3. Roles: Kim, Stacey, and Lynn.

4. Replacement: yes.

Page 159: Finite Math Lecture Slides

Example: A sale catalogue lists 6 rock CD’s and 4 country CD’s.Each of Kim, Stacey, and Lynn picks out her favorite CD from thislist. How many outcomes are possible?

Analysis:

1. Domain: 10 CDs.

2. Qualities: none that matter.

3. Roles: Kim, Stacey, and Lynn.

4. Replacement: yes.

Page 160: Finite Math Lecture Slides

Shorthand

We’ll need a shorthand to use for analyzing problems.

We’ll use D to stand for all possible choices of object.

We’ll use S to stand for all possible outcomes.

So if you have to pick one tomato out of 40, let

D = 40 tomatoes

and then the set S of all possible outcomes can be represented by

role: Tomatoquality: D

how many: 1.

That was the simplest possible example.

Page 161: Finite Math Lecture Slides

Shorthand

We’ll need a shorthand to use for analyzing problems.

We’ll use D to stand for all possible choices of object.

We’ll use S to stand for all possible outcomes.

So if you have to pick one tomato out of 40, let

D = 40 tomatoes

and then the set S of all possible outcomes can be represented by

role: Tomatoquality: D

how many: 1.

That was the simplest possible example.

Page 162: Finite Math Lecture Slides

Shorthand

We’ll need a shorthand to use for analyzing problems.

We’ll use D to stand for all possible choices of object.

We’ll use S to stand for all possible outcomes.

So if you have to pick one tomato out of 40, let

D = 40 tomatoes

and then the set S of all possible outcomes can be represented by

role: Tomatoquality: D

how many: 1.

That was the simplest possible example.

Page 163: Finite Math Lecture Slides

Shorthand

We’ll need a shorthand to use for analyzing problems.

We’ll use D to stand for all possible choices of object.

We’ll use S to stand for all possible outcomes.

So if you have to pick one tomato out of 40, let

D = 40 tomatoes

and then the set S of all possible outcomes can be represented by

role: Tomatoquality: D

how many: 1.

That was the simplest possible example.

Page 164: Finite Math Lecture Slides

Now suppose you must choose two tomatoes.

Roles don’t matter (both tomatoes suffer the same fate).

No Replacements (you can’t eat the same tomato twice).

We’ll use to indicate that there are no replacements.

ThenT = 40 tomatoes

and S is described by

T

2.

Note the role was omitted,because there is only one.

Page 165: Finite Math Lecture Slides

Now suppose you must choose two tomatoes.

Roles don’t matter (both tomatoes suffer the same fate).

No Replacements (you can’t eat the same tomato twice).

We’ll use to indicate that there are no replacements.

ThenT = 40 tomatoes

and S is described by

T

2.

Note the role was omitted,because there is only one.

Page 166: Finite Math Lecture Slides

Now suppose you must choose two tomatoes.

Roles don’t matter (both tomatoes suffer the same fate).

No Replacements (you can’t eat the same tomato twice).

We’ll use to indicate that there are no replacements.

ThenT = 40 tomatoes

and S is described by

T

2.

Note the role was omitted,because there is only one.

Page 167: Finite Math Lecture Slides

Now suppose you must choose two tomatoes.

Roles don’t matter (both tomatoes suffer the same fate).

No Replacements (you can’t eat the same tomato twice).

We’ll use to indicate that there are no replacements.

ThenT = 40 tomatoes

and S is described by

T

2.

Note the role was omitted,because there is only one.

Page 168: Finite Math Lecture Slides

Now suppose you must choose two tomatoes.

Roles don’t matter (both tomatoes suffer the same fate).

No Replacements (you can’t eat the same tomato twice).

We’ll use to indicate that there are no replacements.

ThenT = 40 tomatoes

and S is described by

T

2.

Note the role was omitted,because there is only one.

Page 169: Finite Math Lecture Slides

Now suppose you also need an onion,

and that there are 50 onions to choose from.

That’s a different quality .

I’ll denote the tomatoes by T and the onions by O.

Then S is described by

T O

2 1.

Page 170: Finite Math Lecture Slides

Now suppose you also need an onion,

and that there are 50 onions to choose from.

That’s a different quality .

I’ll denote the tomatoes by T and the onions by O.

Then S is described by

T O

2 1.

Page 171: Finite Math Lecture Slides

Now suppose you also need an onion,

and that there are 50 onions to choose from.

That’s a different quality .

I’ll denote the tomatoes by T and the onions by O.

Then S is described by

T O

2 1.

Page 172: Finite Math Lecture Slides

Now suppose that you need one tomato for a marinara sauce, andthe other tomato and onion for a salad.

Now there are two roles, sauce and salad.

Then S is described by

Sauce SaladT T O

1 1 1

.

Page 173: Finite Math Lecture Slides

Now suppose that you need one tomato for a marinara sauce, andthe other tomato and onion for a salad.

Now there are two roles, sauce and salad.

Then S is described by

Sauce SaladT T O

1 1 1

.

Page 174: Finite Math Lecture Slides

Example: A sale table at a jewelry store has 5 bracelets, 4necklaces, and 8 pairs of earrings, all different and all at the sameprice. Gladys selects 3 items to purchase. How many outcomes arepossible?

Analysis:

1. Domain: 17 items of jewelry.

2. Qualities: none that matter.

3. Roles: none.

4. Replacement: no.

Therefore,D = 17 articles of jewelry

and S = D

3.

Page 175: Finite Math Lecture Slides

Example: A sale table at a jewelry store has 5 bracelets, 4necklaces, and 8 pairs of earrings, all different and all at the sameprice. Gladys selects 3 items to purchase. How many outcomes arepossible?

Analysis:

1. Domain: 17 items of jewelry.

2. Qualities: none that matter.

3. Roles: none.

4. Replacement: no.

Therefore,D = 17 articles of jewelry

and S = D

3.

Page 176: Finite Math Lecture Slides

Example: A sale table has 6 red scarves, 4 blue scarves, 6 greenscarves, and 6 purple scarves. Emily buys one of the red scarves,Kristen buys one of the blue scarves, Sara buys one of the greenscarves, and Laura buys one of the purple scarves. How manyoutcomes are possible?

Analysis:

1. Domain: 22 scarves.2. Qualities: red, blue, green, and purple.3. Roles: Emily, Kristen, Sara, and Laura.4. Replacement: no.

Therefore,

R = 6 red scarves B = 4 blue scarvesG = 6 green scarves P = 6 purple scarves

and S =

Emily Kristen Sara LauraR B G P

1 1 1 1

.

Page 177: Finite Math Lecture Slides

Example: A sale table has 6 red scarves, 4 blue scarves, 6 greenscarves, and 6 purple scarves. Emily buys one of the red scarves,Kristen buys one of the blue scarves, Sara buys one of the greenscarves, and Laura buys one of the purple scarves. How manyoutcomes are possible?

Analysis:

1. Domain: 22 scarves.2. Qualities: red, blue, green, and purple.3. Roles: Emily, Kristen, Sara, and Laura.4. Replacement: no.

Therefore,

R = 6 red scarves B = 4 blue scarvesG = 6 green scarves P = 6 purple scarves

and S =

Emily Kristen Sara LauraR B G P

1 1 1 1

.

Page 178: Finite Math Lecture Slides

Example: 6 IU students and 5 Purdue students go to the SummerIntern Job Fair in Indianapolis. Bank One, Neimann Marcus,Conseco, and the Lilly company each get one summer intern fromthis group. How many outcomes are possible?

Analysis:

1. Domain: 11 students.

2. Qualities: none that matter.

3. Roles: Bank One, Neimann Marcus, Conseco, and Lilly.

4. Replacement: no.

Therefore,D = 11 students

and S =

Bank1 Neimann Conseco LillyD D D D

1 1 1 1

.

Page 179: Finite Math Lecture Slides

Example: 6 IU students and 5 Purdue students go to the SummerIntern Job Fair in Indianapolis. Bank One, Neimann Marcus,Conseco, and the Lilly company each get one summer intern fromthis group. How many outcomes are possible?

Analysis:

1. Domain: 11 students.

2. Qualities: none that matter.

3. Roles: Bank One, Neimann Marcus, Conseco, and Lilly.

4. Replacement: no.

Therefore,D = 11 students

and S =

Bank1 Neimann Conseco LillyD D D D

1 1 1 1

.

Page 180: Finite Math Lecture Slides

Example: A sale catalogue lists 6 rock CD’s and 4 country CD’s.Each of Kim, Stacey, and Lynn picks out her favorite CD from thislist. How many outcomes are possible?

Analysis:

1. Domain: 10 CDs.

2. Qualities: none that matter.

3. Roles: Kim, Stacey, and Lynn.

4. Replacement: yes.

Therefore,D = 10 CDs

and S =

Kim Stacey LynnD D D

1 1 1

.

Page 181: Finite Math Lecture Slides

Example: A sale catalogue lists 6 rock CD’s and 4 country CD’s.Each of Kim, Stacey, and Lynn picks out her favorite CD from thislist. How many outcomes are possible?

Analysis:

1. Domain: 10 CDs.

2. Qualities: none that matter.

3. Roles: Kim, Stacey, and Lynn.

4. Replacement: yes.

Therefore,D = 10 CDs

and S =

Kim Stacey LynnD D D

1 1 1

.

Page 182: Finite Math Lecture Slides

ParadigmsParadigm: City Council Committee: A city council has 5Democratic members (Anne, Bill, Cathy, Dan, and Ellen) and 3Republican members (Frank, Gina, and Hank). The mayor is toappoint a committee consisting of 3 council members.

Analysis:

1. Domain: 8 council members

2. Qualities: none that matter

3. Roles: none that matter

4. Replacement: no

Therefore,D = 8 council members

and S =

D

3.

Page 183: Finite Math Lecture Slides

ParadigmsParadigm: City Council Committee: A city council has 5Democratic members (Anne, Bill, Cathy, Dan, and Ellen) and 3Republican members (Frank, Gina, and Hank). The mayor is toappoint a committee consisting of 3 council members.

Analysis:

1. Domain: 8 council members

2. Qualities: none that matter

3. Roles: none that matter

4. Replacement: no

Therefore,D = 8 council members

and S =

D

3.

Page 184: Finite Math Lecture Slides

ParadigmsParadigm: City Council Committee: A city council has 5Democratic members (Anne, Bill, Cathy, Dan, and Ellen) and 3Republican members (Frank, Gina, and Hank). The mayor is toappoint a committee consisting of 3 council members.

Analysis:

1. Domain: 8 council members

2. Qualities: none that matter

3. Roles: none that matter

4. Replacement: no

Therefore,D = 8 council members

and S =

D

3.

Page 185: Finite Math Lecture Slides

Paradigm: Soup, Salad, Entree, Veg: For dinner, a customer at arestaurant must select one of 3 soups, one of 5 salads, one of 4entrees, and one of 6 vegetables (see the menu below).

Analysis:

1. Domain: 18 menu items2. Qualities: 3 soups, 5 salads, 4 entrees, 6 vegetables3. Roles: soup, salad, entree, vegetable4. Replacement: no (n/a)

Therefore,

P = 3 soups L = 5 saladsE = 4 entrees V = 6 vegetables

and S =

P L E V

1 1 1 1.

Another notation for this is the Cartesian Product:

S = P × L× E × V

Page 186: Finite Math Lecture Slides

Paradigm: Soup, Salad, Entree, Veg: For dinner, a customer at arestaurant must select one of 3 soups, one of 5 salads, one of 4entrees, and one of 6 vegetables (see the menu below).

Analysis:

1. Domain: 18 menu items2. Qualities: 3 soups, 5 salads, 4 entrees, 6 vegetables3. Roles: soup, salad, entree, vegetable4. Replacement: no (n/a)

Therefore,

P = 3 soups L = 5 saladsE = 4 entrees V = 6 vegetables

and S =

P L E V

1 1 1 1.

Another notation for this is the Cartesian Product:

S = P × L× E × V

Page 187: Finite Math Lecture Slides

Paradigm: Soup, Salad, Entree, Veg: For dinner, a customer at arestaurant must select one of 3 soups, one of 5 salads, one of 4entrees, and one of 6 vegetables (see the menu below).

Analysis:

1. Domain: 18 menu items2. Qualities: 3 soups, 5 salads, 4 entrees, 6 vegetables3. Roles: soup, salad, entree, vegetable4. Replacement: no (n/a)

Therefore,

P = 3 soups L = 5 saladsE = 4 entrees V = 6 vegetables

and S =

P L E V

1 1 1 1.

Another notation for this is the Cartesian Product:

S = P × L× E × V

Page 188: Finite Math Lecture Slides

Paradigm: Soup, Salad, Entree, Veg: For dinner, a customer at arestaurant must select one of 3 soups, one of 5 salads, one of 4entrees, and one of 6 vegetables (see the menu below).

Analysis:

1. Domain: 18 menu items2. Qualities: 3 soups, 5 salads, 4 entrees, 6 vegetables3. Roles: soup, salad, entree, vegetable4. Replacement: no (n/a)

Therefore,

P = 3 soups L = 5 saladsE = 4 entrees V = 6 vegetables

and S =

P L E V

1 1 1 1.

Another notation for this is the Cartesian Product:

S = P × L× E × V

Page 189: Finite Math Lecture Slides

Paradigm: Coffee, Tea, Coke, Sprite: John, Mary, and Bill musteach select one of four beverages for dinner (for instance, coffee,tea, coke, or sprite).

Analysis:

1. Domain: 4 beverages

2. Qualities: none that matter

3. Roles: John, Mary, and Bill

4. Replacement: yes

Therefore,D = 4 beverages

and S =

John Mary BillD D D

1 1 1

.

Page 190: Finite Math Lecture Slides

Paradigm: Coffee, Tea, Coke, Sprite: John, Mary, and Bill musteach select one of four beverages for dinner (for instance, coffee,tea, coke, or sprite).

Analysis:

1. Domain: 4 beverages

2. Qualities: none that matter

3. Roles: John, Mary, and Bill

4. Replacement: yes

Therefore,D = 4 beverages

and S =

John Mary BillD D D

1 1 1

.

Page 191: Finite Math Lecture Slides

Paradigm: Coffee, Tea, Coke, Sprite: John, Mary, and Bill musteach select one of four beverages for dinner (for instance, coffee,tea, coke, or sprite).

Analysis:

1. Domain: 4 beverages

2. Qualities: none that matter

3. Roles: John, Mary, and Bill

4. Replacement: yes

Therefore,D = 4 beverages

and S =

John Mary BillD D D

1 1 1

.

Page 192: Finite Math Lecture Slides

Paradigm: Casting a Play: A play has three female roles: thegrandmother, the mother, and the daughter. Six (6) women —Anne, Barb, Carol, Debra, Ellen, and Fran — audition for theseroles. (One should presume that no one can fill two roles, becauseall three characters may have to be on stage simultaneously.)

Analysis:

1. Domain: 6 Actresses

2. Qualities: none that matter

3. Roles: Grandmother, Mother, Daughter

4. Replacement: no

Therefore,D = 6 Actresses

and S =

Grandmother Mother DaughterD D D

1 1 1

.

Page 193: Finite Math Lecture Slides

Paradigm: Casting a Play: A play has three female roles: thegrandmother, the mother, and the daughter. Six (6) women —Anne, Barb, Carol, Debra, Ellen, and Fran — audition for theseroles. (One should presume that no one can fill two roles, becauseall three characters may have to be on stage simultaneously.)

Analysis:

1. Domain: 6 Actresses

2. Qualities: none that matter

3. Roles: Grandmother, Mother, Daughter

4. Replacement: no

Therefore,D = 6 Actresses

and S =

Grandmother Mother DaughterD D D

1 1 1

.

Page 194: Finite Math Lecture Slides

Paradigm: Casting a Play: A play has three female roles: thegrandmother, the mother, and the daughter. Six (6) women —Anne, Barb, Carol, Debra, Ellen, and Fran — audition for theseroles. (One should presume that no one can fill two roles, becauseall three characters may have to be on stage simultaneously.)

Analysis:

1. Domain: 6 Actresses

2. Qualities: none that matter

3. Roles: Grandmother, Mother, Daughter

4. Replacement: no

Therefore,D = 6 Actresses

and S =

Grandmother Mother DaughterD D D

1 1 1

.

Page 195: Finite Math Lecture Slides

Paradigm

City Council Soup, Salad, Coffee, Tea, CastingCommittee Entree, Veg Coke, Sprite a Play

Qualities no yes no no

Roles no yes yes yes

Replacement no no (n/a) yes no

Page 196: Finite Math Lecture Slides

Example: At dinner one night at summer camp, 8 campers and acounselor are sitting at a long table. The counselor must select 3of the campers to carry the dishes to the cleanup window. Howmany overall outcomes are possible?

Analysis:

1. Domain: 8 campers(Counseler? Apply Creative Tunnel Vision.)

2. Qualities: none that matter

3. Roles: none

4. Replacement: no

City Council Committee Paradigm

Therefore,D = 8 campers

and S = D

3.

Page 197: Finite Math Lecture Slides

Example: At dinner one night at summer camp, 8 campers and acounselor are sitting at a long table. The counselor must select 3of the campers to carry the dishes to the cleanup window. Howmany overall outcomes are possible?

Analysis:

1. Domain: 8 campers(Counseler? Apply Creative Tunnel Vision.)

2. Qualities: none that matter

3. Roles: none

4. Replacement: no

City Council Committee Paradigm

Therefore,D = 8 campers

and S = D

3.

Page 198: Finite Math Lecture Slides

Example: At dinner one night at summer camp, 8 campers and acounselor are sitting at a long table. The counselor must select 3of the campers to carry the dishes to the cleanup window. Howmany overall outcomes are possible?

Analysis:

1. Domain: 8 campers(Counseler? Apply Creative Tunnel Vision.)

2. Qualities: none that matter

3. Roles: none

4. Replacement: no

City Council Committee Paradigm

Therefore,D = 8 campers

and S = D

3.

Page 199: Finite Math Lecture Slides

Example: At dinner one night at summer camp, 8 campers and acounselor are sitting at a long table. The counselor must select 3of the campers to carry the dishes to the cleanup window. Howmany overall outcomes are possible?

Analysis:

1. Domain: 8 campers(Counseler? Apply Creative Tunnel Vision.)

2. Qualities: none that matter

3. Roles: none

4. Replacement: no

City Council Committee Paradigm

Therefore,D = 8 campers

and S = D

3.

Page 200: Finite Math Lecture Slides

Example: At dinner one night at summer camp, 8 campers and acounselor are sitting at a long table. The counselor must select 3of the campers to carry the dishes to the cleanup window. Howmany overall outcomes are possible?

Alternate Analysis:

1. Domain: 9 people at dinner

2. Qualities: 8 campers, 1 counseler

3. Roles: none

4. Replacement: no

Therefore,D1 = 8 campersD2 = 1 counseler

and S = D1

3.

Page 201: Finite Math Lecture Slides

Example: A voter must select one city council candidate, oneschool board candidate, and one county commissioner candidate.There are 4 candidates for the city council, 9 candidates for theschool board, and 5 candidates for county commissioner.

Analysis:

1. Domain: 18 candidates

2. Qualities: city council, school board, county commission

3. Roles: city council, school board, county commission

4. Replacement: no

Soup, Salad, Entree, Veg Paradigm

Therefore,

D1 = 4 city council candidatesD2 = 9 school board candidatesD3 = 5 county commission candidates

D1 D2 D3

1 1 1.

Page 202: Finite Math Lecture Slides

Example: A voter must select one city council candidate, oneschool board candidate, and one county commissioner candidate.There are 4 candidates for the city council, 9 candidates for theschool board, and 5 candidates for county commissioner.

Analysis:

1. Domain: 18 candidates

2. Qualities: city council, school board, county commission

3. Roles: city council, school board, county commission

4. Replacement: no

Soup, Salad, Entree, Veg Paradigm

Therefore,

D1 = 4 city council candidatesD2 = 9 school board candidatesD3 = 5 county commission candidates

D1 D2 D3

1 1 1.

Page 203: Finite Math Lecture Slides

Example: A voter must select one city council candidate, oneschool board candidate, and one county commissioner candidate.There are 4 candidates for the city council, 9 candidates for theschool board, and 5 candidates for county commissioner.

Analysis:

1. Domain: 18 candidates

2. Qualities: city council, school board, county commission

3. Roles: city council, school board, county commission

4. Replacement: no

Soup, Salad, Entree, Veg Paradigm

Therefore,

D1 = 4 city council candidatesD2 = 9 school board candidatesD3 = 5 county commission candidates

D1 D2 D3

1 1 1.

Page 204: Finite Math Lecture Slides

Example: A voter must select one city council candidate, oneschool board candidate, and one county commissioner candidate.There are 4 candidates for the city council, 9 candidates for theschool board, and 5 candidates for county commissioner.

Analysis:

1. Domain: 18 candidates

2. Qualities: city council, school board, county commission

3. Roles: city council, school board, county commission

4. Replacement: no

Soup, Salad, Entree, Veg Paradigm

Therefore,

D1 = 4 city council candidatesD2 = 9 school board candidatesD3 = 5 county commission candidates

D1 D2 D3

1 1 1.

Page 205: Finite Math Lecture Slides

Example: Mary, Paul and Ryan are late for breakfast in the dorm.When they arrive, the pastry tray has only 8 different donuts (e.g.,one glazed donut, one bear claw, one apple fritter, etc.), 4 differentDanishes (e.g., one cheese, one cheery, etc.), and 4 differentmuffins (e.g., one blueberry, one apple, etc.). Each of Mary, Paul,and Ryan takes one of these pastries.

Analysis:

1. Domain: 16 pastries

2. Qualities: none that matter

3. Roles: Mary, Paul, Ryan

4. Replacement: no

Casting a Play Paradigm

Therefore, D = 16 pastries, and S =

Mary Paul RyanD D D

1 1 1

.

Page 206: Finite Math Lecture Slides

Example: Mary, Paul and Ryan are late for breakfast in the dorm.When they arrive, the pastry tray has only 8 different donuts (e.g.,one glazed donut, one bear claw, one apple fritter, etc.), 4 differentDanishes (e.g., one cheese, one cheery, etc.), and 4 differentmuffins (e.g., one blueberry, one apple, etc.). Each of Mary, Paul,and Ryan takes one of these pastries.

Analysis:

1. Domain: 16 pastries

2. Qualities: none that matter

3. Roles: Mary, Paul, Ryan

4. Replacement: no

Casting a Play Paradigm

Therefore, D = 16 pastries, and S =

Mary Paul RyanD D D

1 1 1

.

Page 207: Finite Math Lecture Slides

Example: Mary, Paul and Ryan are late for breakfast in the dorm.When they arrive, the pastry tray has only 8 different donuts (e.g.,one glazed donut, one bear claw, one apple fritter, etc.), 4 differentDanishes (e.g., one cheese, one cheery, etc.), and 4 differentmuffins (e.g., one blueberry, one apple, etc.). Each of Mary, Paul,and Ryan takes one of these pastries.

Analysis:

1. Domain: 16 pastries

2. Qualities: none that matter

3. Roles: Mary, Paul, Ryan

4. Replacement: no

Casting a Play Paradigm

Therefore, D = 16 pastries, and S =

Mary Paul RyanD D D

1 1 1

.

Page 208: Finite Math Lecture Slides

Example: Mary, Paul and Ryan are late for breakfast in the dorm.When they arrive, the pastry tray has only 8 different donuts (e.g.,one glazed donut, one bear claw, one apple fritter, etc.), 4 differentDanishes (e.g., one cheese, one cheery, etc.), and 4 differentmuffins (e.g., one blueberry, one apple, etc.). Each of Mary, Paul,and Ryan takes one of these pastries.

Analysis:

1. Domain: 16 pastries

2. Qualities: none that matter

3. Roles: Mary, Paul, Ryan

4. Replacement: no

Casting a Play Paradigm

Therefore, D = 16 pastries, and S =

Mary Paul RyanD D D

1 1 1

.

Page 209: Finite Math Lecture Slides

Example: Claire, Emily, and Laura are at a store that sells works bylocal artists. The store has 6 oil paintings, 5 water color sketches,and 9 textile pieces for sale. Clair purchases an oil painting, Emilypurchases a water color sketch, and Laura purchases a textile piece.

Analysis:

1. Domain: 20 artworks

2. Qualities: oil, water color, textile

3. Roles: Clair, Emily, Laura

4. Replacement: no

Soup, Salad, Entree, Veg Paradigm

Therefore,

O = 6 oil paintings W = 5 water colors T = 9 textiles

and S =

Clair Emily LauraO W T

1 1 1

.

Page 210: Finite Math Lecture Slides

Example: Claire, Emily, and Laura are at a store that sells works bylocal artists. The store has 6 oil paintings, 5 water color sketches,and 9 textile pieces for sale. Clair purchases an oil painting, Emilypurchases a water color sketch, and Laura purchases a textile piece.

Analysis:

1. Domain: 20 artworks

2. Qualities: oil, water color, textile

3. Roles: Clair, Emily, Laura

4. Replacement: no

Soup, Salad, Entree, Veg Paradigm

Therefore,

O = 6 oil paintings W = 5 water colors T = 9 textiles

and S =

Clair Emily LauraO W T

1 1 1

.

Page 211: Finite Math Lecture Slides

Example: Claire, Emily, and Laura are at a store that sells works bylocal artists. The store has 6 oil paintings, 5 water color sketches,and 9 textile pieces for sale. Clair purchases an oil painting, Emilypurchases a water color sketch, and Laura purchases a textile piece.

Analysis:

1. Domain: 20 artworks

2. Qualities: oil, water color, textile

3. Roles: Clair, Emily, Laura

4. Replacement: no

Soup, Salad, Entree, Veg Paradigm

Therefore,

O = 6 oil paintings W = 5 water colors T = 9 textiles

and S =

Clair Emily LauraO W T

1 1 1

.

Page 212: Finite Math Lecture Slides

Example: Claire, Emily, and Laura are at a store that sells works bylocal artists. The store has 6 oil paintings, 5 water color sketches,and 9 textile pieces for sale. Clair purchases an oil painting, Emilypurchases a water color sketch, and Laura purchases a textile piece.

Analysis:

1. Domain: 20 artworks

2. Qualities: oil, water color, textile

3. Roles: Clair, Emily, Laura

4. Replacement: no

Soup, Salad, Entree, Veg Paradigm

Therefore,

O = 6 oil paintings W = 5 water colors T = 9 textiles

and S =

Clair Emily LauraO W T

1 1 1

.

Page 213: Finite Math Lecture Slides

Example: The Dean of Students has 4 Associate Deans, 5Assistant Deans, and 9 program specialists on his staff. To checkfor alcohol violations this weekend, the Dean is going to select oneof these staff members to visit the Sigma Lambda house at 11:00pm on Saturday, one of these staff members to visit the Delta Phihouse at 11:00 pm on Saturday, and one of these staff members tovisit the Phi Sigma house at 11:00 pm on Saturday.

Analysis:

1. Domain: 19 staff members

2. Qualities: none that matter

3. Roles: ΣΛ, ∆Φ, ΦΣ

4. Replacement: no

Casting a Play Paradigm

Therefore, D = 18 staff members, and S =

ΣΛ ∆Φ ΦΣD D D

1 1 1

.

Page 214: Finite Math Lecture Slides

Example: The Dean of Students has 4 Associate Deans, 5Assistant Deans, and 9 program specialists on his staff. To checkfor alcohol violations this weekend, the Dean is going to select oneof these staff members to visit the Sigma Lambda house at 11:00pm on Saturday, one of these staff members to visit the Delta Phihouse at 11:00 pm on Saturday, and one of these staff members tovisit the Phi Sigma house at 11:00 pm on Saturday.

Analysis:

1. Domain: 19 staff members

2. Qualities: none that matter

3. Roles: ΣΛ, ∆Φ, ΦΣ

4. Replacement: no

Casting a Play Paradigm

Therefore, D = 18 staff members, and S =

ΣΛ ∆Φ ΦΣD D D

1 1 1

.

Page 215: Finite Math Lecture Slides

Example: The Dean of Students has 4 Associate Deans, 5Assistant Deans, and 9 program specialists on his staff. To checkfor alcohol violations this weekend, the Dean is going to select oneof these staff members to visit the Sigma Lambda house at 11:00pm on Saturday, one of these staff members to visit the Delta Phihouse at 11:00 pm on Saturday, and one of these staff members tovisit the Phi Sigma house at 11:00 pm on Saturday.

Analysis:

1. Domain: 19 staff members

2. Qualities: none that matter

3. Roles: ΣΛ, ∆Φ, ΦΣ

4. Replacement: no

Casting a Play Paradigm

Therefore, D = 18 staff members, and S =

ΣΛ ∆Φ ΦΣD D D

1 1 1

.

Page 216: Finite Math Lecture Slides

Example: The Dean of Students has 4 Associate Deans, 5Assistant Deans, and 9 program specialists on his staff. To checkfor alcohol violations this weekend, the Dean is going to select oneof these staff members to visit the Sigma Lambda house at 11:00pm on Saturday, one of these staff members to visit the Delta Phihouse at 11:00 pm on Saturday, and one of these staff members tovisit the Phi Sigma house at 11:00 pm on Saturday.

Analysis:

1. Domain: 19 staff members

2. Qualities: none that matter

3. Roles: ΣΛ, ∆Φ, ΦΣ

4. Replacement: no

Casting a Play Paradigm

Therefore, D = 18 staff members, and S =

ΣΛ ∆Φ ΦΣD D D

1 1 1

.

Page 217: Finite Math Lecture Slides

Example: Laura, Susan, and Chris have just enrolled in a universityin Chicago. Each one must open a checking/savings account at alocal institution. They may choose from 8 (regular) banks, 4savings banks, and 4 credit unions.

Analysis:

1. Domain: 16 banks

2. Qualities: none that matter

3. Roles: Laura, Susan, Chris

4. Replacement: yes

Coffee, Tea, Coke, Sprite Paradigm

Therefore,B = 16 banks

and S =

Laura Susan ChrisB B B

1 1 1

.

Page 218: Finite Math Lecture Slides

Example: Laura, Susan, and Chris have just enrolled in a universityin Chicago. Each one must open a checking/savings account at alocal institution. They may choose from 8 (regular) banks, 4savings banks, and 4 credit unions.

Analysis:

1. Domain: 16 banks

2. Qualities: none that matter

3. Roles: Laura, Susan, Chris

4. Replacement: yes

Coffee, Tea, Coke, Sprite Paradigm

Therefore,B = 16 banks

and S =

Laura Susan ChrisB B B

1 1 1

.

Page 219: Finite Math Lecture Slides

Example: Laura, Susan, and Chris have just enrolled in a universityin Chicago. Each one must open a checking/savings account at alocal institution. They may choose from 8 (regular) banks, 4savings banks, and 4 credit unions.

Analysis:

1. Domain: 16 banks

2. Qualities: none that matter

3. Roles: Laura, Susan, Chris

4. Replacement: yes

Coffee, Tea, Coke, Sprite Paradigm

Therefore,B = 16 banks

and S =

Laura Susan ChrisB B B

1 1 1

.

Page 220: Finite Math Lecture Slides

Example: Laura, Susan, and Chris have just enrolled in a universityin Chicago. Each one must open a checking/savings account at alocal institution. They may choose from 8 (regular) banks, 4savings banks, and 4 credit unions.

Analysis:

1. Domain: 16 banks

2. Qualities: none that matter

3. Roles: Laura, Susan, Chris

4. Replacement: yes

Coffee, Tea, Coke, Sprite Paradigm

Therefore,B = 16 banks

and S =

Laura Susan ChrisB B B

1 1 1

.

Page 221: Finite Math Lecture Slides

Example: Alyssa, Brandy,and Carl are having dinner at a Chineserestaurant. The menu lists 9 beef dishes, 6 pork dishes, and 6chicken dishes. Alyssa, Brandy, and Carl have decided that theywill share 3 dishes. They jointly select one beef dish, one porkdish, and one chicken dish.

Analysis:

1. Domain: 21 dishes

2. Qualities: beef, pork, chicken

3. Roles: beef, pork, chicken

4. Replacement: yes(?), n/a

Soup, Salad, Entree, Veg Paradigm

Therefore,

B = 9 beef dishes P = 6 pork dishes C = 6 chicken dishes

and S =

B P C

1 1 1.

Page 222: Finite Math Lecture Slides

Example: Alyssa, Brandy,and Carl are having dinner at a Chineserestaurant. The menu lists 9 beef dishes, 6 pork dishes, and 6chicken dishes. Alyssa, Brandy, and Carl have decided that theywill share 3 dishes. They jointly select one beef dish, one porkdish, and one chicken dish.

Analysis:

1. Domain: 21 dishes

2. Qualities: beef, pork, chicken

3. Roles: beef, pork, chicken

4. Replacement: yes(?), n/a

Soup, Salad, Entree, Veg Paradigm

Therefore,

B = 9 beef dishes P = 6 pork dishes C = 6 chicken dishes

and S =

B P C

1 1 1.

Page 223: Finite Math Lecture Slides

Example: Alyssa, Brandy,and Carl are having dinner at a Chineserestaurant. The menu lists 9 beef dishes, 6 pork dishes, and 6chicken dishes. Alyssa, Brandy, and Carl have decided that theywill share 3 dishes. They jointly select one beef dish, one porkdish, and one chicken dish.

Analysis:

1. Domain: 21 dishes

2. Qualities: beef, pork, chicken

3. Roles: beef, pork, chicken

4. Replacement: yes(?), n/a

Soup, Salad, Entree, Veg Paradigm

Therefore,

B = 9 beef dishes P = 6 pork dishes C = 6 chicken dishes

and S =

B P C

1 1 1.

Page 224: Finite Math Lecture Slides

Example: Alyssa, Brandy,and Carl are having dinner at a Chineserestaurant. The menu lists 9 beef dishes, 6 pork dishes, and 6chicken dishes. Alyssa, Brandy, and Carl have decided that theywill share 3 dishes. They jointly select one beef dish, one porkdish, and one chicken dish.

Analysis:

1. Domain: 21 dishes

2. Qualities: beef, pork, chicken

3. Roles: beef, pork, chicken

4. Replacement: yes(?), n/a

Soup, Salad, Entree, Veg Paradigm

Therefore,

B = 9 beef dishes P = 6 pork dishes C = 6 chicken dishes

and S =

B P C

1 1 1.

Page 225: Finite Math Lecture Slides

Example: Allyson, Beth, and Chelsea, who are sharing anapartment, have gone to a bookstore to purchase 3 cookbooks.The store had 5 different general cookbooks, 3 different Italiancookbooks, and 6 different French cookbooks. Together theyselect 3 of these cookbooks.

Analysis:

1. Domain: 14 cookbooks

2. Qualities: none that matter

3. Roles: none

4. Replacement: no

City Council Committee Paradigm

Therefore,C = 14 cookbooks

and S =

C

3.

Page 226: Finite Math Lecture Slides

Example: Allyson, Beth, and Chelsea, who are sharing anapartment, have gone to a bookstore to purchase 3 cookbooks.The store had 5 different general cookbooks, 3 different Italiancookbooks, and 6 different French cookbooks. Together theyselect 3 of these cookbooks.

Analysis:

1. Domain: 14 cookbooks

2. Qualities: none that matter

3. Roles: none

4. Replacement: no

City Council Committee Paradigm

Therefore,C = 14 cookbooks

and S =

C

3.

Page 227: Finite Math Lecture Slides

Example: Allyson, Beth, and Chelsea, who are sharing anapartment, have gone to a bookstore to purchase 3 cookbooks.The store had 5 different general cookbooks, 3 different Italiancookbooks, and 6 different French cookbooks. Together theyselect 3 of these cookbooks.

Analysis:

1. Domain: 14 cookbooks

2. Qualities: none that matter

3. Roles: none

4. Replacement: no

City Council Committee Paradigm

Therefore,C = 14 cookbooks

and S =

C

3.

Page 228: Finite Math Lecture Slides

Example: Allyson, Beth, and Chelsea, who are sharing anapartment, have gone to a bookstore to purchase 3 cookbooks.The store had 5 different general cookbooks, 3 different Italiancookbooks, and 6 different French cookbooks. Together theyselect 3 of these cookbooks.

Analysis:

1. Domain: 14 cookbooks

2. Qualities: none that matter

3. Roles: none

4. Replacement: no

City Council Committee Paradigm

Therefore,C = 14 cookbooks

and S =

C

3.

Page 229: Finite Math Lecture Slides

Calculating Sizes

Now that you know how to analyze problems,the next thing is to learn:

How do you use your analysis to count the number of outcomes?

Page 230: Finite Math Lecture Slides

Soup, Salad, Entree, Veg Paradigm

S = 3 soups L = 5 saladsE = 4 entrees V = 6 vegetables

and S =

P L E V

1 1 1 1# choices: 3 5 4 6

.

Answer: n[S ] = size of S is

n[S ] = 3 · 5 · 4 · 6 = 360.

Page 231: Finite Math Lecture Slides

Soup, Salad, Entree, Veg Paradigm

S = 3 soups L = 5 saladsE = 4 entrees V = 6 vegetables

and S =

P L E V

1 1 1 1# choices: 3 5 4 6

.

Answer: n[S ] = size of S is

n[S ] = 3 · 5 · 4 · 6 = 360.

Page 232: Finite Math Lecture Slides

Coffee, Tea, Coke, Sprite Paradigm

D = 4 beverages

and S =

Laura Susan ChrisB B B

1 1 1# choices: 4 4 4

.

Answer:n[S ] = 4 · 4 · 4 = 43 = 64.

In general, if d = size of domain and r = # roles,

n[S ] = d r .

Page 233: Finite Math Lecture Slides

Coffee, Tea, Coke, Sprite Paradigm

D = 4 beverages

and S =

Laura Susan ChrisB B B

1 1 1# choices: 4 4 4

.

Answer:n[S ] = 4 · 4 · 4 = 43 = 64.

In general, if d = size of domain and r = # roles,

n[S ] = d r .

Page 234: Finite Math Lecture Slides

Coffee, Tea, Coke, Sprite Paradigm

D = 4 beverages

and S =

Laura Susan ChrisB B B

1 1 1# choices: 4 4 4

.

Answer:n[S ] = 4 · 4 · 4 = 43 = 64.

In general, if d = size of domain and r = # roles,

n[S ] = d r .

Page 235: Finite Math Lecture Slides

Casting a Play Paradigm

D = 6 Actresses

and S =

Grandmother Mother DaughterD D D

1 1 1# choices: 6 5 4

.

Answer:n[S ] = 6 · 5 · 4 = P(6, 3) = 120.

In general, if d = size of domain and r = # roles,

n[S ] = P(d , r).

Page 236: Finite Math Lecture Slides

Casting a Play Paradigm

D = 6 Actresses

and S =

Grandmother Mother DaughterD D D

1 1 1# choices: 6 5 4

.

Answer:n[S ] = 6 · 5 · 4 = P(6, 3) = 120.

In general, if d = size of domain and r = # roles,

n[S ] = P(d , r).

Page 237: Finite Math Lecture Slides

Casting a Play Paradigm

D = 6 Actresses

and S =

Grandmother Mother DaughterD D D

1 1 1# choices: 6 5 4

.

Answer:n[S ] = 6 · 5 · 4 = P(6, 3) = 120.

In general, if d = size of domain and r = # roles,

n[S ] = P(d , r).

Page 238: Finite Math Lecture Slides

City Council Committee Paradigm

D = 8 council members

and S =

D

3# choices: C(8, 3) = 56

.

Answer:n[S ] = C(8, 3) = 56.

In general, if d = size of domain and n = # chosen,

n[S ] = C(d , n).

Page 239: Finite Math Lecture Slides

City Council Committee Paradigm

D = 8 council members

and S =

D

3# choices: C(8, 3) = 56

.

Answer:n[S ] = C(8, 3) = 56.

In general, if d = size of domain and n = # chosen,

n[S ] = C(d , n).

Page 240: Finite Math Lecture Slides

City Council Committee Paradigm

D = 8 council members

and S =

D

3# choices: C(8, 3) = 56

.

Answer:n[S ] = C(8, 3) = 56.

In general, if d = size of domain and n = # chosen,

n[S ] = C(d , n).

Page 241: Finite Math Lecture Slides

CombinationsThe C function calculates the number of combinations.

A combination is an unordered list without repetitions.

(A permutation is an ordered list.)

An outcome in the City Council Committee Paradigm is a combo.

Mathematically,C(d , r) = P(d ,r)

P(r ,r) .

For example,

C(8, 3) = 8·7·63·2·1 = 8·7·66

63·62·1 = 56

C(8, 5) = 8·7·6·5·45·4·3·2·1 = 8·7·66·65·64

65·64·63·62·1 = 56.

In general,

C(d , r) = C(d , d − r).

Page 242: Finite Math Lecture Slides

CombinationsThe C function calculates the number of combinations.

A combination is an unordered list without repetitions.

(A permutation is an ordered list.)

An outcome in the City Council Committee Paradigm is a combo.

Mathematically,C(d , r) = P(d ,r)

P(r ,r) .

For example,

C(8, 3) = 8·7·63·2·1 = 8·7·66

63·62·1 = 56

C(8, 5) = 8·7·6·5·45·4·3·2·1 = 8·7·66·65·64

65·64·63·62·1 = 56.

In general,

C(d , r) = C(d , d − r).

Page 243: Finite Math Lecture Slides

CombinationsThe C function calculates the number of combinations.

A combination is an unordered list without repetitions.

(A permutation is an ordered list.)

An outcome in the City Council Committee Paradigm is a combo.

Mathematically,C(d , r) = P(d ,r)

P(r ,r) .

For example,

C(8, 3) = 8·7·63·2·1 = 8·7·66

63·62·1 = 56

C(8, 5) = 8·7·6·5·45·4·3·2·1 = 8·7·66·65·64

65·64·63·62·1 = 56.

In general,

C(d , r) = C(d , d − r).

Page 244: Finite Math Lecture Slides

Example: At dinner one night at summer camp, 8 campers and acounselor are sitting at a long table. The counselor must select 3of the campers to carry the dishes to the cleanup window. Howmany overall outcomes are possible?

City Council Committee Paradigm

D = 8 campers

and S = D

3C(8, 3) = 56

.

Answer:n[S ] = C(8, 3) = 56

Page 245: Finite Math Lecture Slides

Example: At dinner one night at summer camp, 8 campers and acounselor are sitting at a long table. The counselor must select 3of the campers to carry the dishes to the cleanup window. Howmany overall outcomes are possible?

City Council Committee Paradigm

D = 8 campers

and S = D

3C(8, 3) = 56

.

Answer:n[S ] = C(8, 3) = 56

Page 246: Finite Math Lecture Slides

Example: At dinner one night at summer camp, 8 campers and acounselor are sitting at a long table. The counselor must select 3of the campers to carry the dishes to the cleanup window. Howmany overall outcomes are possible?

City Council Committee Paradigm

D = 8 campers

and S = D

3C(8, 3) = 56

.

Answer:n[S ] = C(8, 3) = 56

Page 247: Finite Math Lecture Slides

Example: A voter must select one city council candidate, oneschool board candidate, and one county commissioner candidate.There are 4 candidates for the city council, 9 candidates for theschool board, and 5 candidates for county commissioner.

D1 = 4 city council candidatesD2 = 9 school board candidatesD3 = 5 county commission candidates

and S =

City Council School Board County CommissionD1 D2 D3

1 1 14 9 5

Soup, Salad, Entree, Veg Paradigm

Answer:n[S ] = 4 · 9 · 5 = 180

Page 248: Finite Math Lecture Slides

Example: A voter must select one city council candidate, oneschool board candidate, and one county commissioner candidate.There are 4 candidates for the city council, 9 candidates for theschool board, and 5 candidates for county commissioner.

D1 = 4 city council candidatesD2 = 9 school board candidatesD3 = 5 county commission candidates

and S =

City Council School Board County CommissionD1 D2 D3

1 1 14 9 5

Soup, Salad, Entree, Veg Paradigm

Answer:n[S ] = 4 · 9 · 5 = 180

Page 249: Finite Math Lecture Slides

Example: A voter must select one city council candidate, oneschool board candidate, and one county commissioner candidate.There are 4 candidates for the city council, 9 candidates for theschool board, and 5 candidates for county commissioner.

D1 = 4 city council candidatesD2 = 9 school board candidatesD3 = 5 county commission candidates

and S =

City Council School Board County CommissionD1 D2 D3

1 1 14 9 5

Soup, Salad, Entree, Veg Paradigm

Answer:n[S ] = 4 · 9 · 5 = 180

Page 250: Finite Math Lecture Slides

Example: Mary, Paul and Ryan are late for breakfast in the dorm.When they arrive, the pastry tray has only 8 different donuts (e.g.,one glazed donut, one bear claw, one apple fritter, etc.), 4 differentDanishes (e.g., one cheese, one cheery, etc.), and 4 differentmuffins (e.g., one blueberry, one apple, etc.). Each of Mary, Paul,and Ryan takes one of these pastries.

D = 16 pastries

and S =

Mary Paul RyanD D D

1 1 116 15 14.

Casting a Play Paradigm

Answer:n[S ] = P(16, 3) = 16 · 15 · 14 = 3360

Page 251: Finite Math Lecture Slides

Example: Mary, Paul and Ryan are late for breakfast in the dorm.When they arrive, the pastry tray has only 8 different donuts (e.g.,one glazed donut, one bear claw, one apple fritter, etc.), 4 differentDanishes (e.g., one cheese, one cheery, etc.), and 4 differentmuffins (e.g., one blueberry, one apple, etc.). Each of Mary, Paul,and Ryan takes one of these pastries.

D = 16 pastries

and S =

Mary Paul RyanD D D

1 1 116 15 14.

Casting a Play Paradigm

Answer:n[S ] = P(16, 3) = 16 · 15 · 14 = 3360

Page 252: Finite Math Lecture Slides

Example: Mary, Paul and Ryan are late for breakfast in the dorm.When they arrive, the pastry tray has only 8 different donuts (e.g.,one glazed donut, one bear claw, one apple fritter, etc.), 4 differentDanishes (e.g., one cheese, one cheery, etc.), and 4 differentmuffins (e.g., one blueberry, one apple, etc.). Each of Mary, Paul,and Ryan takes one of these pastries.

D = 16 pastries

and S =

Mary Paul RyanD D D

1 1 116 15 14.

Casting a Play Paradigm

Answer:n[S ] = P(16, 3) = 16 · 15 · 14 = 3360

Page 253: Finite Math Lecture Slides

Example: Claire, Emily, and Laura are at a store that sells worksby local artists. The store has 6 oil paintings, 5 water colorsketches, and 9 textile pieces for sale. All of the items are differentfrom one another. Clair purchases an oil painting, Emily purchasesa water color sketch, and Laura purchases a textile piece.

O = 6 oil paintingsW = 5 water colorsT = 9 textiles

and S =

Claire Emily LauraO W T

1 1 16 5 9

Soup, Salad, Entree, Veg Paradigm

Answer:n[S ] = 6 · 5 · 9 = 270

Page 254: Finite Math Lecture Slides

Example: Claire, Emily, and Laura are at a store that sells worksby local artists. The store has 6 oil paintings, 5 water colorsketches, and 9 textile pieces for sale. All of the items are differentfrom one another. Clair purchases an oil painting, Emily purchasesa water color sketch, and Laura purchases a textile piece.

O = 6 oil paintingsW = 5 water colorsT = 9 textiles

and S =

Claire Emily LauraO W T

1 1 16 5 9

Soup, Salad, Entree, Veg Paradigm

Answer:n[S ] = 6 · 5 · 9 = 270

Page 255: Finite Math Lecture Slides

Example: Claire, Emily, and Laura are at a store that sells worksby local artists. The store has 6 oil paintings, 5 water colorsketches, and 9 textile pieces for sale. All of the items are differentfrom one another. Clair purchases an oil painting, Emily purchasesa water color sketch, and Laura purchases a textile piece.

O = 6 oil paintingsW = 5 water colorsT = 9 textiles

and S =

Claire Emily LauraO W T

1 1 16 5 9

Soup, Salad, Entree, Veg Paradigm

Answer:n[S ] = 6 · 5 · 9 = 270

Page 256: Finite Math Lecture Slides

Example: The Dean of Students has 4 Associate Deans, 5Assistant Deans, and 9 program specialists on his staff. To checkfor alcohol violations this weekend, the Dean is going to select oneof these staff members to visit the Sigma Lambda house at 11:00pm on Saturday, one of these staff members to visit the Delta Phihouse at 11:00 pm on Saturday, and one of these staff members tovisit the Phi Sigma house at 11:00 pm on Saturday.

D = 18 staff members

and S =

ΣΛ ∆Φ ΦΣD D D

1 1 118 17 16

Casting a Play Paradigm

Answer:n[S ] = P(18, 3) = 18 · 17 · 16 = 4896

Page 257: Finite Math Lecture Slides

Example: The Dean of Students has 4 Associate Deans, 5Assistant Deans, and 9 program specialists on his staff. To checkfor alcohol violations this weekend, the Dean is going to select oneof these staff members to visit the Sigma Lambda house at 11:00pm on Saturday, one of these staff members to visit the Delta Phihouse at 11:00 pm on Saturday, and one of these staff members tovisit the Phi Sigma house at 11:00 pm on Saturday.

D = 18 staff members

and S =

ΣΛ ∆Φ ΦΣD D D

1 1 118 17 16

Casting a Play Paradigm

Answer:n[S ] = P(18, 3) = 18 · 17 · 16 = 4896

Page 258: Finite Math Lecture Slides

Example: The Dean of Students has 4 Associate Deans, 5Assistant Deans, and 9 program specialists on his staff. To checkfor alcohol violations this weekend, the Dean is going to select oneof these staff members to visit the Sigma Lambda house at 11:00pm on Saturday, one of these staff members to visit the Delta Phihouse at 11:00 pm on Saturday, and one of these staff members tovisit the Phi Sigma house at 11:00 pm on Saturday.

D = 18 staff members

and S =

ΣΛ ∆Φ ΦΣD D D

1 1 118 17 16

Casting a Play Paradigm

Answer:n[S ] = P(18, 3) = 18 · 17 · 16 = 4896

Page 259: Finite Math Lecture Slides

Example: Laura, Susan, and Chris have just enrolled in a universityin Chicago. Each one must open a checking/savings account at alocal institution. They may choose from 8 (regular) banks, 4savings banks, and 4 credit unions.

D = 16 banks

and S =

Laura Susan ChrisB B B

1 1 116 16 16

Coffee, Tea, Coke, Sprite Paradigm

Answer:n[S ] = 163 = 4096

Page 260: Finite Math Lecture Slides

Example: Laura, Susan, and Chris have just enrolled in a universityin Chicago. Each one must open a checking/savings account at alocal institution. They may choose from 8 (regular) banks, 4savings banks, and 4 credit unions.

D = 16 banks

and S =

Laura Susan ChrisB B B

1 1 116 16 16

Coffee, Tea, Coke, Sprite Paradigm

Answer:n[S ] = 163 = 4096

Page 261: Finite Math Lecture Slides

Example: Laura, Susan, and Chris have just enrolled in a universityin Chicago. Each one must open a checking/savings account at alocal institution. They may choose from 8 (regular) banks, 4savings banks, and 4 credit unions.

D = 16 banks

and S =

Laura Susan ChrisB B B

1 1 116 16 16

Coffee, Tea, Coke, Sprite Paradigm

Answer:n[S ] = 163 = 4096

Page 262: Finite Math Lecture Slides

Example: Alyssa, Brandy,and Carl are having dinner at a Chineserestaurant. The menu lists 9 beef dishes, 6 pork dishes, and 6chicken dishes. Alyssa, Brandy, and Carl have decided that theywill share 3 dishes. They jointly select one beef dish, one porkdish, and one chicken dish.

B = 9 beef dishesP = 6 pork dishesC = 6 chicken dishes

and S =

B P C

1 1 19 6 6

Soup, Salad, Entree, Veg Paradigm

Answer:n[S ] = 9 · 6 · 6 = 324

Page 263: Finite Math Lecture Slides

Example: Alyssa, Brandy,and Carl are having dinner at a Chineserestaurant. The menu lists 9 beef dishes, 6 pork dishes, and 6chicken dishes. Alyssa, Brandy, and Carl have decided that theywill share 3 dishes. They jointly select one beef dish, one porkdish, and one chicken dish.

B = 9 beef dishesP = 6 pork dishesC = 6 chicken dishes

and S =

B P C

1 1 19 6 6

Soup, Salad, Entree, Veg Paradigm

Answer:n[S ] = 9 · 6 · 6 = 324

Page 264: Finite Math Lecture Slides

Example: Alyssa, Brandy,and Carl are having dinner at a Chineserestaurant. The menu lists 9 beef dishes, 6 pork dishes, and 6chicken dishes. Alyssa, Brandy, and Carl have decided that theywill share 3 dishes. They jointly select one beef dish, one porkdish, and one chicken dish.

B = 9 beef dishesP = 6 pork dishesC = 6 chicken dishes

and S =

B P C

1 1 19 6 6

Soup, Salad, Entree, Veg Paradigm

Answer:n[S ] = 9 · 6 · 6 = 324

Page 265: Finite Math Lecture Slides

Example: Allyson, Beth, and Chelsea, who are sharing anapartment, have gone to a bookstore to purchase 3 cookbooks.The store had 5 different general cookbooks, 3 different Italiancookbooks, and 6 different French cookbooks. Together theyselect 3 of these cookbooks.

D = 14 cookbooks

and S =

D

3C(14, 3).

City Council Committee Paradigm

Answer:n[S ] = C(14, 3) = 14·13·12

3·2·1 = 364

Page 266: Finite Math Lecture Slides

Example: Allyson, Beth, and Chelsea, who are sharing anapartment, have gone to a bookstore to purchase 3 cookbooks.The store had 5 different general cookbooks, 3 different Italiancookbooks, and 6 different French cookbooks. Together theyselect 3 of these cookbooks.

D = 14 cookbooks

and S =

D

3C(14, 3).

City Council Committee Paradigm

Answer:n[S ] = C(14, 3) = 14·13·12

3·2·1 = 364

Page 267: Finite Math Lecture Slides

Example: Allyson, Beth, and Chelsea, who are sharing anapartment, have gone to a bookstore to purchase 3 cookbooks.The store had 5 different general cookbooks, 3 different Italiancookbooks, and 6 different French cookbooks. Together theyselect 3 of these cookbooks.

D = 14 cookbooks

and S =

D

3C(14, 3).

City Council Committee Paradigm

Answer:n[S ] = C(14, 3) = 14·13·12

3·2·1 = 364

Page 268: Finite Math Lecture Slides

Example: A city council has 5 Democratic members and 3Republican members. The mayor is to assign 1 Democratic councilmember to the Police Com. and 1 Republican council member tothe Zoning Com. How many outcomes are possible?

Analysis:

1. Domain: 8 council members2. Qualities: Dem, Rep3. Roles: Police, Zoning4. Replacement: no

Soup, Salad, Entree, Veg Paradigm

D = 5 Dems R = 3 Reps

and S =

Police ZoningD R

1 15 3.

Answer: n[S ] = 5 · 3 = 15

Page 269: Finite Math Lecture Slides

Example: A city council has 5 Democratic members and 3Republican members. The mayor is to assign 1 Democratic councilmember to the Police Com. and 1 Republican council member tothe Zoning Com. How many outcomes are possible?

Analysis:

1. Domain: 8 council members2. Qualities: Dem, Rep3. Roles: Police, Zoning4. Replacement: no

Soup, Salad, Entree, Veg Paradigm

D = 5 Dems R = 3 Reps

and S =

Police ZoningD R

1 15 3.

Answer: n[S ] = 5 · 3 = 15

Page 270: Finite Math Lecture Slides

Example: A city council has 5 Democratic members and 3Republican members. The mayor is to assign 1 Democratic councilmember to the Police Com. and 1 Republican council member tothe Zoning Com. How many outcomes are possible?

Analysis:

1. Domain: 8 council members2. Qualities: Dem, Rep3. Roles: Police, Zoning4. Replacement: no

Soup, Salad, Entree, Veg Paradigm

D = 5 Dems R = 3 Reps

and S =

Police ZoningD R

1 15 3.

Answer: n[S ] = 5 · 3 = 15

Page 271: Finite Math Lecture Slides

Example: A city council has 5 Democratic and 3 Republicanmembers. The mayor is to assign 1 council member to the PoliceCommission and 1 council member to the Zoning Commission.(The same person could be assigned to both commissions.) Howmany outcomes are possible?

Analysis:

1. Domain: 8 council members

2. Qualities: none that matter

3. Roles: Police, Zoning

4. Replacement: yes

Coffee, Tea, Coke, Sprite ParadigmLet C = “8 council members”, then S is

Police ZoningC C

1 18 8 = 64.

Page 272: Finite Math Lecture Slides

Example: A city council has 5 Democratic and 3 Republicanmembers. The mayor is to assign 1 council member to the PoliceCommission and 1 council member to the Zoning Commission.(The same person could be assigned to both commissions.) Howmany outcomes are possible?

Analysis:

1. Domain: 8 council members

2. Qualities: none that matter

3. Roles: Police, Zoning

4. Replacement: yes

Coffee, Tea, Coke, Sprite Paradigm

Let C = “8 council members”, then S is

Police ZoningC C

1 18 8 = 64.

Page 273: Finite Math Lecture Slides

Example: A city council has 5 Democratic and 3 Republicanmembers. The mayor is to assign 1 council member to the PoliceCommission and 1 council member to the Zoning Commission.(The same person could be assigned to both commissions.) Howmany outcomes are possible?

Analysis:

1. Domain: 8 council members

2. Qualities: none that matter

3. Roles: Police, Zoning

4. Replacement: yes

Coffee, Tea, Coke, Sprite ParadigmLet C = “8 council members”, then S is

Police ZoningC C

1 18 8 = 64.

Page 274: Finite Math Lecture Slides

Example: A city council has 5 Democratic members and 3Republican members. The mayor is to assign 1 council member tothe Police Commission and 1 council member to the ZoningCommission. No one can be assigned to two commissions. Howmany outcomes are possible?

Analysis:

1. Domain: 8 council members

2. Qualities: none that matter

3. Roles: Police, Zoning

4. Replacement: no

Casting a Play ParadigmLet C = “8 council members”, then S is

Police ZoningC C

1 18 7 = 56.

Page 275: Finite Math Lecture Slides

Example: A city council has 5 Democratic members and 3Republican members. The mayor is to assign 1 council member tothe Police Commission and 1 council member to the ZoningCommission. No one can be assigned to two commissions. Howmany outcomes are possible?

Analysis:

1. Domain: 8 council members

2. Qualities: none that matter

3. Roles: Police, Zoning

4. Replacement: no

Casting a Play Paradigm

Let C = “8 council members”, then S is

Police ZoningC C

1 18 7 = 56.

Page 276: Finite Math Lecture Slides

Example: A city council has 5 Democratic members and 3Republican members. The mayor is to assign 1 council member tothe Police Commission and 1 council member to the ZoningCommission. No one can be assigned to two commissions. Howmany outcomes are possible?

Analysis:

1. Domain: 8 council members

2. Qualities: none that matter

3. Roles: Police, Zoning

4. Replacement: no

Casting a Play ParadigmLet C = “8 council members”, then S is

Police ZoningC C

1 18 7 = 56.

Page 277: Finite Math Lecture Slides

Example: A city council has 5 Democratic members and 3Republican members. The mayor is to assign 2 council members tothe Police Commission. How many outcomes are possible?

Analysis:

1. Domain: 8 council members

2. Qualities: none that matter

3. Roles: none that matter

4. Replacement: no

City Council Committee ParadigmLet C = “8 council members”, then S is

PoliceC

2C(8, 2) = 28.

Page 278: Finite Math Lecture Slides

Example: A city council has 5 Democratic members and 3Republican members. The mayor is to assign 2 council members tothe Police Commission. How many outcomes are possible?

Analysis:

1. Domain: 8 council members

2. Qualities: none that matter

3. Roles: none that matter

4. Replacement: no

City Council Committee Paradigm

Let C = “8 council members”, then S is

PoliceC

2C(8, 2) = 28.

Page 279: Finite Math Lecture Slides

Example: A city council has 5 Democratic members and 3Republican members. The mayor is to assign 2 council members tothe Police Commission. How many outcomes are possible?

Analysis:

1. Domain: 8 council members

2. Qualities: none that matter

3. Roles: none that matter

4. Replacement: no

City Council Committee ParadigmLet C = “8 council members”, then S is

PoliceC

2C(8, 2) = 28.

Page 280: Finite Math Lecture Slides

Example: A city council has 5 Democratic members and 3Republican members. In how many different ways can the mayorappoint a committee consisting of 3 Democratic council membersand 2 Republican council members?

Analysis:

1. Domain: 8 council members2. Qualities: Dem, Rep3. Roles: Dem, Rep4. Replacement: no

Doesn’t exactly fit any paradigm

D = 5 Dems R = 3 Reps

and S is

CommitteeD R

3 2C(5, 3) C(3, 2) = 30.

Page 281: Finite Math Lecture Slides

Example: A city council has 5 Democratic members and 3Republican members. In how many different ways can the mayorappoint a committee consisting of 3 Democratic council membersand 2 Republican council members?

Analysis:

1. Domain: 8 council members2. Qualities: Dem, Rep3. Roles: Dem, Rep4. Replacement: no

Doesn’t exactly fit any paradigm

D = 5 Dems R = 3 Reps

and S is

CommitteeD R

3 2C(5, 3) C(3, 2) = 30.

Page 282: Finite Math Lecture Slides

Example: A city council has 5 Democratic members and 3Republican members. In how many different ways can the mayorappoint a committee consisting of 3 Democratic council membersand 2 Republican council members?

Analysis:

1. Domain: 8 council members2. Qualities: Dem, Rep3. Roles: Dem, Rep4. Replacement: no

Doesn’t exactly fit any paradigm

D = 5 Dems R = 3 Reps

and S is

CommitteeD R

3 2C(5, 3) C(3, 2) = 30.

Page 283: Finite Math Lecture Slides

Example: A city council has 5 Democratic members and 3Republican members. In how many different ways can the mayorappoint a committee consisting of 3 Democratic council membersand 2 Republican council members?

Analysis:

1. Domain: 8 council members2. Qualities: Dem, Rep3. Roles: Dem, Rep4. Replacement: no

Doesn’t exactly fit any paradigm

D = 5 Dems R = 3 Reps

and S is

CommitteeD R

3 2C(5, 3) C(3, 2) = 30.

Page 284: Finite Math Lecture Slides

Example: A small fraternity consists of 7 seniors, 11 juniors, and 9sophomores. In how many different ways could a steeringcommittee consisting of 3 seniors, 2 juniors, and 1 sophomore beselected.

Analysis:

1. Domain: 27 frat bros

2. Qualities: Senior, Junior, Sophomore

3. Roles: Senior, Junior, Sophomore

4. Replacement: no

N = 7 Seniors J = 11 Juniors M = 9 Sophomores

and S is

CommitteeN J M

3 2 1C(7, 3) C(11, 2) C(9, 1) = 17325.

Page 285: Finite Math Lecture Slides

Example: A small fraternity consists of 7 seniors, 11 juniors, and 9sophomores. In how many different ways could a steeringcommittee consisting of 3 seniors, 2 juniors, and 1 sophomore beselected.

Analysis:

1. Domain: 27 frat bros

2. Qualities: Senior, Junior, Sophomore

3. Roles: Senior, Junior, Sophomore

4. Replacement: no

N = 7 Seniors J = 11 Juniors M = 9 Sophomores

and S is

CommitteeN J M

3 2 1C(7, 3) C(11, 2) C(9, 1) = 17325.

Page 286: Finite Math Lecture Slides

Example: A small fraternity consists of 7 seniors, 11 juniors, and 9sophomores. In how many different ways could a steeringcommittee consisting of 3 seniors, 2 juniors, and 1 sophomore beselected.

Analysis:

1. Domain: 27 frat bros

2. Qualities: Senior, Junior, Sophomore

3. Roles: Senior, Junior, Sophomore

4. Replacement: no

N = 7 Seniors J = 11 Juniors M = 9 Sophomores

and S is

CommitteeN J M

3 2 1C(7, 3) C(11, 2) C(9, 1) = 17325.

Page 287: Finite Math Lecture Slides

Example: The Board of Directors of a company has 12 members. If3 are to be assigned to the budget committee, 2 are to be assignedto the personnel committee, and 4 are to be assigned to thegovernment relations committee, and no one can serve on morethan one committee, how many different outcomes are possible?

Analysis:

1. Domain: 12 Directors

2. Qualities: none that matter

3. Roles: Budget, Personnel, GovRel

4. Replacement: no

Let D = “12 Directors”, then S is

Budget Personnel GovRelD D D

3 2 4C(12, 3) C(9, 2) C(7, 4) = 277200.

Page 288: Finite Math Lecture Slides

Example: The Board of Directors of a company has 12 members. If3 are to be assigned to the budget committee, 2 are to be assignedto the personnel committee, and 4 are to be assigned to thegovernment relations committee, and no one can serve on morethan one committee, how many different outcomes are possible?

Analysis:

1. Domain: 12 Directors

2. Qualities: none that matter

3. Roles: Budget, Personnel, GovRel

4. Replacement: no

Let D = “12 Directors”, then S is

Budget Personnel GovRelD D D

3 2 4C(12, 3) C(9, 2) C(7, 4) = 277200.

Page 289: Finite Math Lecture Slides

Example: The Board of Directors of a company has 12 members. If3 are to be assigned to the budget committee, 2 are to be assignedto the personnel committee, and 4 are to be assigned to thegovernment relations committee, and no one can serve on morethan one committee, how many different outcomes are possible?

Analysis:

1. Domain: 12 Directors

2. Qualities: none that matter

3. Roles: Budget, Personnel, GovRel

4. Replacement: no

Let D = “12 Directors”, then S is

Budget Personnel GovRelD D D

3 2 4C(12, 3) C(9, 2) C(7, 4) = 277200.

Page 290: Finite Math Lecture Slides

Example: The Board of Directors of a company has 12 members. If3 are to be assigned to the budget committee, 2 are to be assignedto the personnel committee, and 4 are to be assigned to thegovernment relations committee, and one can serve on more thanone committee, how many different outcomes are possible?

Analysis:

1. Domain: 12 Directors

2. Qualities: none that matter

3. Roles: Budget, Personnel, GovRel

4. Replacement: within committee: no; between committees: yes

Let D = “12 Directors”, then S is

Budget Personnel GovRelD D D

3 2 4C(12, 3) C(12, 2) C(12, 4) = 7187400.

Page 291: Finite Math Lecture Slides

Example: The Board of Directors of a company has 12 members. If3 are to be assigned to the budget committee, 2 are to be assignedto the personnel committee, and 4 are to be assigned to thegovernment relations committee, and one can serve on more thanone committee, how many different outcomes are possible?

Analysis:

1. Domain: 12 Directors

2. Qualities: none that matter

3. Roles: Budget, Personnel, GovRel

4. Replacement: within committee: no; between committees: yes

Let D = “12 Directors”, then S is

Budget Personnel GovRelD D D

3 2 4C(12, 3) C(12, 2) C(12, 4) = 7187400.

Page 292: Finite Math Lecture Slides

Example: The Board of Directors of a company has 12 members. If3 are to be assigned to the budget committee, 2 are to be assignedto the personnel committee, and 4 are to be assigned to thegovernment relations committee, and one can serve on more thanone committee, how many different outcomes are possible?

Analysis:

1. Domain: 12 Directors

2. Qualities: none that matter

3. Roles: Budget, Personnel, GovRel

4. Replacement: within committee: no; between committees: yes

Let D = “12 Directors”, then S is

Budget Personnel GovRelD D D

3 2 4C(12, 3) C(12, 2) C(12, 4) = 7187400.

Page 293: Finite Math Lecture Slides

Example: (Anagrams) How many “words” can be formed using theletters in the word “card”?

Analysis:

1. Domain: 4 letters (c, a, r, d)

2. Qualities: none that matter

3. Roles: Position (1–4)

4. Replacement: no

D = 4 Letters

and S is

1st 2nd 3rd 4thD D D D

1 1 1 14 3 2 1 = 24

Page 294: Finite Math Lecture Slides

Example: (Anagrams) How many “words” can be formed using theletters in the word “card”?

Analysis:

1. Domain: 4 letters (c, a, r, d)

2. Qualities: none that matter

3. Roles: Position (1–4)

4. Replacement: no

D = 4 Letters

and S is

1st 2nd 3rd 4thD D D D

1 1 1 14 3 2 1 = 24

Page 295: Finite Math Lecture Slides

Example: (Anagrams) How many “words” can be formed using theletters in the word “card”?

Analysis:

1. Domain: 4 letters (c, a, r, d)

2. Qualities: none that matter

3. Roles: Position (1–4)

4. Replacement: no

D = 4 Letters

and S is

1st 2nd 3rd 4thD D D D

1 1 1 14 3 2 1 = 24

Page 296: Finite Math Lecture Slides

Alternate SolutionExample: (Anagrams) How many “words” can be formed using theletters in the word “card”?

Analysis:

1. Domain: 4 Positions

2. Qualities: none that matter

3. Roles: c, a, r, d

4. Replacement: no

D = 4 Positions

and S is

C A R DD D D D

1 1 1 14 3 2 1 = 24

Page 297: Finite Math Lecture Slides

Alternate SolutionExample: (Anagrams) How many “words” can be formed using theletters in the word “card”?

Analysis:

1. Domain: 4 Positions

2. Qualities: none that matter

3. Roles: c, a, r, d

4. Replacement: no

D = 4 Positions

and S is

C A R DD D D D

1 1 1 14 3 2 1 = 24

Page 298: Finite Math Lecture Slides

Alternate SolutionExample: (Anagrams) How many “words” can be formed using theletters in the word “card”?

Analysis:

1. Domain: 4 Positions

2. Qualities: none that matter

3. Roles: c, a, r, d

4. Replacement: no

D = 4 Positions

and S is

C A R DD D D D

1 1 1 14 3 2 1 = 24

Page 299: Finite Math Lecture Slides

Example: How many “words” can be formed using the letters in theword “cool”?

Analysis:

1. Domain: 4 Positions

2. Qualities: none that matter

3. Roles: c, o, l

4. Replacement: no

D = 4 Positions

and S is

C O LD D D

1 2 1C(4, 1) C(3, 2) C(1, 1) = 12

Page 300: Finite Math Lecture Slides

Example: How many “words” can be formed using the letters in theword “cool”?

Analysis:

1. Domain: 4 Positions

2. Qualities: none that matter

3. Roles: c, o, l

4. Replacement: no

D = 4 Positions

and S is

C O LD D D

1 2 1C(4, 1) C(3, 2) C(1, 1) = 12

Page 301: Finite Math Lecture Slides

Example: How many “words” can be formed using the letters in theword “cool”?

Analysis:

1. Domain: 4 Positions

2. Qualities: none that matter

3. Roles: c, o, l

4. Replacement: no

D = 4 Positions

and S is

C O LD D D

1 2 1C(4, 1) C(3, 2) C(1, 1) = 12

Page 302: Finite Math Lecture Slides

Example: How many “words” can be formed using the letters in theword “bananas”?

Analysis:

1. Domain: 7 Positions

2. Qualities: none that matter

3. Roles: b, a, n, s

4. Replacement: no

D = 7 Positions

and S is

B A N SD D D D

1 3 2 1C(7, 1) C(6, 3) C(3, 2) C(1, 1) = 420

Page 303: Finite Math Lecture Slides

Example: How many “words” can be formed using the letters in theword “bananas”?

Analysis:

1. Domain: 7 Positions

2. Qualities: none that matter

3. Roles: b, a, n, s

4. Replacement: no

D = 7 Positions

and S is

B A N SD D D D

1 3 2 1C(7, 1) C(6, 3) C(3, 2) C(1, 1) = 420

Page 304: Finite Math Lecture Slides

Example: How many “words” can be formed using the letters in theword “bananas”?

Analysis:

1. Domain: 7 Positions

2. Qualities: none that matter

3. Roles: b, a, n, s

4. Replacement: no

D = 7 Positions

and S is

B A N SD D D D

1 3 2 1C(7, 1) C(6, 3) C(3, 2) C(1, 1) = 420

Page 305: Finite Math Lecture Slides

Compound Problems

Some problems are like Russian dolls:

problems inside problems.

You’ll need to use multiple strategies on a single problem.

Page 306: Finite Math Lecture Slides

Example: The city council has 5 Democratic and 3 Republicanmembers. The mayor is to appoint 3 or 4 of the council membersto a committee. How many different outcomes are possible?

Analysis:

1. Domain: 8 council members

2. Qualities: none that matter

3. Roles: none

4. Replacement: no

Let D = “8 council members”, then S isD

3C(8, 3) 56

4C(8, 4) 70

126

.

Page 307: Finite Math Lecture Slides

Example: The city council has 5 Democratic and 3 Republicanmembers. The mayor is to appoint 3 or 4 of the council membersto a committee. How many different outcomes are possible?

Analysis:

1. Domain: 8 council members

2. Qualities: none that matter

3. Roles: none

4. Replacement: no

Let D = “8 council members”, then S isD

3C(8, 3) 56

4C(8, 4) 70

126

.

Page 308: Finite Math Lecture Slides

Example: The city council has 5 Democratic and 3 Republicanmembers. The mayor is to appoint 3 or 4 of the council membersto a committee. How many different outcomes are possible?

Analysis:

1. Domain: 8 council members

2. Qualities: none that matter

3. Roles: none

4. Replacement: no

Let D = “8 council members”, then S isD

3C(8, 3) 56

4C(8, 4) 70

126

.

Page 309: Finite Math Lecture Slides

Example: The city council has 5 Democratic and 3 Republicanmembers. The mayor is to appoint 3 of the council members to abipartisan committee. How many different outcomes are possible?

Analysis:

1. Domain: 8 council members2. Qualities: Dem, Rep3. Roles: none that matter4. Replacement: no

D = 5 Democrats R = 3 Republicans

and S is D R

1 2C(5, 1) C(3, 2) 15

2 1C(5, 2) C(3, 1) 30

45

Page 310: Finite Math Lecture Slides

Example: The city council has 5 Democratic and 3 Republicanmembers. The mayor is to appoint 3 of the council members to abipartisan committee. How many different outcomes are possible?

Analysis:

1. Domain: 8 council members2. Qualities: Dem, Rep3. Roles: none that matter4. Replacement: no

D = 5 Democrats R = 3 Republicans

and S is D R

1 2C(5, 1) C(3, 2) 15

2 1C(5, 2) C(3, 1) 30

45

Page 311: Finite Math Lecture Slides

Example: The city council has 5 Democratic and 3 Republicanmembers. The mayor is to appoint 3 of the council members to abipartisan committee. How many different outcomes are possible?

Analysis:

1. Domain: 8 council members2. Qualities: Dem, Rep3. Roles: none that matter4. Replacement: no

D = 5 Democrats R = 3 Republicans

and S is D R

1 2C(5, 1) C(3, 2) 15

2 1C(5, 2) C(3, 1) 30

45

Page 312: Finite Math Lecture Slides

Example: The city council has 5 Democratic and 3 Republicanmembers. The mayor is to appoint 3 of the council members to acommittee with at most 1 Democrat. How many differentoutcomes are possible?

Analysis:

1. Domain: 8 council members2. Qualities: Dem, Rep3. Roles: none that matter4. Replacement: no

D = 5 Democrats R = 3 Republicans

and then S is D R

0 3C(5, 0) C(3, 3) 1

1 2C(5, 1) C(3, 2) 15

16

Page 313: Finite Math Lecture Slides

Example: The city council has 5 Democratic and 3 Republicanmembers. The mayor is to appoint 3 of the council members to acommittee with at most 1 Democrat. How many differentoutcomes are possible?

Analysis:

1. Domain: 8 council members2. Qualities: Dem, Rep3. Roles: none that matter4. Replacement: no

D = 5 Democrats R = 3 Republicans

and then S is D R

0 3C(5, 0) C(3, 3) 1

1 2C(5, 1) C(3, 2) 15

16

Page 314: Finite Math Lecture Slides

Example: The city council has 5 Democratic and 3 Republicanmembers. The mayor is to appoint 3 of the council members to acommittee with at most 1 Democrat. How many differentoutcomes are possible?

Analysis:

1. Domain: 8 council members2. Qualities: Dem, Rep3. Roles: none that matter4. Replacement: no

D = 5 Democrats R = 3 Republicans

and then S is D R

0 3C(5, 0) C(3, 3) 1

1 2C(5, 1) C(3, 2) 15

16

Page 315: Finite Math Lecture Slides

Example: The city council has 5 Democratic and 3 Republicanmembers. The mayor is to appoint 3 of the council members to acommittee with at least 1 Democrat. How many differentoutcomes are possible?

Analysis: House Subcommittee Paradigm

5 Dems 3 Reps

1 2C(5, 1) C(3, 2) 15

2 1C(5, 2) C(3, 1) 30

3 0C(5, 3) C(3, 0) 10

55

Page 316: Finite Math Lecture Slides

Example: The city council has 5 Democratic and 3 Republicanmembers. The mayor is to appoint 3 of the council members to acommittee with at least 1 Democrat. How many differentoutcomes are possible?

Analysis: House Subcommittee Paradigm

5 Dems 3 Reps

1 2C(5, 1) C(3, 2) 15

2 1C(5, 2) C(3, 1) 30

3 0C(5, 3) C(3, 0) 10

55

Page 317: Finite Math Lecture Slides

Example: The city council has 5 Democratic and 3 Republicanmembers. The mayor is to appoint 3 of the council members to acommittee with at least 1 Democrat. How many differentoutcomes are possible?

Analysis: House Subcommittee Paradigm

5 Dems 3 Reps

1 2C(5, 1) C(3, 2) 15

2 1C(5, 2) C(3, 1) 30

3 0C(5, 3) C(3, 0) 10

55

Page 318: Finite Math Lecture Slides

Alternate Method: by Complement

Example: The city council has 5 Democratic and 3 Republicanmembers. The mayor is to appoint 3 of the council members to acommittee with at least 1 Democrat. How many differentoutcomes are possible?

Analysis:

ST = all outcomes with no restrictionsSR = outcomes with at least 1 DemS ′R = outcomes with no Dems

n[ST ] = n[SR ] + n[S ′R ]n[ST ]− n[S ′R ] = n[SR ]

C(8, 3)− C(3, 3) =56− 1 = 55

Page 319: Finite Math Lecture Slides

Alternate Method: by Complement

Example: The city council has 5 Democratic and 3 Republicanmembers. The mayor is to appoint 3 of the council members to acommittee with at least 1 Democrat. How many differentoutcomes are possible?

Analysis:

ST = all outcomes with no restrictionsSR = outcomes with at least 1 DemS ′R = outcomes with no Dems

n[ST ] = n[SR ] + n[S ′R ]n[ST ]− n[S ′R ] = n[SR ]

C(8, 3)− C(3, 3) =56− 1 = 55

Page 320: Finite Math Lecture Slides

Alternate Method: by Complement

Example: The city council has 5 Democratic and 3 Republicanmembers. The mayor is to appoint 3 of the council members to acommittee with at least 1 Democrat. How many differentoutcomes are possible?

Analysis:

ST = all outcomes with no restrictionsSR = outcomes with at least 1 DemS ′R = outcomes with no Dems

n[ST ] = n[SR ] + n[S ′R ]

n[ST ]− n[S ′R ] = n[SR ]C(8, 3)− C(3, 3) =

56− 1 = 55

Page 321: Finite Math Lecture Slides

Alternate Method: by Complement

Example: The city council has 5 Democratic and 3 Republicanmembers. The mayor is to appoint 3 of the council members to acommittee with at least 1 Democrat. How many differentoutcomes are possible?

Analysis:

ST = all outcomes with no restrictionsSR = outcomes with at least 1 DemS ′R = outcomes with no Dems

n[ST ] = n[SR ] + n[S ′R ]n[ST ]− n[S ′R ] = n[SR ]

C(8, 3)− C(3, 3) =56− 1 = 55

Page 322: Finite Math Lecture Slides

Alternate Method: by Complement

Example: The city council has 5 Democratic and 3 Republicanmembers. The mayor is to appoint 3 of the council members to acommittee with at least 1 Democrat. How many differentoutcomes are possible?

Analysis:

ST = all outcomes with no restrictionsSR = outcomes with at least 1 DemS ′R = outcomes with no Dems

n[ST ] = n[SR ] + n[S ′R ]n[ST ]− n[S ′R ] = n[SR ]

C(8, 3)−

C(3, 3) =56− 1 = 55

Page 323: Finite Math Lecture Slides

Alternate Method: by Complement

Example: The city council has 5 Democratic and 3 Republicanmembers. The mayor is to appoint 3 of the council members to acommittee with at least 1 Democrat. How many differentoutcomes are possible?

Analysis:

ST = all outcomes with no restrictionsSR = outcomes with at least 1 DemS ′R = outcomes with no Dems

n[ST ] = n[SR ] + n[S ′R ]n[ST ]− n[S ′R ] = n[SR ]

C(8, 3)− C(3, 3) =56− 1 = 55

Page 324: Finite Math Lecture Slides

The Complement Principle

The symbol ′ denotes the complement of a set:

A′ = S A = { x | x ∈ S & x 6∈ A }.

Then:

In general,

n[A] = n[S ]− n[A′].

The Complement Principle (CP) is worth considering if theproblem contains phrases like “at least 1” or “at most 5”.

Page 325: Finite Math Lecture Slides

The Complement Principle

The symbol ′ denotes the complement of a set:

A′ = S A = { x | x ∈ S & x 6∈ A }.

Then:

In general,

n[A] = n[S ]− n[A′].

The Complement Principle (CP) is worth considering if theproblem contains phrases like “at least 1” or “at most 5”.

Page 326: Finite Math Lecture Slides

Example: The House committee on Education has 8 Democraticand 9 Republican members. They want to form a subcommittee of7 to investigate corruption in Finite Math, and the subcommitteemust have at least 1 Democrat. How many different outcomes arepossible?

Analysis: House Subcommittee Paradigm

S = all outcomes with no restrictionsE = outcomes with at least 1 Dem

E ′ = outcomes with no Dems= outcomes with all Reps

Then by the CP,

n[E ] = n[S ]− n[E ′]= C(17, 7)− C(9, 7)= 19412.

Page 327: Finite Math Lecture Slides

Example: The House committee on Education has 8 Democraticand 9 Republican members. They want to form a subcommittee of7 to investigate corruption in Finite Math, and the subcommitteemust have at least 1 Democrat. How many different outcomes arepossible?

Analysis: House Subcommittee Paradigm

S = all outcomes with no restrictionsE = outcomes with at least 1 Dem

E ′ = outcomes with no Dems= outcomes with all Reps

Then by the CP,

n[E ] = n[S ]− n[E ′]= C(17, 7)− C(9, 7)= 19412.

Page 328: Finite Math Lecture Slides

Example: The House committee on Education has 8 Democraticand 9 Republican members. They want to form a subcommittee of7 to investigate corruption in Finite Math, and the subcommitteemust have at least 1 Democrat. How many different outcomes arepossible?

Analysis: House Subcommittee Paradigm

S = all outcomes with no restrictionsE = outcomes with at least 1 Dem

E ′ = outcomes with no Dems= outcomes with all Reps

Then by the CP,

n[E ] = n[S ]− n[E ′]

= C(17, 7)− C(9, 7)= 19412.

Page 329: Finite Math Lecture Slides

Example: The House committee on Education has 8 Democraticand 9 Republican members. They want to form a subcommittee of7 to investigate corruption in Finite Math, and the subcommitteemust have at least 1 Democrat. How many different outcomes arepossible?

Analysis: House Subcommittee Paradigm

S = all outcomes with no restrictionsE = outcomes with at least 1 Dem

E ′ = outcomes with no Dems= outcomes with all Reps

Then by the CP,

n[E ] = n[S ]− n[E ′]= C(17, 7)− C(9, 7)= 19412.

Page 330: Finite Math Lecture Slides

Example: A class has 5 sophomores, 7 juniors, and 6 seniors. If 3students are selected to give a report, how many different outcomesare possible in which the students are not all in the same year?

Analysis:

S = all outcomes with no restrictionsE = outcomes with students not all in same year

E ′ = outcomes with students all in same yearn[E ] = n[S ]− n[E ′] (CP)

And we easily get

n[S ] = C(18, 3) = 816.

But what is n[E ′]?

Page 331: Finite Math Lecture Slides

Example: A class has 5 sophomores, 7 juniors, and 6 seniors. If 3students are selected to give a report, how many different outcomesare possible in which the students are not all in the same year?

Analysis:

S = all outcomes with no restrictionsE = outcomes with students not all in same year

E ′ = outcomes with students all in same yearn[E ] = n[S ]− n[E ′] (CP)

And we easily get

n[S ] = C(18, 3) = 816.

But what is n[E ′]?

Page 332: Finite Math Lecture Slides

Subproblem: A class has 5 sophomores, 7 juniors, and 6 seniors. If3 students are selected to give a report, how many differentoutcomes are possible in which all students are in the same year?

Analysis:

Year5 Soph 7 Jun 6 Sen size

Alt

ern

ativ

es

3 0 0C(5, 3) = 10 1 1 10

0 3 01 C(7, 3) = 35 1 350 0 31 1 C(6, 3) = 20 20

65

Page 333: Finite Math Lecture Slides

Subproblem: A class has 5 sophomores, 7 juniors, and 6 seniors. If3 students are selected to give a report, how many differentoutcomes are possible in which all students are in the same year?

Analysis:

Year5 Soph 7 Jun 6 Sen size

Alt

ern

ativ

es

3 0 0C(5, 3) = 10 1 1 10

0 3 01 C(7, 3) = 35 1 350 0 31 1 C(6, 3) = 20 20

65

Page 334: Finite Math Lecture Slides

Subproblem: A class has 5 sophomores, 7 juniors, and 6 seniors. If3 students are selected to give a report, how many differentoutcomes are possible in which all students are in the same year?

Analysis:

Year5 Soph 7 Jun 6 Sen size

Alt

ern

ativ

es

3 0 0C(5, 3) = 10 1 1 10

0 3 01 C(7, 3) = 35 1 350 0 31 1 C(6, 3) = 20 20

65

Page 335: Finite Math Lecture Slides

Example: (cont.) A class has 5 sophomores, 7 juniors, and 6seniors. If 3 students are selected to give a report, how manydifferent outcomes are possible in which the students are not all inthe same year?

Analysis:

S = all outcomes with no restrictionsE = outcomes with students not all in same year

E ′ = outcomes with students all in same yearn[E ] = n[S ]− n[E ′] (CP)

n[S ] = 816n[E ′] = 65n[E ] = 816− 65 = 751

Page 336: Finite Math Lecture Slides

Example: (cont.) A class has 5 sophomores, 7 juniors, and 6seniors. If 3 students are selected to give a report, how manydifferent outcomes are possible in which the students are not all inthe same year?

Analysis:

S = all outcomes with no restrictionsE = outcomes with students not all in same year

E ′ = outcomes with students all in same yearn[E ] = n[S ]− n[E ′] (CP)

n[S ] = 816n[E ′] = 65n[E ] = 816− 65 = 751

Page 337: Finite Math Lecture Slides

Example: The math chairperson has a meeting at 11 to decide onthe recipient of this year’s Outstanding Finite Student award. Healso has time for meetings at 10, 12, and 2. He has three othertasks to do today: meet with the winner of the award, draft abudget proposal, and finalize the summer teaching schedule. Howmany ways are there to schedule his day?

Analysis:

1. Objects: 3 tasks

2. Qualities: award and nonaward

3. Roles: 10, 12, 2

4. Replacement: no

Page 338: Finite Math Lecture Slides

Example: The math chairperson has a meeting at 11 to decide onthe recipient of this year’s Outstanding Finite Student award. Healso has time for meetings at 10, 12, and 2. He has three othertasks to do today: meet with the winner of the award, draft abudget proposal, and finalize the summer teaching schedule. Howmany ways are there to schedule his day?

Analysis:

1. Objects: 3 tasks

2. Qualities: award and nonaward

3. Roles: 10, 12, 2

4. Replacement: no

Page 339: Finite Math Lecture Slides

Example: The math chairperson has a meeting at 11 to decide onthe recipient of this year’s Outstanding Finite Student award. Healso has time for meetings at 10, 12, and 2. He has three othertasks to do today: meet with the winner of the award, draft abudget proposal, and finalize the summer teaching schedule. Howmany ways are there to schedule his day?

Analysis:

1. Objects: 3 tasks

2. Qualities: award and nonaward

3. Roles: 10, 12, 2

4. Replacement: no

Page 340: Finite Math Lecture Slides

Analysis:

1. Objects: 3 tasks

2. Qualities: award and nonaward

3. Roles: 10, 12, 2

4. Replacement: no

D = 3 tasksA = meeting with award winnerN = 2 other meetings

S :

10 12 2D D D

1 1 13 2 1 6

Page 341: Finite Math Lecture Slides

Analysis:

1. Objects: 3 tasks

2. Qualities: award and nonaward

3. Roles: 10, 12, 2

4. Replacement: no

D = 3 tasksA = meeting with award winnerN = 2 other meetings

S :

10 12 2D D D

1 1 13 2 1 6

Page 342: Finite Math Lecture Slides

Analysis:

1. Objects: 3 tasks

2. Qualities: award and nonaward

3. Roles: 10, 12, 2

4. Replacement: no

D = 3 tasksA = meeting with award winnerN = 2 other meetings

E = “feasible schedules”:

10 12 2N D D

1 1 12 2 1 4

Page 343: Finite Math Lecture Slides

Example: A department chairperson has 4 tasks to do today: selectan award recipient; have a meeting with this recipient; write aletter to the dean; and prepare the department’s budget request fornext year. If he schedules these tasks one after another, how manydifferent schedules are possible. (Note that he cannot meet withthe committee until after he has selected its members.)

Analysis:

1. Objects: 4 tasks

2. Qualities: award and nonaward

3. Roles: 4 time slots

4. Replacement: no

We can break this problem down into alternatives.

Page 344: Finite Math Lecture Slides

Example: A department chairperson has 4 tasks to do today: selectan award recipient; have a meeting with this recipient; write aletter to the dean; and prepare the department’s budget request fornext year. If he schedules these tasks one after another, how manydifferent schedules are possible. (Note that he cannot meet withthe committee until after he has selected its members.)

Analysis:

1. Objects: 4 tasks

2. Qualities: award and nonaward

3. Roles: 4 time slots

4. Replacement: no

We can break this problem down into alternatives.

Page 345: Finite Math Lecture Slides

Example: A department chairperson has 4 tasks to do today: selectan award recipient; have a meeting with this recipient; write aletter to the dean; and prepare the department’s budget request fornext year. If he schedules these tasks one after another, how manydifferent schedules are possible. (Note that he cannot meet withthe committee until after he has selected its members.)

Analysis:

1. Objects: 4 tasks

2. Qualities: award and nonaward

3. Roles: 4 time slots

4. Replacement: no

We can break this problem down into alternatives.

Page 346: Finite Math Lecture Slides

C = meeting to choose award winnerN = 3 other meetings, including one with awardee

and E = “feasible schedules” is

1st 2nd 3rd 4th

C N N N1 3 2 1 6

N C N N2 1 2 1 4N N C N2 1 1 1 2N N N C2 1 0 1 0

12

Since we are always picking just one task for each slotI have omitted all the 1’s.

Page 347: Finite Math Lecture Slides

C = meeting to choose award winnerN = 3 other meetings, including one with awardee

and E = “feasible schedules” is

1st 2nd 3rd 4th

C N N N1 3 2 1 6N C N N2 1 2 1 4N N C N2 1 1 1 2N N N C2 1 0 1 0

12

Since we are always picking just one task for each slotI have omitted all the 1’s.

Page 348: Finite Math Lecture Slides

Example: The Personnel Manager at a company must schedule 4job interviews and 3 different committee meetings for tomorrow. Ifshe decides to schedule the job interviews one after another and toschedule the committee meetings one after another, how manydifferent schedules are possible?

Analysis:

1. Objects: 7 tasks

2. Qualities: interviews, meetings

3. Roles: 2 time blocks

4. Replacement: no

Page 349: Finite Math Lecture Slides

Example: The Personnel Manager at a company must schedule 4job interviews and 3 different committee meetings for tomorrow. Ifshe decides to schedule the job interviews one after another and toschedule the committee meetings one after another, how manydifferent schedules are possible?

Analysis:

1. Objects: 7 tasks

2. Qualities: interviews, meetings

3. Roles: 2 time blocks

4. Replacement: no

Page 350: Finite Math Lecture Slides

LetI = 4 job interviews

M = 3 committee meetings

then E = “desired schedules” is

Block 1 Block 2I M4 3

P(4, 4) P(3, 3) 144M I3 4

P(3, 3) P(4, 4) 144

288

Page 351: Finite Math Lecture Slides

Example: A student has 4 novels by Thomas Hardy, 2 novels byGeorge Eliot, and 5 novels by Charles Dickens. In how manydifferent ways could she arrange these books on a book shelf (fromleft to right) if each author’s books are kept together?

Analysis:

1. Objects: 11 books

2. Qualities: Hardy, Eliot, Dickens

3. Roles: 3 Blocks

4. Replacement: no

H = 4 Hardy novelsE = 2 Eliot novelsD = 5 Dickens novels

Page 352: Finite Math Lecture Slides

Example: A student has 4 novels by Thomas Hardy, 2 novels byGeorge Eliot, and 5 novels by Charles Dickens. In how manydifferent ways could she arrange these books on a book shelf (fromleft to right) if each author’s books are kept together?

Analysis:

1. Objects: 11 books

2. Qualities: Hardy, Eliot, Dickens

3. Roles: 3 Blocks

4. Replacement: no

H = 4 Hardy novelsE = 2 Eliot novelsD = 5 Dickens novels

Page 353: Finite Math Lecture Slides

Example: A student has 4 novels by Thomas Hardy, 2 novels byGeorge Eliot, and 5 novels by Charles Dickens. In how manydifferent ways could she arrange these books on a book shelf (fromleft to right) if each author’s books are kept together?

Analysis:

1. Objects: 11 books

2. Qualities: Hardy, Eliot, Dickens

3. Roles: 3 Blocks

4. Replacement: no

H = 4 Hardy novelsE = 2 Eliot novelsD = 5 Dickens novels

Page 354: Finite Math Lecture Slides

Block 1 Block 2 Block 3H E DP(4, 4) P(2, 2) P(5, 5) 5760H D EP(4, 4) P(5, 5) P(2, 2) 5760E H DP(2, 2) P(4, 4) P(5, 5) 5760E D HP(2, 2) P(5, 5) P(4, 4) 5760D H EP(5, 5) P(4, 4) P(2, 2) 5760D E HP(5, 5) P(2, 2) P(4, 4) 5760

34560

Page 355: Finite Math Lecture Slides

Alternate Solution

Note that in each row the qualities are anagrams of HED.

So the number of rows must be C(3, 1)C(2, 1)C(1, 1) = 6.

Since each row has the same size,

n[E ] = 6 · 5760 = 34560.

This is an example of the Same Size Principle (SSP).

Page 356: Finite Math Lecture Slides

Alternate Solution

Note that in each row the qualities are anagrams of HED.

So the number of rows must be C(3, 1)C(2, 1)C(1, 1) = 6.

Since each row has the same size,

n[E ] = 6 · 5760 = 34560.

This is an example of the Same Size Principle (SSP).

Page 357: Finite Math Lecture Slides

Alternate Solution

Note that in each row the qualities are anagrams of HED.

So the number of rows must be C(3, 1)C(2, 1)C(1, 1) = 6.

Since each row has the same size,

n[E ] = 6 · 5760 = 34560.

This is an example of the Same Size Principle (SSP).

Page 358: Finite Math Lecture Slides

Alternate Solution

Note that in each row the qualities are anagrams of HED.

So the number of rows must be C(3, 1)C(2, 1)C(1, 1) = 6.

Since each row has the same size,

n[E ] = 6 · 5760 = 34560.

This is an example of the Same Size Principle (SSP).

Page 359: Finite Math Lecture Slides

Block 1 Block 2 Block 3

6

H E D

P(4, 4) P(2, 2) P(5, 5) 5760H D EP(4, 4) P(5, 5) P(2, 2) 5760E H DP(2, 2) P(4, 4) P(5, 5) 5760E D HP(2, 2) P(5, 5) P(4, 4) 5760D H EP(5, 5) P(4, 4) P(2, 2) 5760D E HP(5, 5) P(2, 2) P(4, 4) 5760

6 · 5760 = 34560

Page 360: Finite Math Lecture Slides

Example: In how many ways could Anne, Bill, Carol, Dan, and Ellenbe arranged in a line for a picture if Anne and Dan insist onstanding next to one another?

Analysis:

1. Objects: 5 people

2. Qualities: Ann, Dan, Others

3. Roles: 5 positions

4. Replacement: no

A = AnnD = DanP = 3 people besides Ann and Dan

Page 361: Finite Math Lecture Slides

Example: In how many ways could Anne, Bill, Carol, Dan, and Ellenbe arranged in a line for a picture if Anne and Dan insist onstanding next to one another?

Analysis:

1. Objects: 5 people

2. Qualities: Ann, Dan, Others

3. Roles: 5 positions

4. Replacement: no

A = AnnD = DanP = 3 people besides Ann and Dan

Page 362: Finite Math Lecture Slides

Then E = “desired orders” is

1st 2nd 3rd 4th 5th

8

A D P P P1 1 3 2 1 6

D A P P P1 1 3 2 1 6P A D P P3 1 1 2 1 6P D A P PP P A D PP P D A PP P P A DP P P D A

8 · 6 = 48

Page 363: Finite Math Lecture Slides

Alternative Method

There are 4 blocks of people:AD and B, C , and E .

1st 2nd 3rd 4th

P(4, 4) = 24

AD B C E2 1 1 1 2

E AD C B1 2 1 1 2

......

......

...

24 · 2 = 48

Page 364: Finite Math Lecture Slides

Alternative Method

There are 4 blocks of people:AD and B, C , and E .

1st 2nd 3rd 4th

P(4, 4) = 24

AD B C E

2 1 1 1 2

E AD C B1 2 1 1 2

......

......

...

24 · 2 = 48

Page 365: Finite Math Lecture Slides

Alternative Method

There are 4 blocks of people:AD and B, C , and E .

1st 2nd 3rd 4th

P(4, 4) = 24

AD B C E2 1 1 1 2

E AD C B1 2 1 1 2

......

......

...

24 · 2 = 48

Page 366: Finite Math Lecture Slides

Alternative Method

There are 4 blocks of people:AD and B, C , and E .

1st 2nd 3rd 4th

P(4, 4) = 24

AD B C E2 1 1 1 2

E AD C B1 2 1 1 2

......

......

...

24 · 2 = 48

Page 367: Finite Math Lecture Slides

Alternative Method

There are 4 blocks of people:AD and B, C , and E .

1st 2nd 3rd 4th

P(4, 4) = 24

AD B C E2 1 1 1 2

E AD C B1 2 1 1 2

......

......

...

24 · 2 = 48

Page 368: Finite Math Lecture Slides

Example: A committee has 5 freshmen, 5 sophomores, and 5juniors. In how many different ways could a panel consisting of 2people from each of 2 classes be selected?

1. Objects: 15 students

2. Qualities: Freshmen, Sophomores, Juniors

3. Roles: none that matter

4. Replacement: no

Freshmen Sophomores Juniors

2 2 0C(5, 2) C(5, 2) C(5, 0) 100

2 0 2C(5, 2) C(5, 0) C(5, 2) 100

0 2 2C(5, 0) C(5, 2) C(5, 2) 100

300

Note the three cases are all “anagrams” of 2, 2, 0.

Page 369: Finite Math Lecture Slides

Example: A committee has 5 freshmen, 5 sophomores, and 5juniors. In how many different ways could a panel consisting of 2people from each of 2 classes be selected?

1. Objects: 15 students

2. Qualities: Freshmen, Sophomores, Juniors

3. Roles: none that matter

4. Replacement: no

Freshmen Sophomores Juniors

2 2 0C(5, 2) C(5, 2) C(5, 0) 100

2 0 2C(5, 2) C(5, 0) C(5, 2) 100

0 2 2C(5, 0) C(5, 2) C(5, 2) 100

300

Note the three cases are all “anagrams” of 2, 2, 0.

Page 370: Finite Math Lecture Slides

Example: A committee has 5 freshmen, 5 sophomores, and 5juniors. In how many different ways could a panel consisting of 2people from each of 2 classes be selected?

1. Objects: 15 students

2. Qualities: Freshmen, Sophomores, Juniors

3. Roles: none that matter

4. Replacement: no

Freshmen Sophomores Juniors

2 2 0C(5, 2) C(5, 2) C(5, 0) 100

2 0 2C(5, 2) C(5, 0) C(5, 2) 100

0 2 2C(5, 0) C(5, 2) C(5, 2) 100

300

Note the three cases are all “anagrams” of 2, 2, 0.

Page 371: Finite Math Lecture Slides

Example: A committee has 5 freshmen, 5 sophomores, 5 juniors,and 5 seniors. In how many different ways could a panel be chosenif it must consist of 2 people from one class and 1 person eachfrom two other classes?

Analysis:

1. Objects: 20 students

2. Qualities: Freshmen, Sophomores, Juniors, Seniors

3. Roles: none that matter

4. Replacement: no

Page 372: Finite Math Lecture Slides

Example: A committee has 5 freshmen, 5 sophomores, 5 juniors,and 5 seniors. In how many different ways could a panel be chosenif it must consist of 2 people from one class and 1 person eachfrom two other classes?

Analysis:

1. Objects: 20 students

2. Qualities: Freshmen, Sophomores, Juniors, Seniors

3. Roles: none that matter

4. Replacement: no

Page 373: Finite Math Lecture Slides

The “desired panels” can be described as follows.

Freshmen Sophomores Juniors Seniors

12

2 1 1 0C(5, 2) C(5, 1) C(5, 1) C(5, 0) 250

1 1 0 2C(5, 1) C(5, 1) C(5, 0) C(5, 2) 250

......

......

...

3000

Why 12?

# anagrams of 2110 = C(4, 1)C(3, 2)C(1, 1) = 12

Page 374: Finite Math Lecture Slides

The “desired panels” can be described as follows.

Freshmen Sophomores Juniors Seniors

12

2 1 1 0C(5, 2) C(5, 1) C(5, 1) C(5, 0) 250

1 1 0 2C(5, 1) C(5, 1) C(5, 0) C(5, 2) 250

......

......

...

3000

Why 12?

# anagrams of 2110 = C(4, 1)C(3, 2)C(1, 1) = 12

Page 375: Finite Math Lecture Slides

Example: How many different hands consisting of 5 cards from aregular deck of 52 cards (i.e., four suits — spades, hearts,diamonds, and clubs — with thirteen cards each and no jokers)have 3 cards from one suit and 2 cards from one other suit?

Analysis:

1. Objects: 52 cards

2. Qualities: 13 ♠, 13 ♥, 13 ♦, 13 ♣3. Roles: none that matter

4. Replacement: no

Page 376: Finite Math Lecture Slides

Example: How many different hands consisting of 5 cards from aregular deck of 52 cards (i.e., four suits — spades, hearts,diamonds, and clubs — with thirteen cards each and no jokers)have 3 cards from one suit and 2 cards from one other suit?

Analysis:

1. Objects: 52 cards

2. Qualities: 13 ♠, 13 ♥, 13 ♦, 13 ♣3. Roles: none that matter

4. Replacement: no

Page 377: Finite Math Lecture Slides

The desired hands can be described as follows.

♠ ♥ ♦ ♣

12

3 2 0 0C(13, 3) C(13, 2) C(13, 0) C(13, 0) 22308

2 0 3 0C(13, 2) C(13, 0) C(13, 3) C(13, 0) 22308

......

......

...

267696

12 = # anagrams of 3200 = C(4, 1)C(3, 1)C(2, 2)

Page 378: Finite Math Lecture Slides

Example: A small retail chain has vacancies for 4 store managers allin different cities. Because the positions are in different cities, noone could hold more than one of these positions. The company has6 female assistant managers and 5 male assistant managers whoare eligible for promotion to these store manager positions. Howmany outcomes are there in which 2 women and 2 men arepromoted?

Analysis:

1. Objects: 11 managers

2. Qualities: 6 female, 5 male

3. Roles: 4 stores

4. Replacement: no

Page 379: Finite Math Lecture Slides

Example: A small retail chain has vacancies for 4 store managers allin different cities. Because the positions are in different cities, noone could hold more than one of these positions. The company has6 female assistant managers and 5 male assistant managers whoare eligible for promotion to these store manager positions. Howmany outcomes are there in which 2 women and 2 men arepromoted?

Analysis:

1. Objects: 11 managers

2. Qualities: 6 female, 5 male

3. Roles: 4 stores

4. Replacement: no

Page 380: Finite Math Lecture Slides

F = 6 femalesM = 5 males

Store 1 Store 2 Store 3 Store 4

C(4, 2)C(2, 2) = 6

F F M M

6 5 5 4 600F M M F6 5 4 5 600

......

......

...

3600

Page 381: Finite Math Lecture Slides

Example: 9 seniors and 4 juniors in English each submitted onepoem for the English Department’s annual poetry competition.The department will award 1st, 2nd, 3rd, 4th, and 5th prizes. Inhow many overall outcomes would seniors be selected for exactly 3prizes?

Analysis:

1. Objects: 13 students

2. Qualities: 9 Seniors, 4 Juniors

3. Roles: 5 prizes

4. Replacement: no

Page 382: Finite Math Lecture Slides

Example: 9 seniors and 4 juniors in English each submitted onepoem for the English Department’s annual poetry competition.The department will award 1st, 2nd, 3rd, 4th, and 5th prizes. Inhow many overall outcomes would seniors be selected for exactly 3prizes?

Analysis:

1. Objects: 13 students

2. Qualities: 9 Seniors, 4 Juniors

3. Roles: 5 prizes

4. Replacement: no

Page 383: Finite Math Lecture Slides

N = 9 SeniorsJ = 4 Juniors

1st 2nd 3rd 4th 5th

C(5, 3) = 10

N N N J J

9 8 7 4 3 6048N J N N J9 4 8 7 3 6048

......

......

......

60480

Page 384: Finite Math Lecture Slides

Equally Likely OutcomesWhen all outcomes are equally likely, probabilities are easy tocompute.

Example: There are 15 people in a club, including Adrienne andBen, who run the door prize committee. Two people are randomlychosen to receive door prizes, and it turns out to be Adrienne andBen! What is the probability of that happening?

Solution: For the sample space,

n[S ] = C(15, 2) = 105.

The event we’re interested in is

E = A and B both get prizes

and n[E ] = 1. Then

Pr[E ] =1

105< 1%.

Page 385: Finite Math Lecture Slides

Equally Likely OutcomesWhen all outcomes are equally likely, probabilities are easy tocompute.

Example: There are 15 people in a club, including Adrienne andBen, who run the door prize committee. Two people are randomlychosen to receive door prizes, and it turns out to be Adrienne andBen! What is the probability of that happening?

Solution: For the sample space,

n[S ] = C(15, 2) = 105.

The event we’re interested in is

E = A and B both get prizes

and n[E ] = 1.

Then

Pr[E ] =1

105< 1%.

Page 386: Finite Math Lecture Slides

Equally Likely OutcomesWhen all outcomes are equally likely, probabilities are easy tocompute.

Example: There are 15 people in a club, including Adrienne andBen, who run the door prize committee. Two people are randomlychosen to receive door prizes, and it turns out to be Adrienne andBen! What is the probability of that happening?

Solution: For the sample space,

n[S ] = C(15, 2) = 105.

The event we’re interested in is

E = A and B both get prizes

and n[E ] = 1. Then

Pr[E ] =1

105< 1%.

Page 387: Finite Math Lecture Slides

Formula for ELO Probability

In general,

Pr[E ] =n[E ]

n[S ].

Page 388: Finite Math Lecture Slides

Example: You are dealt 5 cards at random from a standard deck ofcards. What is the probability the hand is a full house (3 cards ofone rank and 2 of another)?

Analysis:S = all 5 card handsE = full houses

n[S ] = C(52, 5) = 2598960n[E ] = ?

Page 389: Finite Math Lecture Slides

Example: You are dealt 5 cards at random from a standard deck ofcards. What is the probability the hand is a full house (3 cards ofone rank and 2 of another)?

Analysis:S = all 5 card handsE = full houses

n[S ] = C(52, 5) = 2598960n[E ] = ?

Page 390: Finite Math Lecture Slides

n[E ] = ?:

2 3 4 5 6 7 8 9 10 J Q K A

156

3 2 0 0 0 0 0 0 0 0 0 0 04 6 24

0 0 0 0 2 0 0 0 0 0 0 0 36 4 24

......

......

......

......

......

......

...

3744

Note

4 = C(4, 3) 6 = C(4, 2)156 = C(13, 1)C(12, 1)C(11, 11) 3744 = 156 · 24.

156 is the number of anagrams of 3200000000000.

Page 391: Finite Math Lecture Slides

Example: You are dealt 5 cards at random from a standard deck ofcards. What is the probability the hand is a full house (3 cards ofone rank and 2 of another)?

Answer:Since n[E ] = 3744 and n[S ] = 2598960, therefore

Pr[E ] =n[E ]

n[S ]=

3744

2598960= .001441.

Page 392: Finite Math Lecture Slides

Conditional Probability

Partial information about an outcome can change the probability.

Example: If you know that someone won $60 million in thepowerball last night, the probability the winner was an IU studentis about

107, 160

6, 516, 922= .01644 = 1.644%.

But if you know the winner lived in Bloomington the probability

goes up to about1

3= 33%.

Page 393: Finite Math Lecture Slides

Conditional Probability

Partial information about an outcome can change the probability.

Example: If you know that someone won $60 million in thepowerball last night, the probability the winner was an IU studentis about

107, 160

6, 516, 922= .01644 = 1.644%.

But if you know the winner lived in Bloomington the probability

goes up to about1

3= 33%.

Page 394: Finite Math Lecture Slides

Example: A special die has the numbers 1–3 in yellow and 4–6 inred, otherwise the die is normal and fair. You roll the die, but youdon’t have your glasses on, so all you tell is that the number is red.What is the probability it is even?

Solution: Normally the probability of an even number is1

2, but

here we have the extra information that the number is red.

Of the 3 red numbers, 2 are even so the answer is2

3.

Pr[ E |R ] =2

3

Page 395: Finite Math Lecture Slides

Example: A special die has the numbers 1–3 in yellow and 4–6 inred, otherwise the die is normal and fair. You roll the die, but youdon’t have your glasses on, so all you tell is that the number is red.What is the probability it is even?

Solution: Normally the probability of an even number is1

2, but

here we have the extra information that the number is red.

Of the 3 red numbers, 2 are even so the answer is2

3.

Pr[ E |R ] =2

3

Page 396: Finite Math Lecture Slides

Example: A special die has the numbers 1–3 in yellow and 4–6 inred, otherwise the die is normal and fair. You roll the die, but youdon’t have your glasses on, so all you tell is that the number is red.What is the probability it is even?

Solution: Normally the probability of an even number is1

2, but

here we have the extra information that the number is red.

Of the 3 red numbers, 2 are even so the answer is2

3.

Pr[ E |R ] =2

3

Page 397: Finite Math Lecture Slides

Let E be the even numbers and R the red ones.

Page 398: Finite Math Lecture Slides

We know the number is red.

Page 399: Finite Math Lecture Slides

We’re wondering whether it is even.

Page 400: Finite Math Lecture Slides

So the answer is

Pr[ E |R ] =©

=2

3.

Page 401: Finite Math Lecture Slides

In general,

Assuming ELO,

Pr[ B |A ] =n[B ∩ A]

n[A].

Page 402: Finite Math Lecture Slides

Example: A city council has 5 Democratic members and 3Republican members. A committee consisting of 3 members isselected at random. If the selected committee has at least 1Democrat, then what is the probability that it has at least 1Republican also?

Solution: Let E be the event that the committee has at least 1Democrat, and let F be the event that it has at least 1Republican. Then we want

Pr[ F |E ] =n[F ∩ E ]

n[E ].

So there are two subproblems: n[E ] and n[F ∩ E ].

Page 403: Finite Math Lecture Slides

Example: A city council has 5 Democratic members and 3Republican members. A committee consisting of 3 members isselected at random. If the selected committee has at least 1Democrat, then what is the probability that it has at least 1Republican also?

Solution: Let E be the event that the committee has at least 1Democrat, and let F be the event that it has at least 1Republican. Then we want

Pr[ F |E ] =n[F ∩ E ]

n[E ].

So there are two subproblems: n[E ] and n[F ∩ E ].

Page 404: Finite Math Lecture Slides

Example: A city council has 5 Democratic members and 3Republican members. A committee consisting of 3 members isselected at random. If the selected committee has at least 1Democrat, then what is the probability that it has at least 1Republican also?

Solution: Let E be the event that the committee has at least 1Democrat, and let F be the event that it has at least 1Republican. Then we want

Pr[ F |E ] =n[F ∩ E ]

n[E ].

So there are two subproblems: n[E ] and n[F ∩ E ].

Page 405: Finite Math Lecture Slides

Example: A city council has 5 Democratic members and 3Republican members. A committee consisting of 3 members isselected at random. If the selected committee has at least 1Democrat, then what is the probability that it has at least 1Republican also?

n[E ]:

CP:

E = the committee has at least 1 DemocratE ′ = the committee has at no Democrats

= the committee has 3 Republicans

n[E ′] = C(3, 3) = 1n[S ] = C(8, 3) = 56n[E ] = 56− 1 = 55

Page 406: Finite Math Lecture Slides

Example: A city council has 5 Democratic members and 3Republican members. A committee consisting of 3 members isselected at random. If the selected committee has at least 1Democrat, then what is the probability that it has at least 1Republican also?

n[E ]:

CP:

E = the committee has at least 1 DemocratE ′ = the committee has at no Democrats

= the committee has 3 Republicans

n[E ′] = C(3, 3) = 1n[S ] = C(8, 3) = 56n[E ] = 56− 1 = 55

Page 407: Finite Math Lecture Slides

Example: A city council has 5 Democratic members and 3Republican members. A committee consisting of 3 members isselected at random. If the selected committee has at least 1Democrat, then what is the probability that it has at least 1Republican also?

n[F ∩ E ]:

5 Dem 3 Rep size

1 2C(5, 1) = 5 C(3, 2) = 3 15

2 1C(5, 2) = 10 C(3, 1) = 3 30

45

Page 408: Finite Math Lecture Slides

Example: A city council has 5 Democratic members and 3Republican members. A committee consisting of 3 members isselected at random. If the selected committee has at least 1Democrat, then what is the probability that it has at least 1Republican also?

Therefore

Pr[ F |E ] =n[F ∩ E ]

n[E ]=

45

55.

Page 409: Finite Math Lecture Slides

Example: A city council has 5 Democratic members and 3Republican members. The mayor appoints one randomly selectedmember to the Police Commission and another randomly selectedmember to the Zoning Commission.

1. What is the probability that the member appointed to theZoning Commission is a Democrat?

2. What is the probability that the member appointed to theZoning Commission is a Democrat given that the memberappointed to the Police Commission is a Democrat?

3. What is the probability that the member appointed to theZoning Commission is a Democrat given that the memberappointed to the Police Commission is a Republican?

Answers:

(1) 58 ; (2) 4

7 ; (3) 57 .

Page 410: Finite Math Lecture Slides

Example: A city council has 5 Democratic members and 3Republican members. The mayor appoints one randomly selectedmember to the Police Commission and another randomly selectedmember to the Zoning Commission.

1. What is the probability that the member appointed to theZoning Commission is a Democrat?

2. What is the probability that the member appointed to theZoning Commission is a Democrat given that the memberappointed to the Police Commission is a Democrat?

3. What is the probability that the member appointed to theZoning Commission is a Democrat given that the memberappointed to the Police Commission is a Republican?

Answers: (1) 58 ; (2) 4

7 ; (3) 57 .

Page 411: Finite Math Lecture Slides

Example: A city council has 5 Democratic members and 3Republican members. The mayor appoints one randomly selectedmember to the Police Commission and another randomly selectedmember to the Zoning Commission.What is the probability that the member appointed to the ZoningCommission is a Democrat?

Solution:

S = all possible appointees to two commissionsE = appointee to Zoning Com is a Dem

n[S ] = P(8, 2) = 56n[E ] = 5 · 7 = 35

Pr[E ] =35

56=

5

8

Page 412: Finite Math Lecture Slides

Example: A city council has 5 Democratic members and 3Republican members. The mayor appoints one randomly selectedmember to the Police Commission and another randomly selectedmember to the Zoning Commission.What is the probability that the member appointed to the ZoningCommission is a Democrat?

Solution:

S = all possible appointees to two commissionsE = appointee to Zoning Com is a Dem

n[S ] = P(8, 2) = 56n[E ] = 5 · 7 = 35

Pr[E ] =35

56=

5

8

Page 413: Finite Math Lecture Slides

Example: A city council has 5 Democratic members and 3Republican members. The mayor appoints one randomly selectedmember to the Police Commission and another randomly selectedmember to the Zoning Commission.What is the probability that the member appointed to the ZoningCommission is a Democrat?

Solution:

S = all possible appointees to two commissionsE = appointee to Zoning Com is a Dem

n[S ] = P(8, 2) = 56n[E ] = 5 · 7 = 35

Pr[E ] =35

56=

5

8

Page 414: Finite Math Lecture Slides

Example: A city council has 5 Democratic members and 3Republican members. The mayor appoints one randomly selectedmember to the Police Commission and another randomly selectedmember to the Zoning Commission.What is the probability that the member appointed to the ZoningCommission is a Democrat given that the member appointed tothe Police Commission is a Democrat?

Solution:

E = appointee to Zoning Com is a DemF = appointee to Police Com is a Dem

We want Pr[ E |F ] =n[E ∩ F ]

n[F ].

n[E ∩ F ] = 5 · 4 = 20n[F ] = 5 · 7 = 35

Pr[ E |F ] =20

35=

4

7

Page 415: Finite Math Lecture Slides

Example: A city council has 5 Democratic members and 3Republican members. The mayor appoints one randomly selectedmember to the Police Commission and another randomly selectedmember to the Zoning Commission.What is the probability that the member appointed to the ZoningCommission is a Democrat given that the member appointed tothe Police Commission is a Democrat?

Solution:

E = appointee to Zoning Com is a DemF = appointee to Police Com is a Dem

We want Pr[ E |F ] =n[E ∩ F ]

n[F ].

n[E ∩ F ] = 5 · 4 = 20n[F ] = 5 · 7 = 35

Pr[ E |F ] =20

35=

4

7

Page 416: Finite Math Lecture Slides

Example: A city council has 5 Democratic members and 3Republican members. The mayor appoints one randomly selectedmember to the Police Commission and another randomly selectedmember to the Zoning Commission.What is the probability that the member appointed to the ZoningCommission is a Democrat given that the member appointed tothe Police Commission is a Republican?

Solution:

E = appointee to Zoning Com is a DemF = appointee to Police Com is a Dem

We want Pr[ E |F ′ ] =n[E ∩ F ′]

n[F ′].

n[E ∩ F ′] = 5 · 3 = 15n[F ′] = 3 · 7 = 21

Pr[ E |F ′ ] =15

21=

5

7

Page 417: Finite Math Lecture Slides

Example: A city council has 5 Democratic members and 3Republican members. The mayor appoints one randomly selectedmember to the Police Commission and another randomly selectedmember to the Zoning Commission.What is the probability that the member appointed to the ZoningCommission is a Democrat given that the member appointed tothe Police Commission is a Republican?

Solution:

E = appointee to Zoning Com is a DemF = appointee to Police Com is a Dem

We want Pr[ E |F ′ ] =n[E ∩ F ′]

n[F ′].

n[E ∩ F ′] = 5 · 3 = 15n[F ′] = 3 · 7 = 21

Pr[ E |F ′ ] =15

21=

5

7

Page 418: Finite Math Lecture Slides

Independence

If the truth of an event has no influence on the probability ofanother event being true, we say those two events are independent.

Example:Suppose E is the event that you win the lottery today;

and F is the event that it rains today.

Pr[ E |F ] = Pr[E ]Pr[ F |E ] = Pr[F ]

The two equations are equivalent, and in fact they’re bothequivalent to

Pr[E ∩ F ] = Pr[E ]Pr[F ].

Page 419: Finite Math Lecture Slides

Independence

If the truth of an event has no influence on the probability ofanother event being true, we say those two events are independent.

Example:Suppose E is the event that you win the lottery today;

and F is the event that it rains today.

Pr[ E |F ] = Pr[E ]Pr[ F |E ] = Pr[F ]

The two equations are equivalent, and in fact they’re bothequivalent to

Pr[E ∩ F ] = Pr[E ]Pr[F ].

Page 420: Finite Math Lecture Slides

Pr[E ∩ F ] = Pr[E ] · Pr[F ]

n[E ∩ F ]

n[S ]=

n[E ]

n[S ]· Pr[F ]

n[E ∩ F ] = n[E ] · Pr[F ]

n[E ∩ F ]

n[E ]= Pr[F ]

Pr[ F |E ] = Pr[F ]

Page 421: Finite Math Lecture Slides

Example: A special die has the numbers 1–3 in yellow and 4–6 inred, otherwise the die is normal and fair. Let E be the event thenumber is even and R the event the number is red. Are E and Rindependent?

Solution: Since the die is fair we know

Pr[E ] = 1/2Pr[R] = 1/2.

In an earlier problem we calculated

Pr[ E |R ] = 2/3.

Since Pr[E ] 6= Pr[ E |R ], then E and R are not independent.

Page 422: Finite Math Lecture Slides

Example: A special die has the numbers 1–3 in yellow and 4–6 inred, otherwise the die is normal and fair. Let E be the event thenumber is even and R the event the number is red. Are E and Rindependent?

Solution: Since the die is fair we know

Pr[E ] = 1/2Pr[R] = 1/2.

In an earlier problem we calculated

Pr[ E |R ] = 2/3.

Since Pr[E ] 6= Pr[ E |R ], then E and R are not independent.

Page 423: Finite Math Lecture Slides

Example: A special die has the numbers 1–3 in yellow and 4–6 inred, otherwise the die is normal and fair. Let E be the event thenumber is even and R the event the number is red. Are E and Rindependent?

Solution: Since the die is fair we know

Pr[E ] = 1/2Pr[R] = 1/2.

In an earlier problem we calculated

Pr[ E |R ] = 2/3.

Since Pr[E ] 6= Pr[ E |R ], then E and R are not independent.

Page 424: Finite Math Lecture Slides

Example: A special die has the numbers 1–3 in yellow and 4–6 inred, otherwise the die is normal and fair. Let E be the event thenumber is even and R the event the number is red. Are E and Rindependent?

Solution: Since the die is fair we know

Pr[E ] = 1/2Pr[R] = 1/2.

In an earlier problem we calculated

Pr[ E |R ] = 2/3.

Since Pr[E ] 6= Pr[ E |R ], then E and R are not independent.

Page 425: Finite Math Lecture Slides

Example: A special die has the numbers 1–3 in yellow and 4–6 inred, otherwise the die is normal and fair. Let E be the event thenumber is even and R the event the number is red. Are E and Rindependent?

Alternate Solution: Since the die is fair we know

Pr[E ] = 1/2Pr[R] = 1/2.

Also

Pr[E ∩ R] =n[E ∩ R]

n[S ]=

2

6= 1/3.

But Pr[E ∩ R] 6= Pr[E ]Pr[R], so E and R are not independent.

Page 426: Finite Math Lecture Slides

Example: A special die has the numbers 1–3 in yellow and 4–6 inred, otherwise the die is normal and fair. Let E be the event thenumber is even and R the event the number is red. Are E and Rindependent?

Alternate Solution: Since the die is fair we know

Pr[E ] = 1/2Pr[R] = 1/2.

Also

Pr[E ∩ R] =n[E ∩ R]

n[S ]=

2

6= 1/3.

But Pr[E ∩ R] 6= Pr[E ]Pr[R], so E and R are not independent.

Page 427: Finite Math Lecture Slides

Example: A special die has the numbers 1–3 in yellow and 4–6 inred, otherwise the die is normal and fair. Let E be the event thenumber is even and R the event the number is red. Are E and Rindependent?

Alternate Solution: Since the die is fair we know

Pr[E ] = 1/2Pr[R] = 1/2.

Also

Pr[E ∩ R] =n[E ∩ R]

n[S ]=

2

6= 1/3.

But Pr[E ∩ R] 6= Pr[E ]Pr[R], so E and R are not independent.

Page 428: Finite Math Lecture Slides

Example: 120 tennis balls have been numbered 1–120. In addition,1/3 of the balls are green and 2/3 are white. How many greenballs with even numbers are there?

Solution: There’s not enough information given to answer thequestion.

Page 429: Finite Math Lecture Slides

Example: 120 tennis balls have been numbered 1–120. In addition,1/3 of the balls are green and 2/3 are white. How many greenballs with even numbers are there?

Solution: There’s not enough information given to answer thequestion.

Page 430: Finite Math Lecture Slides

Example: 120 tennis balls have been numbered 1–120. In addition,1/3 of the balls are green and 2/3 are white. Suppose the events

G = ball is greenE = ball is even

are known to be independent. How many green balls with evennumbers are there?

Solution: We know that

Pr[G ] = 1/3Pr[E ] = 1/2.

Since G and E are independent,

Pr[G ∩ E ] = Pr[G ] Pr[E ] = 1/3 · 1/2 = 1/6.

Therefore

n[G ∩ E ] = Pr[G ∩ E ] n[S ] = 1/6 · 120 = 20.

(Because Pr[A] =n[A]

n[S ].)

Page 431: Finite Math Lecture Slides

Example: 120 tennis balls have been numbered 1–120. In addition,1/3 of the balls are green and 2/3 are white. Suppose the events

G = ball is greenE = ball is even

are known to be independent. How many green balls with evennumbers are there?

Solution: We know that

Pr[G ] = 1/3Pr[E ] = 1/2.

Since G and E are independent,

Pr[G ∩ E ] = Pr[G ] Pr[E ] = 1/3 · 1/2 = 1/6.

Therefore

n[G ∩ E ] = Pr[G ∩ E ] n[S ] = 1/6 · 120 = 20.

(Because Pr[A] =n[A]

n[S ].)

Page 432: Finite Math Lecture Slides

Example: 120 tennis balls have been numbered 1–120. In addition,1/3 of the balls are green and 2/3 are white. Suppose the events

G = ball is greenE = ball is even

are known to be independent. How many green balls with evennumbers are there?

Solution: We know that

Pr[G ] = 1/3Pr[E ] = 1/2.

Since G and E are independent,

Pr[G ∩ E ] = Pr[G ] Pr[E ] = 1/3 · 1/2 = 1/6.

Therefore

n[G ∩ E ] = Pr[G ∩ E ] n[S ] = 1/6 · 120 = 20.

(Because Pr[A] =n[A]

n[S ].)

Page 433: Finite Math Lecture Slides

Example: 120 tennis balls have been numbered 1–120. In addition,1/3 of the balls are green and 2/3 are white. Suppose the events

G = ball is greenE = ball is even

are known to be independent. How many green balls with evennumbers are there?

Solution: We know that

Pr[G ] = 1/3Pr[E ] = 1/2.

Since G and E are independent,

Pr[G ∩ E ] = Pr[G ] Pr[E ] = 1/3 · 1/2 = 1/6.

Therefore

n[G ∩ E ] = Pr[G ∩ E ] n[S ] = 1/6 · 120 = 20.

(Because Pr[A] =n[A]

n[S ].)

Page 434: Finite Math Lecture Slides

Example: 120 tennis balls have been numbered 1–120. In addition,1/3 of the balls are green and 2/3 are white. Suppose the events

G = ball is greenE = ball is even

are known to be independent. How many green balls with evennumbers are there?

Solution: We know that

Pr[G ] = 1/3Pr[E ] = 1/2.

Since G and E are independent,

Pr[G ∩ E ] = Pr[G ] Pr[E ] = 1/3 · 1/2 = 1/6.

Therefore

n[G ∩ E ] = Pr[G ∩ E ] n[S ] = 1/6 · 120 = 20.

(Because Pr[A] =n[A]

n[S ].)

Page 435: Finite Math Lecture Slides

Example: Suppose you shuffle a deck of cards, then cut the deck toreveal one card. You do this 10 times. What is the probability youget exactly 3 face cards?

Solution:In a standard deck there are 12 face cards (F)

and 40 nonface cards (N).

There are 10 slots to fill, each with one card.

There are many cases to consider, for example:

F F F N N N N N N NF F N F N N N N N N...

Page 436: Finite Math Lecture Slides

Example: Suppose you shuffle a deck of cards, then cut the deck toreveal one card. You do this 10 times. What is the probability youget exactly 3 face cards?

Solution:In a standard deck there are 12 face cards (F)

and 40 nonface cards (N).

There are 10 slots to fill, each with one card.

There are many cases to consider, for example:

F F F N N N N N N NF F N F N N N N N N...

Page 437: Finite Math Lecture Slides

The number of cases is the number of anagrams of“FFFNNNNNNN”:

C(10, 3) C(7, 7) = C(10, 3).

The number of outcomes in each case is 123407,so the total number of outcomes is C(10, 3)123407.

The size of the sample space is

n[S ] = 5210.

Therefore the probability is

C(10, 3)123407

5210.

Page 438: Finite Math Lecture Slides

The number of cases is the number of anagrams of“FFFNNNNNNN”:

C(10, 3) C(7, 7) = C(10, 3).

The number of outcomes in each case is 123407,

so the total number of outcomes is C(10, 3)123407.

The size of the sample space is

n[S ] = 5210.

Therefore the probability is

C(10, 3)123407

5210.

Page 439: Finite Math Lecture Slides

The number of cases is the number of anagrams of“FFFNNNNNNN”:

C(10, 3) C(7, 7) = C(10, 3).

The number of outcomes in each case is 123407,so the total number of outcomes is C(10, 3)123407.

The size of the sample space is

n[S ] = 5210.

Therefore the probability is

C(10, 3)123407

5210.

Page 440: Finite Math Lecture Slides

The number of cases is the number of anagrams of“FFFNNNNNNN”:

C(10, 3) C(7, 7) = C(10, 3).

The number of outcomes in each case is 123407,so the total number of outcomes is C(10, 3)123407.

The size of the sample space is

n[S ] = 5210.

Therefore the probability is

C(10, 3)123407

5210.

Page 441: Finite Math Lecture Slides

The number of cases is the number of anagrams of“FFFNNNNNNN”:

C(10, 3) C(7, 7) = C(10, 3).

The number of outcomes in each case is 123407,so the total number of outcomes is C(10, 3)123407.

The size of the sample space is

n[S ] = 5210.

Therefore the probability is

C(10, 3)123407

5210.

Page 442: Finite Math Lecture Slides

C(10, 3)123 407

5210

This can be rewritten:

C(10, 3)123 407

523 527= C(10, 3)

123

523407

527

= C(10, 3)

(12

52

)3(40

52

)7

.

Note that

Pr[F ] =12

52

Pr[N] =40

52.

Page 443: Finite Math Lecture Slides

C(10, 3)123 407

5210

This can be rewritten:

C(10, 3)123 407

523 527= C(10, 3)

123

523407

527

= C(10, 3)

(12

52

)3(40

52

)7

.

Note that

Pr[F ] =12

52

Pr[N] =40

52.

Page 444: Finite Math Lecture Slides

Bernoulli Processes

Situations like that occur often.

A Bernoulli Process is a repeated process of selecting an objectwith replacement, in which we keep track of the number of “good”objects selected.

In the previous problem the good objects were face cards.

Page 445: Finite Math Lecture Slides

Bernoulli FormulaSuppose there are N objects to choose from, of which G areconsidered “good”. We select n objects with replacement.

N = # objectsG = # good objects

N − G = # bad objectsn = # selectionsg = # good selections

n − g = # bad selections

p =G

N= probability of getting a good object

1− p =N − G

N= probability of getting a bad object

Then the probability of exactly g good objects se-lected is

C(n, g)G g (N − G )(n−g)

Nn= C(n, g)pg (1− p)(n−g).

Page 446: Finite Math Lecture Slides

The probability of g good objects in n picks with replacement is

C(n, g)(prob of good) (# good picks)(prob of bad) (# bad picks).

So in the last problem, the probability of 3 face cards in 10 picks is

C(10, 3)

(12

52

)3(40

52

)7

.

Page 447: Finite Math Lecture Slides

The probability of g good objects in n picks with replacement is

C(n, g)(prob of good) (# good picks)(prob of bad) (# bad picks).

So in the last problem, the probability of 3 face cards in 10 picks is

C(10, 3)

(12

52

)3(40

52

)7

.

Page 448: Finite Math Lecture Slides

Example: A fair standard die is rolled 12 times. What’s theprobability of getting exactly 6 even numbers?

Solution:

n = # selections = 12g = # good selections = 6

n − g = # bad selections = 6p = probability of getting a good object = 1/2

1− p = probability of getting a bad object = 1/2

The probability of g = 6 even numbers and 6 odd numbers inn = 12 rolls is

C(n, g)(prob of good) (# good picks)(prob of bad) (# bad picks)

= C(12, 6)(1/2)6(1/2)

6

= 0.2256.

Page 449: Finite Math Lecture Slides

Example: A fair standard die is rolled 12 times. What’s theprobability of getting exactly 6 even numbers?

Solution:

n = # selections = 12g = # good selections = 6

n − g = # bad selections = 6p = probability of getting a good object = 1/2

1− p = probability of getting a bad object = 1/2

The probability of g = 6 even numbers and 6 odd numbers inn = 12 rolls is

C(n, g)(prob of good) (# good picks)(prob of bad) (# bad picks)

= C(12, 6)(1/2)6(1/2)

6

= 0.2256.

Page 450: Finite Math Lecture Slides

Example: A fair standard die is rolled 12 times. What’s theprobability of getting exactly 6 even numbers?

Solution:

n = # selections = 12g = # good selections = 6

n − g = # bad selections = 6p = probability of getting a good object = 1/2

1− p = probability of getting a bad object = 1/2

The probability of g = 6 even numbers and 6 odd numbers inn = 12 rolls is

C(n, g)(prob of good) (# good picks)(prob of bad) (# bad picks)

= C(12, 6)(1/2)6(1/2)

6

= 0.2256.

Page 451: Finite Math Lecture Slides

Example: A fair standard die is rolled 12 times. What’s theprobability of getting exactly 6 even numbers?

Solution:

n = # selections = 12g = # good selections = 6

n − g = # bad selections = 6p = probability of getting a good object = 1/2

1− p = probability of getting a bad object = 1/2

The probability of g = 6 even numbers and 6 odd numbers inn = 12 rolls is

C(n, g)(prob of good) (# good picks)(prob of bad) (# bad picks)

= C(12, 6)(1/2)6(1/2)

6

= 0.2256.

Page 452: Finite Math Lecture Slides

Example: A roulette wheel has 38 slots numbered 0, 00, and 1–36.Suppose a fair roulette wheel is spun 100 times. What’s theprobability of getting 17 exactly three times?

Solution:

n = # selections = 100g = # good selections = 3

n − g = # bad selections = 97p = probability of getting a good object = 1/38

1− p = probability of getting a bad object = 37/38

The probability of three 17’s is

C(n, g)(prob of good) (# good picks)(prob of bad) (# bad picks)

= C(100, 3)(1/38)3(37/38)

97

= .2218.

Page 453: Finite Math Lecture Slides

Example: A roulette wheel has 38 slots numbered 0, 00, and 1–36.Suppose a fair roulette wheel is spun 100 times. What’s theprobability of getting 17 exactly three times?

Solution:

n = # selections = 100g = # good selections = 3

n − g = # bad selections = 97p = probability of getting a good object = 1/38

1− p = probability of getting a bad object = 37/38

The probability of three 17’s is

C(n, g)(prob of good) (# good picks)(prob of bad) (# bad picks)

= C(100, 3)(1/38)3(37/38)

97

= .2218.

Page 454: Finite Math Lecture Slides

Example: A roulette wheel has 38 slots numbered 0, 00, and 1–36.Suppose a fair roulette wheel is spun 100 times. What’s theprobability of getting 17 exactly three times?

Solution:

n = # selections = 100g = # good selections = 3

n − g = # bad selections = 97p = probability of getting a good object = 1/38

1− p = probability of getting a bad object = 37/38

The probability of three 17’s is

C(n, g)(prob of good) (# good picks)(prob of bad) (# bad picks)

= C(100, 3)(1/38)3(37/38)

97

= .2218.

Page 455: Finite Math Lecture Slides

Example: A roulette wheel has 38 slots numbered 0, 00, and 1–36.Suppose a fair roulette wheel is spun 100 times. What’s theprobability of getting 17 exactly three times?

Solution:

n = # selections = 100g = # good selections = 3

n − g = # bad selections = 97p = probability of getting a good object = 1/38

1− p = probability of getting a bad object = 37/38

The probability of three 17’s is

C(n, g)(prob of good) (# good picks)(prob of bad) (# bad picks)

= C(100, 3)(1/38)3(37/38)

97

= .2218.

Page 456: Finite Math Lecture Slides

Example: Suppose a fair roulette wheel is spun 100 times. What’sthe probability of getting 17 at most three times?

Solution: “At most 3” means 0, 1, 2, or 3.

So repeat the previous method for the cases g = 0, 1, 2, 3 and add.

C(100, 0)(1/38)0(37/38)

100

+ C(100, 1)(1/38)1(37/38)

99

+ C(100, 2)(1/38)2(37/38)

98

+ C(100, 3)(1/38)3(37/38)

97

= .7302

Page 457: Finite Math Lecture Slides

Example: Suppose a fair roulette wheel is spun 100 times. What’sthe probability of getting 17 at most three times?

Solution: “At most 3” means 0, 1, 2, or 3.

So repeat the previous method for the cases g = 0, 1, 2, 3 and add.

C(100, 0)(1/38)0(37/38)

100

+ C(100, 1)(1/38)1(37/38)

99

+ C(100, 2)(1/38)2(37/38)

98

+ C(100, 3)(1/38)3(37/38)

97

= .7302

Page 458: Finite Math Lecture Slides

Example: Suppose a fair roulette wheel is spun 100 times. What’sthe probability of getting 17 at most three times?

Solution: “At most 3” means 0, 1, 2, or 3.

So repeat the previous method for the cases g = 0, 1, 2, 3 and add.

C(100, 0)(1/38)0(37/38)

100

+ C(100, 1)(1/38)1(37/38)

99

+ C(100, 2)(1/38)2(37/38)

98

+ C(100, 3)(1/38)3(37/38)

97

= .7302

Page 459: Finite Math Lecture Slides

NonELO Probability

If a sample space does not have ELO, then we have to make a fewadjustments.

The Bernoulli formula can still be used.

But any formula that uses n[ ] to measure the size of an outcomemust use Pr[ ] instead.

ELO:

Pr[ E |F ] =n[E ∩ F ]

n[F ]

NonELO:

Pr[ E |F ] =Pr[E ∩ F ]

Pr[F ]

The nonELO version can be used even in ELO problems,but not vice-versa.

Page 460: Finite Math Lecture Slides

NonELO Probability

If a sample space does not have ELO, then we have to make a fewadjustments.

The Bernoulli formula can still be used.

But any formula that uses n[ ] to measure the size of an outcomemust use Pr[ ] instead.

ELO:

Pr[ E |F ] =n[E ∩ F ]

n[F ]

NonELO:

Pr[ E |F ] =Pr[E ∩ F ]

Pr[F ]

The nonELO version can be used even in ELO problems,but not vice-versa.

Page 461: Finite Math Lecture Slides

NonELO Probability

If a sample space does not have ELO, then we have to make a fewadjustments.

The Bernoulli formula can still be used.

But any formula that uses n[ ] to measure the size of an outcomemust use Pr[ ] instead.

ELO:

Pr[ E |F ] =n[E ∩ F ]

n[F ]

NonELO:

Pr[ E |F ] =Pr[E ∩ F ]

Pr[F ]

The nonELO version can be used even in ELO problems,but not vice-versa.

Page 462: Finite Math Lecture Slides

NonELO Probability

If a sample space does not have ELO, then we have to make a fewadjustments.

The Bernoulli formula can still be used.

But any formula that uses n[ ] to measure the size of an outcomemust use Pr[ ] instead.

ELO:

Pr[ E |F ] =n[E ∩ F ]

n[F ]

NonELO:

Pr[ E |F ] =Pr[E ∩ F ]

Pr[F ]

The nonELO version can be used even in ELO problems,but not vice-versa.

Page 463: Finite Math Lecture Slides

The MP for counting outcomes can be translatedusing the conditional probability formula.

Pr[E ∩ F ]

Pr[F ]= Pr[ E |F ]

Pr[E ∩ F ] = Pr[F ] Pr[ E |F ]

Pr[E and F both happen] = Pr[F happens first]·Pr[then E happens]

Page 464: Finite Math Lecture Slides

The MP for counting outcomes can be translatedusing the conditional probability formula.

Pr[E ∩ F ]

Pr[F ]= Pr[ E |F ]

Pr[E ∩ F ] = Pr[F ] Pr[ E |F ]

Pr[E and F both happen] = Pr[F happens first]·Pr[then E happens]

Page 465: Finite Math Lecture Slides

The MP for counting outcomes can be translatedusing the conditional probability formula.

Pr[E ∩ F ]

Pr[F ]= Pr[ E |F ]

Pr[E ∩ F ] = Pr[F ] Pr[ E |F ]

Pr[E and F both happen] = Pr[F happens first]·Pr[then E happens]

Page 466: Finite Math Lecture Slides

Example: A standard deck of cards is shuffled and then 2 cards aredrawn without replacement. What’s the probability that both cardsare Aces?

Solution: Let

E = first card is an AceF = second card is an Ace.

Then

Pr[E ] =4

52

Pr[ F |E ] =3

51

Pr[F ] =4

52

Therefore

Pr[E ∩ F ] = Pr[E ] Pr[ F |E ] =4

52

3

51= .004525.

Page 467: Finite Math Lecture Slides

Example: A standard deck of cards is shuffled and then 2 cards aredrawn without replacement. What’s the probability that both cardsare Aces?

Solution: Let

E = first card is an AceF = second card is an Ace.

Then

Pr[E ] =4

52

Pr[ F |E ] =3

51

Pr[F ] =4

52

Therefore

Pr[E ∩ F ] = Pr[E ] Pr[ F |E ] =4

52

3

51= .004525.

Page 468: Finite Math Lecture Slides

Example: A standard deck of cards is shuffled and then 2 cards aredrawn without replacement. What’s the probability that both cardsare Aces?

Solution: Let

E = first card is an AceF = second card is an Ace.

Then

Pr[E ] =4

52

Pr[ F |E ] =3

51

Pr[F ] =4

52

Therefore

Pr[E ∩ F ] = Pr[E ] Pr[ F |E ] =4

52

3

51= .004525.

Page 469: Finite Math Lecture Slides

Example: A standard deck of cards is shuffled and then 2 cards aredrawn without replacement. What’s the probability that both cardsare Aces?

Solution: Let

E = first card is an AceF = second card is an Ace.

Then

Pr[E ] =4

52

Pr[ F |E ] =3

51

Pr[F ] =4

52

Therefore

Pr[E ∩ F ] = Pr[E ] Pr[ F |E ] =4

52

3

51= .004525.

Page 470: Finite Math Lecture Slides

Venn diagrams can still be used,but sizes must be indicated by probability rather than # elements.

Example: Suppose that Pr[A] = .6, Pr[B] = .3, andPr[A ∪ B] = .8. Find Pr[ B |A ].

Venn Diagram:

Page 471: Finite Math Lecture Slides

Venn diagrams can still be used,but sizes must be indicated by probability rather than # elements.

Example: Suppose that Pr[A] = .6, Pr[B] = .3, andPr[A ∪ B] = .8. Find Pr[ B |A ].

Venn Diagram:

Page 472: Finite Math Lecture Slides

Venn diagrams can still be used,but sizes must be indicated by probability rather than # elements.

Example: Suppose that Pr[A] = .6, Pr[B] = .3, andPr[A ∪ B] = .8. Find Pr[ B |A ].

Venn Diagram:

Page 473: Finite Math Lecture Slides

Then

Pr[ B |A ] =Pr[B ∩ A]

Pr[A]

=.1

.6= 1/6.

Page 474: Finite Math Lecture Slides

Then

Pr[ B |A ] =Pr[B ∩ A]

Pr[A]

=.1

.6= 1/6.

Page 475: Finite Math Lecture Slides

Then

Pr[ B |A ] =Pr[B ∩ A]

Pr[A]

=.1

.6= 1/6.

Page 476: Finite Math Lecture Slides

Tree Diagrams: The Last ResortIf nothing else works you can try a tree diagram.

Tree diagrams are often useful for multistage processes in whichlater stages depend on earlier ones.

Example: Suppose you have 5 candies in your pocket, 2 of whichare butterscotch and the other 3 cherry. You grab one candy atrandom and eat it. Of course you are only aware of the flavor ofthe candy.

Tree Diagram: There are two possible outcomes.

Page 477: Finite Math Lecture Slides

Tree Diagrams: The Last ResortIf nothing else works you can try a tree diagram.

Tree diagrams are often useful for multistage processes in whichlater stages depend on earlier ones.

Example: Suppose you have 5 candies in your pocket, 2 of whichare butterscotch and the other 3 cherry. You grab one candy atrandom and eat it. Of course you are only aware of the flavor ofthe candy.

Tree Diagram: There are two possible outcomes.

Page 478: Finite Math Lecture Slides

Tree Diagrams: The Last ResortIf nothing else works you can try a tree diagram.

Tree diagrams are often useful for multistage processes in whichlater stages depend on earlier ones.

Example: Suppose you have 5 candies in your pocket, 2 of whichare butterscotch and the other 3 cherry. You grab one candy atrandom and eat it. Of course you are only aware of the flavor ofthe candy.

Tree Diagram: There are two possible outcomes.

Page 479: Finite Math Lecture Slides

“Begin”, “B”, and “C” are called nodes,the connecting lines are called edges.

Page 480: Finite Math Lecture Slides

We label the edges with probabilities.

Page 481: Finite Math Lecture Slides

For clarity, I’ll indicate under each nodethe number of candies of each flavor available.

Page 482: Finite Math Lecture Slides

Now suppose you grab and eat another candy.Now the possibilities look like this.

Page 483: Finite Math Lecture Slides

Notice that the probabilities at the first stageare ordinary probabilities.

If B1 is the event the first candy was butterscotch, thenPr[B1] = 2/5.

Page 484: Finite Math Lecture Slides

But the probabilities at the second stageare conditional probabilities.

If C2 is the event the second candy was cherry, thenPr[ C2 |B1 ] = 3/4.

Page 485: Finite Math Lecture Slides

Everywhere, the probabilities from a single node sum to 1.

Page 486: Finite Math Lecture Slides

If we keep track of the flavor of each candy as we eat it,there are 4 possible outcomes.

Page 487: Finite Math Lecture Slides

The probability of each outcome can be computed using the MP:Pr[E ]Pr[ F |E ] = Pr[E ∩ F ].

Page 488: Finite Math Lecture Slides

The probability of each outcome can be computed using the MP:Pr[E ]Pr[ F |E ] = Pr[E ∩ F ].

Page 489: Finite Math Lecture Slides

The sum of those four probabilities should be 1.

Page 490: Finite Math Lecture Slides

Now we can answer questions.Example: What’s the probability of eating two cherry candies?

Solution:

Answer: 6/20

Page 491: Finite Math Lecture Slides

Example: What’s the probability of eating one of each flavor?

Solution:

Answer: 6/20 + 6/20 = 12/20

Page 492: Finite Math Lecture Slides

Example: What’s the conditional probability that the second ischerry given that the first was butterscotch?

Solution:

Answer:3

4

Page 493: Finite Math Lecture Slides

Example: What’s the conditional probability that the second ischerry given that the first was butterscotch?

Solution:

Answer:3

4

Page 494: Finite Math Lecture Slides

Example: What’s the conditional probability that the first wascherry given that the second was butterscotch?

Solution:

Pr[ 1C | 2B ] =Pr[1C ∩ 2B]

Pr[2B]=

6/202/20 + 6/20

=6

8=

3

4

Page 495: Finite Math Lecture Slides

Example: What’s the conditional probability that the first wascherry given that the second was butterscotch?

Solution:

Pr[ 1C | 2B ] =Pr[1C ∩ 2B]

Pr[2B]

=6/20

2/20 + 6/20=

6

8=

3

4

Page 496: Finite Math Lecture Slides

Example: What’s the conditional probability that the first wascherry given that the second was butterscotch?

Solution:

Pr[ 1C | 2B ] =Pr[1C ∩ 2B]

Pr[2B]=

6/202/20 + 6/20

=6

8=

3

4

Page 497: Finite Math Lecture Slides

Example: What’s the conditional probability that the first wascherry given that the second was butterscotch?

Solution:

Pr[ 1C | 2B ] =Pr[1C ∩ 2B]

Pr[2B]=

6/202/20 + 6/20

=6

8=

3

4

Page 498: Finite Math Lecture Slides

Random VariablesA random variable is a variable whose value depends onthe outcome of a random process.

Example: Let X be the value that comes up when a die is rolled.

Whether a die is physically rolled ora computer generates a random number,the only important thing is the probability distribution (p.d.f.),the table of values of X and their probabilities.

X Pr[X ]

1 1/62 1/63 1/64 1/65 1/66 1/6

Page 499: Finite Math Lecture Slides

Random VariablesA random variable is a variable whose value depends onthe outcome of a random process.

Example: Let X be the value that comes up when a die is rolled.

Whether a die is physically rolled ora computer generates a random number,the only important thing is the probability distribution (p.d.f.),the table of values of X and their probabilities.

X Pr[X ]

1 1/62 1/63 1/64 1/65 1/66 1/6

Page 500: Finite Math Lecture Slides

Example: Suppose there are 3 red marbles and 2 green marbles ina bowl. Two marbles are taken simultaneously at random from thebowl. Let X be the number of green marbles taken. Write out thep.d.f. of X .

X Pr[X ]

0 ?1 ?2 ?

Each individual probability like Pr[X = 1]can be computed using the C function.

Pr[X = 1] =C(3, 1) C(2, 1)

C(5, 2)=

6

10=

3

5

Page 501: Finite Math Lecture Slides

Example: Suppose there are 3 red marbles and 2 green marbles ina bowl. Two marbles are taken simultaneously at random from thebowl. Let X be the number of green marbles taken. Write out thep.d.f. of X .

X Pr[X ]

0 ?1 ?2 ?

Each individual probability like Pr[X = 1]can be computed using the C function.

Pr[X = 1] =C(3, 1) C(2, 1)

C(5, 2)=

6

10=

3

5

Page 502: Finite Math Lecture Slides

Example: Suppose there are 3 red marbles and 2 green marbles ina bowl. Two marbles are taken simultaneously at random from thebowl. Let X be the number of green marbles taken. Write out thep.d.f. of X .

X Pr[X ]

0 ?1 ?2 ?

Each individual probability like Pr[X = 1]can be computed using the C function.

Pr[X = 1] =C(3, 1) C(2, 1)

C(5, 2)=

6

10=

3

5

Page 503: Finite Math Lecture Slides

Example: Suppose there are 3 red marbles and 2 green marbles ina bowl. Two marbles are taken simultaneously at random from thebowl. Let X be the number of green marbles taken. Write out thep.d.f. of X .

X Pr[X ]

0 ?1 ?2 ?

Each individual probability like Pr[X = 1]can be computed using the C function.

Pr[X = 1] =C(3, 1) C(2, 1)

C(5, 2)=

6

10=

3

5

Page 504: Finite Math Lecture Slides

Example: Suppose there are 3 red marbles and 2 green marbles ina bowl. Two marbles are taken simultaneously at random from thebowl. Let X be the number of green marbles taken. Write out thep.d.f. of X .

X Pr[X ]

0 C(3,2)C(5,2)

3/10

1 C(3,1)C(2,1)C(5,2)

6/10

2 C(2,2)C(5,2)

1/10

1

Note the sum of the probabilities must be 1.

Page 505: Finite Math Lecture Slides

Example: Suppose there are 3 red marbles and 2 green marbles ina bowl. Two marbles are taken simultaneously at random from thebowl. Let X be the number of green marbles taken. Write out thep.d.f. of X .

X Pr[X ]

0 C(3,2)C(5,2)

3/10

1 C(3,1)C(2,1)C(5,2)

6/10

2 C(2,2)C(5,2)

1/10

1

Note the sum of the probabilities must be 1.

Page 506: Finite Math Lecture Slides

Let’s play a game.

We both put $1 in the pot.Then we roll a fair die.If the die comes up 6, you win the pot.If anything else comes up, I win.

Is this game fair? No.Exactly how unfair is it?

Page 507: Finite Math Lecture Slides

Let’s play a game.

We both put $1 in the pot.Then we roll a fair die.If the die comes up 6, you win the pot.If anything else comes up, I win.

Is this game fair?

No.Exactly how unfair is it?

Page 508: Finite Math Lecture Slides

Let’s play a game.

We both put $1 in the pot.Then we roll a fair die.If the die comes up 6, you win the pot.If anything else comes up, I win.

Is this game fair? No.Exactly how unfair is it?

Page 509: Finite Math Lecture Slides

We both put $1 in the pot.Then we roll a fair die.If the die comes up 6, you win the pot.If anything else comes up, I win.

Let X be my net winnings.

Then X can take only the values ±1.

Here’s the p.d.f.

X Pr[X ]

-1 1/61 5/6

Page 510: Finite Math Lecture Slides

We both put $1 in the pot.Then we roll a fair die.If the die comes up 6, you win the pot.If anything else comes up, I win.

Let X be my net winnings.

Then X can take only the values ±1.

Here’s the p.d.f.

X Pr[X ]

-1 1/61 5/6

Page 511: Finite Math Lecture Slides

X Pr[X ]

-1 1/61 5/6

The expected value of X , denoted E(X ), is the weighted averageof the X values:

E(X ) = −1 · (1/6) + 1 · (5/6) = 4/6 = 2/3.

This means that on average I win 66.6¢ every time we play.

If we play 100 times, I should win about $67.

Page 512: Finite Math Lecture Slides

X Pr[X ]

-1 1/61 5/6

The expected value of X , denoted E(X ), is the weighted averageof the X values:

E(X ) = −1 · (1/6) + 1 · (5/6) = 4/6 = 2/3.

This means that on average I win 66.6¢ every time we play.

If we play 100 times, I should win about $67.

Page 513: Finite Math Lecture Slides

X Pr[X ]

-1 1/61 5/6

The expected value of X , denoted E(X ), is the weighted averageof the X values:

E(X ) = −1 · (1/6) + 1 · (5/6) = 4/6 = 2/3.

This means that on average I win 66.6¢ every time we play.

If we play 100 times, I should win about $67.

Page 514: Finite Math Lecture Slides

The easiest way to compute E(X ) is to add an extra column to thep.d.f.

X Pr[X ] X · Pr[X ]

-1 1/6 −1/61 5/6

5/6

4/6 = E(X )

Then E(X ) is the sum of the entries in the X · Pr[X ] column.

Page 515: Finite Math Lecture Slides

Example: Suppose there are 3 red marbles and 2 green marbles ina bowl. Two marbles are taken simultaneously at random from thebowl. Let X be the number of green marbles taken. Find theexpected value of X .

Solution: Recall the p.d.f.

X Pr[X ] X · Pr[X ]

0 3/10 0

1 6/106/10

2 1/102/10

8/10

Add the extra column.

So E(X ) = 8/10 = 4/5.

Page 516: Finite Math Lecture Slides

Example: Suppose there are 3 red marbles and 2 green marbles ina bowl. Two marbles are taken simultaneously at random from thebowl. Let X be the number of green marbles taken. Find theexpected value of X .

Solution: Recall the p.d.f.

X Pr[X ]

0 3/10

1 6/10

2 1/10

Add the extra column.

So E(X ) = 8/10 = 4/5.

Page 517: Finite Math Lecture Slides

Example: Suppose there are 3 red marbles and 2 green marbles ina bowl. Two marbles are taken simultaneously at random from thebowl. Let X be the number of green marbles taken. Find theexpected value of X .

Solution: Recall the p.d.f.

X Pr[X ] X · Pr[X ]

0 3/10 0

1 6/106/10

2 1/102/10

8/10

Add the extra column.

So E(X ) = 8/10 = 4/5.

Page 518: Finite Math Lecture Slides

Example: Suppose there are 3 red marbles and 2 green marbles ina bowl. Two marbles are taken simultaneously at random from thebowl. Let X be the number of green marbles taken. Find theexpected value of X .

Solution: Recall the p.d.f.

X Pr[X ] X · Pr[X ]

0 3/10 0

1 6/106/10

2 1/102/10

8/10

Add the extra column.

So E(X ) = 8/10 = 4/5.

Page 519: Finite Math Lecture Slides

Example: Suppose there are 3 red marbles and 2 green marbles ina bowl. Two marbles are taken simultaneously at random from thebowl. Let X be the number of green marbles taken. Find theexpected value of X .

E(X ) = 4/5

Shouldn’t that have been “obvious”?

Of all the marbles, 2/5 are green.So 2/5 of the 2 marbles we take should be green.

E(X ) = 2/5 · 2

Page 520: Finite Math Lecture Slides

Example: Suppose there are 3 red marbles and 2 green marbles ina bowl. Two marbles are taken simultaneously at random from thebowl. Let X be the number of green marbles taken. Find theexpected value of X .

E(X ) = 4/5

Shouldn’t that have been “obvious”?

Of all the marbles, 2/5 are green.So 2/5 of the 2 marbles we take should be green.

E(X ) = 2/5 · 2

Page 521: Finite Math Lecture Slides

This shortcut works for sampling problems,i.e. when objects are chosen from the universe with ELO,with or without replacement.

In general,

If n samples are taken from a universewhere the fraction of “good” objects isp, and X is the number of good objectsin the sample, then

E(X ) = p · n.

Page 522: Finite Math Lecture Slides

This shortcut works for sampling problems,i.e. when objects are chosen from the universe with ELO,with or without replacement.

In general,

If n samples are taken from a universewhere the fraction of “good” objects isp, and X is the number of good objectsin the sample, then

E(X ) = p · n.

Page 523: Finite Math Lecture Slides

Example: You sit at the roulette wheel for 36 hours, betting 1000times on your lucky number, 17. What is the expected number oftimes that you win?

Solution:

E(X ) =1

38· 1000 = 26.32

Page 524: Finite Math Lecture Slides

Example: You sit at the roulette wheel for 36 hours, betting 1000times on your lucky number, 17. What is the expected number oftimes that you win?

Solution:

E(X ) =1

38· 1000 = 26.32

Page 525: Finite Math Lecture Slides

Linear Algebra

A linear equation is one in whichvariables are used in the simplest possible way:multiplied by a constant and added to the other terms.

ExamplesLinear:

2x + 3y = 4

.1a−√

5b = 137c

Nonlinear:x

y= 1

xy = 52√

x − 3y2 = z

Page 526: Finite Math Lecture Slides

Linear Algebra

A linear equation is one in whichvariables are used in the simplest possible way:multiplied by a constant and added to the other terms.

ExamplesLinear:

2x + 3y = 4

.1a−√

5b = 137c

Nonlinear:x

y= 1

xy = 52√

x − 3y2 = z

Page 527: Finite Math Lecture Slides

Linear Algebra

A linear equation is one in whichvariables are used in the simplest possible way:multiplied by a constant and added to the other terms.

ExamplesLinear:

2x + 3y = 4

.1a−√

5b = 137c

Nonlinear:x

y= 1

xy = 52√

x − 3y2 = z

Page 528: Finite Math Lecture Slides

Linear equations get their name from their graphs.

Here is the graph of 3x − 2y = 6.

-6

-4

-2

0

2

4

6

-6 -4 -2 0 2 4 6

x

y

Each point on the blue line has coordinates that satisfy 3x −2y = 6.

Page 529: Finite Math Lecture Slides

Linear equations get their name from their graphs.

Here is the graph of 3x − 2y = 6.

-6

-4

-2

0

2

4

6

-6 -4 -2 0 2 4 6

x

y

Each point on the blue line has coordinates that satisfy 3x −2y = 6.

Page 530: Finite Math Lecture Slides

The points where the graph crosses the axes are called intercepts.

-6

-4

-2

0

2

4

6

-6 -4 -2 0 2 4 6

x

y

For the line 3x − 2y = 6the x-intercept is (2, 0) andthe y -intercept is (0,−3).

Page 531: Finite Math Lecture Slides

The points where the graph crosses the axes are called intercepts.

-6

-4

-2

0

2

4

6

-6 -4 -2 0 2 4 6

x

y

For the line 3x − 2y = 6the x-intercept is (2, 0) andthe y -intercept is (0,−3).

Page 532: Finite Math Lecture Slides

To find the intercepts for a linear equation:

x-intercept This is where the line crosses the x-axis, so y = 0.Plug in y = 0 and solve for x .

y -intercept This is where the line crosses the y -axis, so x = 0.Plug in x = 0 and solve for y .

Example: For 3x − 2y = 6,

x : 3x − 2(0) = 6 =⇒ x = 2y : 3(0)− 2y = 6 =⇒ y = −3.

Page 533: Finite Math Lecture Slides

To find the intercepts for a linear equation:

x-intercept This is where the line crosses the x-axis, so y = 0.Plug in y = 0 and solve for x .

y -intercept This is where the line crosses the y -axis, so x = 0.Plug in x = 0 and solve for y .

Example: For 3x − 2y = 6,

x : 3x − 2(0) = 6 =⇒ x = 2y : 3(0)− 2y = 6 =⇒ y = −3.

Page 534: Finite Math Lecture Slides

If you’re asked to graph a linear equation,all you need are two distinct points satisfying the equation(for example, the intercepts).Just plot them and draw a line through them.

Example: Graph the linear equation x + 3y = 4.

Solution: Here it’s easy to find one solution: (1, 1).You can find others without much trouble, e.g. (−2, 2).Or you can find the intercepts:

x : x + 3(0) = 4 =⇒ x = 4y : 0 + 3y = 4 =⇒ y = 4/3.

That gives us (4, 0) and (0, 4/3).

Page 535: Finite Math Lecture Slides

If you’re asked to graph a linear equation,all you need are two distinct points satisfying the equation(for example, the intercepts).Just plot them and draw a line through them.

Example: Graph the linear equation x + 3y = 4.

Solution: Here it’s easy to find one solution: (1, 1).You can find others without much trouble, e.g. (−2, 2).Or you can find the intercepts:

x : x + 3(0) = 4 =⇒ x = 4y : 0 + 3y = 4 =⇒ y = 4/3.

That gives us (4, 0) and (0, 4/3).

Page 536: Finite Math Lecture Slides

It doesn’t matter which two points you use since they all lie on thesame line.

-6

-4

-2

0

2

4

6

-6 -4 -2 0 2 4 6

But you might want to plot extra points as a check.

Page 537: Finite Math Lecture Slides

It doesn’t matter which two points you use since they all lie on thesame line.

-6

-4

-2

0

2

4

6

-6 -4 -2 0 2 4 6

But you might want to plot extra points as a check.

Page 538: Finite Math Lecture Slides

Slope

Slope is a measure of the steepness of a line.

If the line rises from left to right, slope is +.

If the line falls from left to right, slope is −.

The formula for slope is

slope =rise

run=

vertical change

horizontal change.

Page 539: Finite Math Lecture Slides

Slope

Slope is a measure of the steepness of a line.

If the line rises from left to right, slope is +.

If the line falls from left to right, slope is −.

The formula for slope is

slope =rise

run=

vertical change

horizontal change.

Page 540: Finite Math Lecture Slides

Slope

Slope is a measure of the steepness of a line.

If the line rises from left to right, slope is +.

If the line falls from left to right, slope is −.

The formula for slope is

slope =rise

run=

vertical change

horizontal change.

Page 541: Finite Math Lecture Slides

Consider the line we just graphed.

-6

-4

-2

0

2

4

6

-6 -4 -2 0 2 4 6

As the line travels from (−2, 2)to (4, 0):the vertical change is

0− 2 = −2

and the horizontal change is

4− (−2) = 6.

So the slope of the line is

rise

run=−2

6= −1

3.

We could have used any two pointsbecause they all give the same result.

Page 542: Finite Math Lecture Slides

Consider the line we just graphed.

-6

-4

-2

0

2

4

6

-6 -4 -2 0 2 4 6

As the line travels from (−2, 2)to (4, 0):the vertical change is

0− 2 = −2

and the horizontal change is

4− (−2) = 6.

So the slope of the line is

rise

run=−2

6= −1

3.

We could have used any two pointsbecause they all give the same result.

Page 543: Finite Math Lecture Slides

Consider the line we just graphed.

-6

-4

-2

0

2

4

6

-6 -4 -2 0 2 4 6

As the line travels from (−2, 2)to (4, 0):the vertical change is

0− 2 = −2

and the horizontal change is

4− (−2) = 6.

So the slope of the line is

rise

run=−2

6= −1

3.

We could have used any two pointsbecause they all give the same result.

Page 544: Finite Math Lecture Slides

Jack and Jill have a date on a hill.

Jack is climbing up the line from (1,5) to Jill’s picnic site at (4,14).

Jack’s path;

1. Straight line; use Pythagorean Theorem to find distance;2. Horizontal and vertical components.

2.1 Horizontal: 4− 1 = 32.2 Vertical: 14− 5 = 9

Page 545: Finite Math Lecture Slides

Jack and Jill have a date on a hill.

Jack is climbing up the line from (1,5) to Jill’s picnic site at (4,14).

Jack’s path;

1. Straight line; use Pythagorean Theorem to find distance;

2. Horizontal and vertical components.

2.1 Horizontal: 4− 1 = 32.2 Vertical: 14− 5 = 9

Page 546: Finite Math Lecture Slides

Jack and Jill have a date on a hill.

Jack is climbing up the line from (1,5) to Jill’s picnic site at (4,14).

Jack’s path;

1. Straight line; use Pythagorean Theorem to find distance;2. Horizontal and vertical components.

2.1 Horizontal: 4− 1 = 32.2 Vertical: 14− 5 = 9

Page 547: Finite Math Lecture Slides

Jack and Jill have a date on a hill.

Jack is climbing up the line from (1,5) to Jill’s picnic site at (4,14).

Jack’s path;

1. Straight line; use Pythagorean Theorem to find distance;2. Horizontal and vertical components.

2.1 Horizontal: 4− 1 = 3

2.2 Vertical: 14− 5 = 9

Page 548: Finite Math Lecture Slides

Jack and Jill have a date on a hill.

Jack is climbing up the line from (1,5) to Jill’s picnic site at (4,14).

Jack’s path;

1. Straight line; use Pythagorean Theorem to find distance;2. Horizontal and vertical components.

2.1 Horizontal: 4− 1 = 32.2 Vertical: 14− 5 = 9

Page 549: Finite Math Lecture Slides

In general, if a line passes through the points

(x1, y1) and (x2, y2)

then its slope is

Slope = m =y2 − y1x2 − x1

=y1 − y2x1 − x2

.

Page 550: Finite Math Lecture Slides

In general, if a line passes through the points

(x1, y1) and (x2, y2)

then its slope is

Slope = m =y2 − y1x2 − x1

=y1 − y2x1 − x2

.

Page 551: Finite Math Lecture Slides

In general, if a line passes through the points

(x1, y1) and (x2, y2)

then its slope is

Slope = m =y2 − y1x2 − x1

=y1 − y2x1 − x2

.

Page 552: Finite Math Lecture Slides

You may also remember this rule from high school:

The slope of the line Ax + By = C is −B/A.

Example: Find the slope of the line 2x − 3y = 5.

Solution: We have A = 2, B = −3, and C = 5, so

m = −(−3/2) = 3/2.

Page 553: Finite Math Lecture Slides

You may also remember this rule from high school:

The slope of the line Ax + By = C is −B/A.

Example: Find the slope of the line 2x − 3y = 5.

Solution: We have A = 2, B = −3, and C = 5, so

m = −(−3/2) = 3/2.

Page 554: Finite Math Lecture Slides

You may also remember this rule from high school:

The slope of the line Ax + By = C is −B/A.

Example: Find the slope of the line 2x − 3y = 5.

Solution: We have A = 2, B = −3, and C = 5, so

m = −(−3/2) = 3/2.

Page 555: Finite Math Lecture Slides

What’s the formula for the line with the following properties?

1. The vertical intercept is 2, i.e. it passes through (0, 2);

2. The slope is 3.

Fact 2 means the height grows like 3x :one unit of x produces three units of y .So

y = 3x + ?.

To find the value of ?, use Fact 1:

2 = 3 · 0 + ?.

So ? has a value of 2.

y = 3x + 2

In general, the equation of a line can be written

y = mx + b (Slope-Intercept Form)

where m is the slope and b is from the vertical intercept.

Page 556: Finite Math Lecture Slides

What’s the formula for the line with the following properties?

1. The vertical intercept is 2, i.e. it passes through (0, 2);

2. The slope is 3.

Fact 2 means the height grows like 3x :one unit of x produces three units of y .So

y = 3x + ?.

To find the value of ?, use Fact 1:

2 = 3 · 0 + ?.

So ? has a value of 2.

y = 3x + 2

In general, the equation of a line can be written

y = mx + b (Slope-Intercept Form)

where m is the slope and b is from the vertical intercept.

Page 557: Finite Math Lecture Slides

What’s the formula for the line with the following properties?

1. The vertical intercept is 2, i.e. it passes through (0, 2);

2. The slope is 3.

Fact 2 means the height grows like 3x :one unit of x produces three units of y .

Soy = 3x + ?.

To find the value of ?, use Fact 1:

2 = 3 · 0 + ?.

So ? has a value of 2.

y = 3x + 2

In general, the equation of a line can be written

y = mx + b (Slope-Intercept Form)

where m is the slope and b is from the vertical intercept.

Page 558: Finite Math Lecture Slides

What’s the formula for the line with the following properties?

1. The vertical intercept is 2, i.e. it passes through (0, 2);

2. The slope is 3.

Fact 2 means the height grows like 3x :one unit of x produces three units of y .So

y = 3x + ?.

To find the value of ?, use Fact 1:

2 = 3 · 0 + ?.

So ? has a value of 2.

y = 3x + 2

In general, the equation of a line can be written

y = mx + b (Slope-Intercept Form)

where m is the slope and b is from the vertical intercept.

Page 559: Finite Math Lecture Slides

What’s the formula for the line with the following properties?

1. The vertical intercept is 2, i.e. it passes through (0, 2);

2. The slope is 3.

Fact 2 means the height grows like 3x :one unit of x produces three units of y .So

y = 3x + ?.

To find the value of ?, use Fact 1:

2 = 3 · 0 + ?.

So ? has a value of 2.

y = 3x + 2

In general, the equation of a line can be written

y = mx + b (Slope-Intercept Form)

where m is the slope and b is from the vertical intercept.

Page 560: Finite Math Lecture Slides

What’s the formula for the line with the following properties?

1. The vertical intercept is 2, i.e. it passes through (0, 2);

2. The slope is 3.

Fact 2 means the height grows like 3x :one unit of x produces three units of y .So

y = 3x + ?.

To find the value of ?, use Fact 1:

2 = 3 · 0 + ?.

So ? has a value of 2.

y = 3x + 2

In general, the equation of a line can be written

y = mx + b (Slope-Intercept Form)

where m is the slope and b is from the vertical intercept.

Page 561: Finite Math Lecture Slides

What’s the formula for the line with the following properties?

1. The vertical intercept is 2, i.e. it passes through (0, 2);

2. The slope is 3.

Fact 2 means the height grows like 3x :one unit of x produces three units of y .So

y = 3x + ?.

To find the value of ?, use Fact 1:

2 = 3 · 0 + ?.

So ? has a value of 2.

y = 3x + 2

In general, the equation of a line can be written

y = mx + b (Slope-Intercept Form)

where m is the slope and b is from the vertical intercept.

Page 562: Finite Math Lecture Slides

What’s the formula for the line with the following properties?

1. The vertical intercept is 2, i.e. it passes through (0, 2);

2. The slope is 3.

Fact 2 means the height grows like 3x :one unit of x produces three units of y .So

y = 3x + ?.

To find the value of ?, use Fact 1:

2 = 3 · 0 + ?.

So ? has a value of 2.

y = 3x + 2

In general, the equation of a line can be written

y = mx + b (Slope-Intercept Form)

where m is the slope and b is from the vertical intercept.

Page 563: Finite Math Lecture Slides

Here’s another form of the linear equation:

Start with the definition of slope:

Slope = m =y2 − y1x2 − x1

.

Now rearrange terms to get rid of the fraction:

m(x2 − x1) = y2 − y1.

Since this holds for any (x2, y2) on the line, we can think of it asthe defining equation of the line:

m(x − x1) = y − y1.

This is called point-slope form.

Both forms have their strengths and weaknesses for problemsolving.

Page 564: Finite Math Lecture Slides

Here’s another form of the linear equation:

Start with the definition of slope:

Slope = m =y2 − y1x2 − x1

.

Now rearrange terms to get rid of the fraction:

m(x2 − x1) = y2 − y1.

Since this holds for any (x2, y2) on the line, we can think of it asthe defining equation of the line:

m(x − x1) = y − y1.

This is called point-slope form.

Both forms have their strengths and weaknesses for problemsolving.

Page 565: Finite Math Lecture Slides

Here’s another form of the linear equation:

Start with the definition of slope:

Slope = m =y2 − y1x2 − x1

.

Now rearrange terms to get rid of the fraction:

m(x2 − x1) = y2 − y1.

Since this holds for any (x2, y2) on the line, we can think of it asthe defining equation of the line:

m(x − x1) = y − y1.

This is called point-slope form.

Both forms have their strengths and weaknesses for problemsolving.

Page 566: Finite Math Lecture Slides

Here’s another form of the linear equation:

Start with the definition of slope:

Slope = m =y2 − y1x2 − x1

.

Now rearrange terms to get rid of the fraction:

m(x2 − x1) = y2 − y1.

Since this holds for any (x2, y2) on the line, we can think of it asthe defining equation of the line:

m(x − x1) = y − y1.

This is called point-slope form.

Both forms have their strengths and weaknesses for problemsolving.

Page 567: Finite Math Lecture Slides

Here’s another form of the linear equation:

Start with the definition of slope:

Slope = m =y2 − y1x2 − x1

.

Now rearrange terms to get rid of the fraction:

m(x2 − x1) = y2 − y1.

Since this holds for any (x2, y2) on the line, we can think of it asthe defining equation of the line:

m(x − x1) = y − y1.

This is called point-slope form.

Both forms have their strengths and weaknesses for problemsolving.

Page 568: Finite Math Lecture Slides

Here’s another form of the linear equation:

Start with the definition of slope:

Slope = m =y2 − y1x2 − x1

.

Now rearrange terms to get rid of the fraction:

m(x2 − x1) = y2 − y1.

Since this holds for any (x2, y2) on the line, we can think of it asthe defining equation of the line:

m(x − x1) = y − y1.

This is called point-slope form.

Both forms have their strengths and weaknesses for problemsolving.

Page 569: Finite Math Lecture Slides

Here’s another form of the linear equation:

Start with the definition of slope:

Slope = m =y2 − y1x2 − x1

.

Now rearrange terms to get rid of the fraction:

m(x2 − x1) = y2 − y1.

Since this holds for any (x2, y2) on the line, we can think of it asthe defining equation of the line:

m(x − x1) = y − y1.

This is called point-slope form.

Both forms have their strengths and weaknesses for problemsolving.

Page 570: Finite Math Lecture Slides

Types of problems:

1. Given information about a line, figure out its equation.

Point-Slope form is usually best.

2. Given the equation of a line, determine facts about it.

Slope-Intercept form is usually best.

Page 571: Finite Math Lecture Slides

Types of problems:

1. Given information about a line, figure out its equation.

Point-Slope form is usually best.

2. Given the equation of a line, determine facts about it.

Slope-Intercept form is usually best.

Page 572: Finite Math Lecture Slides

Types of problems:

1. Given information about a line, figure out its equation.

Point-Slope form is usually best.

2. Given the equation of a line, determine facts about it.

Slope-Intercept form is usually best.

Page 573: Finite Math Lecture Slides

Types of problems:

1. Given information about a line, figure out its equation.

Point-Slope form is usually best.

2. Given the equation of a line, determine facts about it.

Slope-Intercept form is usually best.

Page 574: Finite Math Lecture Slides

Types of problems:

1. Given information about a line, figure out its equation.

Point-Slope form is usually best.

2. Given the equation of a line, determine facts about it.

Slope-Intercept form is usually best.

Page 575: Finite Math Lecture Slides

Types of problems:

1. Given information about a line, figure out its equation.

Point-Slope form is usually best.

2. Given the equation of a line, determine facts about it.

Slope-Intercept form is usually best.

Page 576: Finite Math Lecture Slides

Types of problems:

1. Given information about a line, figure out its equation.

Point-Slope form is usually best.

2. Given the equation of a line, determine facts about it.

Slope-Intercept form is usually best.

Page 577: Finite Math Lecture Slides

Example:

Given the equation −4y + 2x + 8 = 0, find the slope andy -intercept.

Slope-Intercept form is usually best.

Solution Rewrite in Slope-Intercept form:

−4y = −2x − 8

y =1

2x + 2.

Now we can read off the info we need:

m =1

2b = 2

Page 578: Finite Math Lecture Slides

Example:

Given the equation −4y + 2x + 8 = 0, find the slope andy -intercept.

Slope-Intercept form is usually best.

Solution Rewrite in Slope-Intercept form:

−4y = −2x − 8

y =1

2x + 2.

Now we can read off the info we need:

m =1

2b = 2

Page 579: Finite Math Lecture Slides

Example:

Given the equation −4y + 2x + 8 = 0, find the slope andy -intercept.

Slope-Intercept form is usually best.

Solution Rewrite in Slope-Intercept form:

−4y = −2x − 8

y =1

2x + 2.

Now we can read off the info we need:

m =1

2b = 2

Page 580: Finite Math Lecture Slides

Example:

Given the equation −4y + 2x + 8 = 0, find the slope andy -intercept.

Slope-Intercept form is usually best.

Solution Rewrite in Slope-Intercept form:

−4y = −2x − 8

y =1

2x + 2.

Now we can read off the info we need:

m =1

2b = 2

Page 581: Finite Math Lecture Slides

Example:

A line passes through the points (4, 5) and (2,−1). What is theequation of the line?

Point-Slope form is usually best.

Solution:First find the slope.

m =y2 − y1x2 − x1

=−1− 5

2− 4=−6

−2= 3

Now apply the point-slope form.

y − 5 = 3(x − 4)

Page 582: Finite Math Lecture Slides

Example:

A line passes through the points (4, 5) and (2,−1). What is theequation of the line?

Point-Slope form is usually best.

Solution:First find the slope.

m =y2 − y1x2 − x1

=−1− 5

2− 4=−6

−2= 3

Now apply the point-slope form.

y − 5 = 3(x − 4)

Page 583: Finite Math Lecture Slides

Example:

A line passes through the points (4, 5) and (2,−1). What is theequation of the line?

Point-Slope form is usually best.

Solution:First find the slope.

m =y2 − y1x2 − x1

=−1− 5

2− 4=−6

−2= 3

Now apply the point-slope form.

y − 5 = 3(x − 4)

Page 584: Finite Math Lecture Slides

Example:

A line passes through the points (4, 5) and (2,−1). What is theequation of the line?

Point-Slope form is usually best.

Solution:First find the slope.

m =y2 − y1x2 − x1

=−1− 5

2− 4=−6

−2= 3

Now apply the point-slope form.

y − 5 = 3(x − 4)

Page 585: Finite Math Lecture Slides

Example:

A line passes through the points (4, 5) and (2,−1). What is theequation of the line?

Point-Slope form is usually best.

Solution:First find the slope.

m =y2 − y1x2 − x1

=−1− 5

2− 4=−6

−2= 3

Now apply the point-slope form.

y − 5 = 3(x − 4)

Page 586: Finite Math Lecture Slides

There are exceptions.

Example:

The population was 30,700 and grew at a rate of 850 per year.

1. Give the linear equation for p and t (years since 2000).

2. What is the population in the year 2010?

3. When will the population be 45,000?

Solution

1. p = 850t + 30, 700.

2. Plug in t = 10.

p = 850 · 10 + 30,700 = 39,200

3. Plug in p = 45,000.

45,000 = 850t + 30,70014,300 = 850t

t ≈ 16.8

Page 587: Finite Math Lecture Slides

There are exceptions.

Example:

The population was 30,700 and grew at a rate of 850 per year.

1. Give the linear equation for p and t (years since 2000).

2. What is the population in the year 2010?

3. When will the population be 45,000?

Solution

1. p = 850t + 30, 700.

2. Plug in t = 10.

p = 850 · 10 + 30,700 = 39,200

3. Plug in p = 45,000.

45,000 = 850t + 30,70014,300 = 850t

t ≈ 16.8

Page 588: Finite Math Lecture Slides

There are exceptions.

Example:

The population was 30,700 and grew at a rate of 850 per year.

1. Give the linear equation for p and t (years since 2000).

2. What is the population in the year 2010?

3. When will the population be 45,000?

Solution

1. p = 850t + 30, 700.

2. Plug in t = 10.

p = 850 · 10 + 30,700 = 39,200

3. Plug in p = 45,000.

45,000 = 850t + 30,70014,300 = 850t

t ≈ 16.8

Page 589: Finite Math Lecture Slides

There are exceptions.

Example:

The population was 30,700 and grew at a rate of 850 per year.

1. Give the linear equation for p and t (years since 2000).

2. What is the population in the year 2010?

3. When will the population be 45,000?

Solution

1. p = 850t + 30, 700.

2. Plug in t = 10.

p = 850 · 10 + 30,700 = 39,200

3. Plug in p = 45,000.

45,000 = 850t + 30,70014,300 = 850t

t ≈ 16.8

Page 590: Finite Math Lecture Slides

There are exceptions.

Example:

The population was 30,700 and grew at a rate of 850 per year.

1. Give the linear equation for p and t (years since 2000).

2. What is the population in the year 2010?

3. When will the population be 45,000?

Solution

1. p = 850t + 30, 700.

2. Plug in t = 10.

p = 850 · 10 + 30,700 = 39,200

3. Plug in p = 45,000.

45,000 = 850t + 30,70014,300 = 850t

t ≈ 16.8

Page 591: Finite Math Lecture Slides

There are exceptions.

Example:

The population was 30,700 and grew at a rate of 850 per year.

1. Give the linear equation for p and t (years since 2000).

2. What is the population in the year 2010?

3. When will the population be 45,000?

Solution

1. p = 850t + 30, 700.

2. Plug in t = 10.

p = 850 · 10 + 30,700 = 39,200

3. Plug in p = 45,000.

45,000 = 850t + 30,70014,300 = 850t

t ≈ 16.8

Page 592: Finite Math Lecture Slides

There are exceptions.

Example:

The population was 30,700 and grew at a rate of 850 per year.

1. Give the linear equation for p and t (years since 2000).

2. What is the population in the year 2010?

3. When will the population be 45,000?

Solution

1. p = 850t + 30, 700.

2. Plug in t = 10.

p = 850 · 10 + 30,700 = 39,200

3. Plug in p = 45,000.

45,000 = 850t + 30,70014,300 = 850t

t ≈ 16.8

Page 593: Finite Math Lecture Slides

There are exceptions.

Example:

The population was 30,700 and grew at a rate of 850 per year.

1. Give the linear equation for p and t (years since 2000).

2. What is the population in the year 2010?

3. When will the population be 45,000?

Solution

1. p = 850t + 30, 700.

2. Plug in t = 10.

p = 850 · 10 + 30,700 = 39,200

3. Plug in p = 45,000.

45,000 = 850t + 30,70014,300 = 850t

t ≈ 16.8

Page 594: Finite Math Lecture Slides

There are exceptions.

Example:

The population was 30,700 and grew at a rate of 850 per year.

1. Give the linear equation for p and t (years since 2000).

2. What is the population in the year 2010?

3. When will the population be 45,000?

Solution

1. p = 850t + 30, 700.

2. Plug in t = 10.

p = 850 · 10 + 30,700 = 39,200

3. Plug in p = 45,000.

45,000 = 850t + 30,70014,300 = 850t

t ≈ 16.8

Page 595: Finite Math Lecture Slides

There are exceptions.

Example:

The population was 30,700 and grew at a rate of 850 per year.

1. Give the linear equation for p and t (years since 2000).

2. What is the population in the year 2010?

3. When will the population be 45,000?

Solution

1. p = 850t + 30, 700.

2. Plug in t = 10.

p = 850 · 10 + 30,700 = 39,200

3. Plug in p = 45,000.

45,000 = 850t + 30,70014,300 = 850t

t ≈ 16.8

Page 596: Finite Math Lecture Slides

There are exceptions.

Example:

The population was 30,700 and grew at a rate of 850 per year.

1. Give the linear equation for p and t (years since 2000).

2. What is the population in the year 2010?

3. When will the population be 45,000?

Solution

1. p = 850t + 30, 700.

2. Plug in t = 10.

p = 850 · 10 + 30,700 = 39,200

3. Plug in p = 45,000.

45,000 = 850t + 30,700

14,300 = 850tt ≈ 16.8

Page 597: Finite Math Lecture Slides

There are exceptions.

Example:

The population was 30,700 and grew at a rate of 850 per year.

1. Give the linear equation for p and t (years since 2000).

2. What is the population in the year 2010?

3. When will the population be 45,000?

Solution

1. p = 850t + 30, 700.

2. Plug in t = 10.

p = 850 · 10 + 30,700 = 39,200

3. Plug in p = 45,000.

45,000 = 850t + 30,70014,300 = 850t

t ≈ 16.8

Page 598: Finite Math Lecture Slides

There are exceptions.

Example:

The population was 30,700 and grew at a rate of 850 per year.

1. Give the linear equation for p and t (years since 2000).

2. What is the population in the year 2010?

3. When will the population be 45,000?

Solution

1. p = 850t + 30, 700.

2. Plug in t = 10.

p = 850 · 10 + 30,700 = 39,200

3. Plug in p = 45,000.

45,000 = 850t + 30,70014,300 = 850t

t ≈ 16.8

Page 599: Finite Math Lecture Slides

Example:

Latitude # Species

11◦ 3444◦ 26

(a) First find the slope:

m =N2 − N1

l2 − l1=

34− 26

11− 44= − 8

33.

Now apply point-slope form:

N − 26 = − 8

33(l − 44).

Page 600: Finite Math Lecture Slides

Example:

Latitude # Species

11◦ 3444◦ 26

(a) First find the slope:

m =N2 − N1

l2 − l1=

34− 26

11− 44= − 8

33.

Now apply point-slope form:

N − 26 = − 8

33(l − 44).

Page 601: Finite Math Lecture Slides

Example:

Latitude # Species

11◦ 3444◦ 26

(a) First find the slope:

m =N2 − N1

l2 − l1=

34− 26

11− 44= − 8

33.

Now apply point-slope form:

N − 26 = − 8

33(l − 44).

Page 602: Finite Math Lecture Slides

Example:

Latitude # Species

11◦ 3444◦ 26

(a) First find the slope:

m =N2 − N1

l2 − l1=

34− 26

11− 44= − 8

33.

Now apply point-slope form:

N − 26 = − 8

33(l − 44).

Page 603: Finite Math Lecture Slides

N − 26 = − 8

33(l − 44).

(b) Find the slope and y -intercept.

Let’s first rewrite the equation in slope-intercept form:

N = − 8

33l +

8

3344 + 26 = − 8

33l +

110

3.

Now we can say:

m = − 833 species per degree;

b = 1103 species at the equator.

(c) Graph the line.

Page 604: Finite Math Lecture Slides

N − 26 = − 8

33(l − 44).

(b) Find the slope and y -intercept.Let’s first rewrite the equation in slope-intercept form:

N = − 8

33l +

8

3344 + 26 = − 8

33l +

110

3.

Now we can say:

m = − 833 species per degree;

b = 1103 species at the equator.

(c) Graph the line.

Page 605: Finite Math Lecture Slides

N − 26 = − 8

33(l − 44).

(b) Find the slope and y -intercept.Let’s first rewrite the equation in slope-intercept form:

N = − 8

33l +

8

3344 + 26 = − 8

33l +

110

3.

Now we can say:

m = − 833 species per degree;

b = 1103 species at the equator.

(c) Graph the line.

Page 606: Finite Math Lecture Slides

N − 26 = − 8

33(l − 44).

(b) Find the slope and y -intercept.Let’s first rewrite the equation in slope-intercept form:

N = − 8

33l +

8

3344 + 26 = − 8

33l +

110

3.

Now we can say:

m = − 833 species per degree;

b = 1103 species at the equator.

(c) Graph the line.

Page 607: Finite Math Lecture Slides

N − 26 = − 8

33(l − 44).

(b) Find the slope and y -intercept.Let’s first rewrite the equation in slope-intercept form:

N = − 8

33l +

8

3344 + 26 = − 8

33l +

110

3.

Now we can say:

m = − 833 species per degree;

b = 1103 species at the equator.

(c) Graph the line.

Page 608: Finite Math Lecture Slides

Problem: A coffee shop sells coffee to go in three sizes.

Assuming the relationship between price and size is linear, whatdoes the 12 oz. coffee cost?

Page 609: Finite Math Lecture Slides

Problem: A coffee shop sells coffee to go in three sizes.

Assuming the relationship between price and size is linear, whatdoes the 12 oz. coffee cost?

Page 610: Finite Math Lecture Slides

Problem: A coffee shop sells coffee to go in three sizes.

Assuming the relationship between price and size is linear, whatdoes the 12 oz. coffee cost?

Page 611: Finite Math Lecture Slides

Size in oz. Price in $x y

8 1.0012 ?20 2.08

(It’s OK to reverse x and y , just be consistent.)

Solution: First find the slope:

m =2.08− 1.00

20− 8=

1.08

12= 0.09.

Thereforey − 1.00 = 0.09(x − 8)

andy = 0.09x + 0.28.

Finally, when x = 12,

y = 0.09 · 12 + 0.28 = $1.36.

Page 612: Finite Math Lecture Slides

Size in oz. Price in $x y

8 1.0012 ?20 2.08

(It’s OK to reverse x and y , just be consistent.)

Solution: First find the slope:

m =2.08− 1.00

20− 8=

1.08

12= 0.09.

Thereforey − 1.00 = 0.09(x − 8)

andy = 0.09x + 0.28.

Finally, when x = 12,

y = 0.09 · 12 + 0.28 = $1.36.

Page 613: Finite Math Lecture Slides

Size in oz. Price in $x y

8 1.0012 ?20 2.08

(It’s OK to reverse x and y , just be consistent.)

Solution: First find the slope:

m =2.08− 1.00

20− 8=

1.08

12= 0.09.

Thereforey − 1.00 = 0.09(x − 8)

andy = 0.09x + 0.28.

Finally, when x = 12,

y = 0.09 · 12 + 0.28 = $1.36.

Page 614: Finite Math Lecture Slides

Size in oz. Price in $x y

8 1.0012 ?20 2.08

(It’s OK to reverse x and y , just be consistent.)

Solution: First find the slope:

m =2.08− 1.00

20− 8=

1.08

12= 0.09.

Thereforey − 1.00 = 0.09(x − 8)

andy = 0.09x + 0.28.

Finally, when x = 12,

y = 0.09 · 12 + 0.28 = $1.36.

Page 615: Finite Math Lecture Slides

Size in oz. Price in $x y

8 1.0012 ?20 2.08

(It’s OK to reverse x and y , just be consistent.)

Solution: First find the slope:

m =2.08− 1.00

20− 8=

1.08

12= 0.09.

Thereforey − 1.00 = 0.09(x − 8)

andy = 0.09x + 0.28.

Finally, when x = 12,

y = 0.09 · 12 + 0.28 = $1.36.

Page 616: Finite Math Lecture Slides

Size in oz. Price in $x y

8 1.0012 ?20 2.08

(It’s OK to reverse x and y , just be consistent.)

Solution: First find the slope:

m =2.08− 1.00

20− 8=

1.08

12= 0.09.

Thereforey − 1.00 = 0.09(x − 8)

andy = 0.09x + 0.28.

Finally, when x = 12,

y = 0.09 · 12 + 0.28 = $1.36.

Page 617: Finite Math Lecture Slides

y = 0.09x + 0.28.

Problem: What do m = 0.09 and b = 0.28 (the y -intercept)mean in this problem?

Solution: The slope is rise/run, in this case $/oz.

So evidently coffee costs 9¢ per ounce.

The value of b is the value that y takes when x = 0.

So if you buy 0 ounces of coffee it still costs you 28¢.

Evidently that is the price of renting a mug.

b = fixed costsmx = variable costs

Page 618: Finite Math Lecture Slides

y = 0.09x + 0.28.

Problem: What do m = 0.09 and b = 0.28 (the y -intercept)mean in this problem?

Solution: The slope is rise/run, in this case $/oz.

So evidently coffee costs 9¢ per ounce.

The value of b is the value that y takes when x = 0.

So if you buy 0 ounces of coffee it still costs you 28¢.

Evidently that is the price of renting a mug.

b = fixed costsmx = variable costs

Page 619: Finite Math Lecture Slides

y = 0.09x + 0.28.

Problem: What do m = 0.09 and b = 0.28 (the y -intercept)mean in this problem?

Solution: The slope is rise/run, in this case $/oz.

So evidently coffee costs 9¢ per ounce.

The value of b is the value that y takes when x = 0.

So if you buy 0 ounces of coffee it still costs you 28¢.

Evidently that is the price of renting a mug.

b = fixed costsmx = variable costs

Page 620: Finite Math Lecture Slides

y = 0.09x + 0.28.

Problem: What do m = 0.09 and b = 0.28 (the y -intercept)mean in this problem?

Solution: The slope is rise/run, in this case $/oz.

So evidently coffee costs 9¢ per ounce.

The value of b is the value that y takes when x = 0.

So if you buy 0 ounces of coffee it still costs you 28¢.

Evidently that is the price of renting a mug.

b = fixed costsmx = variable costs

Page 621: Finite Math Lecture Slides

y = 0.09x + 0.28.

Problem: What do m = 0.09 and b = 0.28 (the y -intercept)mean in this problem?

Solution: The slope is rise/run, in this case $/oz.

So evidently coffee costs 9¢ per ounce.

The value of b is the value that y takes when x = 0.

So if you buy 0 ounces of coffee it still costs you 28¢.

Evidently that is the price of renting a mug.

b = fixed costsmx = variable costs

Page 622: Finite Math Lecture Slides

y = 0.09x + 0.28.

Problem: What do m = 0.09 and b = 0.28 (the y -intercept)mean in this problem?

Solution: The slope is rise/run, in this case $/oz.

So evidently coffee costs 9¢ per ounce.

The value of b is the value that y takes when x = 0.

So if you buy 0 ounces of coffee it still costs you 28¢.

Evidently that is the price of renting a mug.

b = fixed costsmx = variable costs

Page 623: Finite Math Lecture Slides

y = 0.09x + 0.28.

Problem: What do m = 0.09 and b = 0.28 (the y -intercept)mean in this problem?

Solution: The slope is rise/run, in this case $/oz.

So evidently coffee costs 9¢ per ounce.

The value of b is the value that y takes when x = 0.

So if you buy 0 ounces of coffee it still costs you 28¢.

Evidently that is the price of renting a mug.

b = fixed costsmx = variable costs

Page 624: Finite Math Lecture Slides

y = 0.09x + 0.28.

Problem: What do m = 0.09 and b = 0.28 (the y -intercept)mean in this problem?

Solution: The slope is rise/run, in this case $/oz.

So evidently coffee costs 9¢ per ounce.

The value of b is the value that y takes when x = 0.

So if you buy 0 ounces of coffee it still costs you 28¢.

Evidently that is the price of renting a mug.

b = fixed costsmx = variable costs

Page 625: Finite Math Lecture Slides

Systems of Equations

Sometimes it’s easier to set up story problems using more than onevariable.

Example: Suppose that a mother is 25 years older than her son,and that in 15 years she will be twice as old as her son. How oldare they now?

Solution: Letx = Mother’s agey = Son’s age.

The first phrase says that

x = y + 25,

and the second phrase says that

x + 15 = 2(y + 15).

Page 626: Finite Math Lecture Slides

Systems of Equations

Sometimes it’s easier to set up story problems using more than onevariable.

Example: Suppose that a mother is 25 years older than her son,and that in 15 years she will be twice as old as her son. How oldare they now?

Solution: Letx = Mother’s agey = Son’s age.

The first phrase says that

x = y + 25,

and the second phrase says that

x + 15 = 2(y + 15).

Page 627: Finite Math Lecture Slides

Systems of Equations

Sometimes it’s easier to set up story problems using more than onevariable.

Example: Suppose that a mother is 25 years older than her son,and that in 15 years she will be twice as old as her son. How oldare they now?

Solution: Letx = Mother’s agey = Son’s age.

The first phrase says that

x = y + 25,

and the second phrase says that

x + 15 = 2(y + 15).

Page 628: Finite Math Lecture Slides

Systems of Equations

Sometimes it’s easier to set up story problems using more than onevariable.

Example: Suppose that a mother is 25 years older than her son,and that in 15 years she will be twice as old as her son. How oldare they now?

Solution: Letx = Mother’s agey = Son’s age.

The first phrase says that

x = y + 25,

and the second phrase says that

x + 15 = 2(y + 15).

Page 629: Finite Math Lecture Slides

Systems of Equations

Sometimes it’s easier to set up story problems using more than onevariable.

Example: Suppose that a mother is 25 years older than her son,and that in 15 years she will be twice as old as her son. How oldare they now?

Solution: Letx = Mother’s agey = Son’s age.

The first phrase says that

x = y + 25,

and the second phrase says that

x + 15 = 2(y + 15).

Page 630: Finite Math Lecture Slides

Systems of Equations

Sometimes it’s easier to set up story problems using more than onevariable.

Example: Suppose that a mother is 25 years older than her son,and that in 15 years she will be twice as old as her son. How oldare they now?

Solution: Letx = Mother’s agey = Son’s age.

The first phrase says that

x = y + 25,

and the second phrase says that

x + 15 = 2(y + 15).

Page 631: Finite Math Lecture Slides

This gives us the system of Equations

x = y + 25x + 15 = 2(y + 15).

This particular system can be solved easilyusing the substitution method.

(y + 25) + 15 = 2(y + 15)

y + 40 = 2y + 30

−y − 30 = −y − 30

10 = y

Then x = 35.

Page 632: Finite Math Lecture Slides

This gives us the system of Equations

x = y + 25x + 15 = 2(y + 15).

This particular system can be solved easilyusing the substitution method.

(y + 25) + 15 = 2(y + 15)

y + 40 = 2y + 30

−y − 30 = −y − 30

10 = y

Then x = 35.

Page 633: Finite Math Lecture Slides

This gives us the system of Equations

x = y + 25x + 15 = 2(y + 15).

This particular system can be solved easilyusing the substitution method.

(y + 25) + 15 = 2(y + 15)

y + 40 = 2y + 30

−y − 30 = −y − 30

10 = y

Then x = 35.

Page 634: Finite Math Lecture Slides

This gives us the system of Equations

x = y + 25x + 15 = 2(y + 15).

This particular system can be solved easilyusing the substitution method.

(y + 25) + 15 = 2(y + 15)

y + 40 = 2y + 30

−y − 30 = −y − 30

10 = y

Then x = 35.

Page 635: Finite Math Lecture Slides

This gives us the system of Equations

x = y + 25x + 15 = 2(y + 15).

This particular system can be solved easilyusing the substitution method.

(y + 25) + 15 = 2(y + 15)

y + 40 = 2y + 30

−y − 30 = −y − 30

10 = y

Then x = 35.

Page 636: Finite Math Lecture Slides

This gives us the system of Equations

x = y + 25x + 15 = 2(y + 15).

This particular system can be solved easilyusing the substitution method.

(y + 25) + 15 = 2(y + 15)

y + 40 = 2y + 30

−y − 30 = −y − 30

10 = y

Then x = 35.

Page 637: Finite Math Lecture Slides

This gives us the system of Equations

x = y + 25x + 15 = 2(y + 15).

This particular system can be solved easilyusing the substitution method.

(y + 25) + 15 = 2(y + 15)

y + 40 = 2y + 30

−y − 30 = −y − 30

10 = y

Then x = 35.

Page 638: Finite Math Lecture Slides

Substitution Method

1. Solve for one variable in one equation.

2. Plug that result into the other equation.

3. Solve for the remaining variable.

4. Go back to (1) to evaluate the first variable.

Example:x + y = 2x − y = 0

1. x = 2− y

2. (2− y)− y = 0

3.2− 2y = 0−2y = −2

y = 1

4. x = 2− y = 2− 1 = 1

Page 639: Finite Math Lecture Slides

Substitution Method

1. Solve for one variable in one equation.

2. Plug that result into the other equation.

3. Solve for the remaining variable.

4. Go back to (1) to evaluate the first variable.

Example:x + y = 2x − y = 0

1. x = 2− y

2. (2− y)− y = 0

3.2− 2y = 0−2y = −2

y = 1

4. x = 2− y = 2− 1 = 1

Page 640: Finite Math Lecture Slides

Substitution Method

1. Solve for one variable in one equation.

2. Plug that result into the other equation.

3. Solve for the remaining variable.

4. Go back to (1) to evaluate the first variable.

Example:x + y = 2x − y = 0

1. x = 2− y

2. (2− y)− y = 0

3.2− 2y = 0−2y = −2

y = 1

4. x = 2− y = 2− 1 = 1

Page 641: Finite Math Lecture Slides

Substitution Method

1. Solve for one variable in one equation.

2. Plug that result into the other equation.

3. Solve for the remaining variable.

4. Go back to (1) to evaluate the first variable.

Example:x + y = 2x − y = 0

1. x = 2− y

2. (2− y)− y = 0

3.2− 2y = 0−2y = −2

y = 1

4. x = 2− y = 2− 1 = 1

Page 642: Finite Math Lecture Slides

Substitution Method

1. Solve for one variable in one equation.

2. Plug that result into the other equation.

3. Solve for the remaining variable.

4. Go back to (1) to evaluate the first variable.

Example:x + y = 2x − y = 0

1. x = 2− y

2. (2− y)− y = 0

3.2− 2y = 0−2y = −2

y = 1

4. x = 2− y = 2− 1 = 1

Page 643: Finite Math Lecture Slides

Substitution Method

1. Solve for one variable in one equation.

2. Plug that result into the other equation.

3. Solve for the remaining variable.

4. Go back to (1) to evaluate the first variable.

Example:x + y = 2x − y = 0

1. x = 2− y

2. (2− y)− y = 0

3.2− 2y = 0−2y = −2

y = 1

4. x = 2− y = 2− 1 = 1

Page 644: Finite Math Lecture Slides

Elimination Method

1. Put both equations in standard form (variables on left).

2. Multiply either or both equations by constant.

3. Add the equations together, eliminating one variable.

4. Solve for the remaining variable.

5. Plug that into the first equation and solve for the first variable.

Example:x + y = 2x − y = 0

1. Already done.

2. Already done.

3. 2x = 2.

4. x = 1.

5.1 + y = 2

y = 1

Page 645: Finite Math Lecture Slides

Elimination Method

1. Put both equations in standard form (variables on left).

2. Multiply either or both equations by constant.

3. Add the equations together, eliminating one variable.

4. Solve for the remaining variable.

5. Plug that into the first equation and solve for the first variable.

Example:x + y = 2x − y = 0

1. Already done.

2. Already done.

3. 2x = 2.

4. x = 1.

5.1 + y = 2

y = 1

Page 646: Finite Math Lecture Slides

Elimination Method

1. Put both equations in standard form (variables on left).

2. Multiply either or both equations by constant.

3. Add the equations together, eliminating one variable.

4. Solve for the remaining variable.

5. Plug that into the first equation and solve for the first variable.

Example:x + y = 2x − y = 0

1. Already done.

2. Already done.

3. 2x = 2.

4. x = 1.

5.1 + y = 2

y = 1

Page 647: Finite Math Lecture Slides

Elimination Method

1. Put both equations in standard form (variables on left).

2. Multiply either or both equations by constant.

3. Add the equations together, eliminating one variable.

4. Solve for the remaining variable.

5. Plug that into the first equation and solve for the first variable.

Example:x + y = 2x − y = 0

1. Already done.

2. Already done.

3. 2x = 2.

4. x = 1.

5.1 + y = 2

y = 1

Page 648: Finite Math Lecture Slides

Elimination Method

1. Put both equations in standard form (variables on left).

2. Multiply either or both equations by constant.

3. Add the equations together, eliminating one variable.

4. Solve for the remaining variable.

5. Plug that into the first equation and solve for the first variable.

Example:x + y = 2x − y = 0

1. Already done.

2. Already done.

3. 2x = 2.

4. x = 1.

5.1 + y = 2

y = 1

Page 649: Finite Math Lecture Slides

Elimination Method

1. Put both equations in standard form (variables on left).

2. Multiply either or both equations by constant.

3. Add the equations together, eliminating one variable.

4. Solve for the remaining variable.

5. Plug that into the first equation and solve for the first variable.

Example:x + y = 2x − y = 0

1. Already done.

2. Already done.

3. 2x = 2.

4. x = 1.

5.1 + y = 2

y = 1

Page 650: Finite Math Lecture Slides

Example: Solve the system

2x = 4− 3y4y = 5− 3x .

Solution:

1.2x + 3y = 43x + 4y = 5.

2. Multiply the first equation by 3, the second by −2.

6x + 9y = 12−6x − 8y = −10.

3. y = 24. y = 25.

2x = 4− 3(2)2x = −2

x = −1

Page 651: Finite Math Lecture Slides

Example: Solve the system

2x = 4− 3y4y = 5− 3x .

Solution:

1.2x + 3y = 43x + 4y = 5.

2. Multiply the first equation by 3, the second by −2.

6x + 9y = 12−6x − 8y = −10.

3. y = 24. y = 25.

2x = 4− 3(2)2x = −2

x = −1

Page 652: Finite Math Lecture Slides

Example: Solve the system

2x = 4− 3y4y = 5− 3x .

Solution:

1.2x + 3y = 43x + 4y = 5.

2. Multiply the first equation by 3, the second by −2.

6x + 9y = 12−6x − 8y = −10.

3. y = 24. y = 25.

2x = 4− 3(2)2x = −2

x = −1

Page 653: Finite Math Lecture Slides

Example: Solve the system

2x = 4− 3y4y = 5− 3x .

Solution:

1.2x + 3y = 43x + 4y = 5.

2. Multiply the first equation by 3, the second by −2.

6x + 9y = 12−6x − 8y = −10.

3. y = 24. y = 2

5.2x = 4− 3(2)2x = −2

x = −1

Page 654: Finite Math Lecture Slides

Example: Solve the system

2x = 4− 3y4y = 5− 3x .

Solution:

1.2x + 3y = 43x + 4y = 5.

2. Multiply the first equation by 3, the second by −2.

6x + 9y = 12−6x − 8y = −10.

3. y = 24. y = 25.

2x = 4− 3(2)2x = −2

x = −1

Page 655: Finite Math Lecture Slides

Graphic Systems

2x = 4− 3y4y = 5− 3x .

0.5

1

1.5

2

2.5

3

-2 -1.5 -1 -0.5 0 0.5

Solution: (−1, 2)The solution is the point of intersection.

Page 656: Finite Math Lecture Slides

Graphic Systems

2x = 4− 3y4y = 5− 3x .

0.5

1

1.5

2

2.5

3

-2 -1.5 -1 -0.5 0 0.5

Solution: (−1, 2)The solution is the point of intersection.

Page 657: Finite Math Lecture Slides

System WeirdnessThings can go wrong.

Example:x + y = 2

y = 4− x

Substitution produces

x + (4− x) = 24 = 2

Contradiction.

0

0.5

1

1.5

2

2.5

3

0 0.5 1 1.5 2 2.5 3

So this system of equationshas no solution,because the lines are parallel.

Page 658: Finite Math Lecture Slides

System WeirdnessThings can go wrong.

Example:x + y = 2

y = 4− x

Substitution produces

x + (4− x) = 24 = 2

Contradiction.

0

0.5

1

1.5

2

2.5

3

0 0.5 1 1.5 2 2.5 3

So this system of equationshas no solution,because the lines are parallel.

Page 659: Finite Math Lecture Slides

System WeirdnessThings can go wrong.

Example:x + y = 2

y = 4− x

Substitution produces

x + (4− x) = 2

4 = 2

Contradiction.

0

0.5

1

1.5

2

2.5

3

0 0.5 1 1.5 2 2.5 3

So this system of equationshas no solution,because the lines are parallel.

Page 660: Finite Math Lecture Slides

System WeirdnessThings can go wrong.

Example:x + y = 2

y = 4− x

Substitution produces

x + (4− x) = 24 = 2

Contradiction.

0

0.5

1

1.5

2

2.5

3

0 0.5 1 1.5 2 2.5 3

So this system of equationshas no solution,because the lines are parallel.

Page 661: Finite Math Lecture Slides

System WeirdnessThings can go wrong.

Example:x + y = 2

y = 4− x

Substitution produces

x + (4− x) = 24 = 2

Contradiction.

0

0.5

1

1.5

2

2.5

3

0 0.5 1 1.5 2 2.5 3

So this system of equationshas no solution,because the lines are parallel.

Page 662: Finite Math Lecture Slides

Here’s another flavor of weirdness.

x + y = 2y = 2− x

This time substitution produces

x + (2− x) = 22 = 2.

This is an identity.

The interpretation is thatthe two equations must have been equivalent.

So the lines are actually the same.

So in this case there are infinitely many solutions:every point on the line satisfies both equations.

Page 663: Finite Math Lecture Slides

Here’s another flavor of weirdness.

x + y = 2y = 2− x

This time substitution produces

x + (2− x) = 2

2 = 2.

This is an identity.

The interpretation is thatthe two equations must have been equivalent.

So the lines are actually the same.

So in this case there are infinitely many solutions:every point on the line satisfies both equations.

Page 664: Finite Math Lecture Slides

Here’s another flavor of weirdness.

x + y = 2y = 2− x

This time substitution produces

x + (2− x) = 22 = 2.

This is an identity.

The interpretation is thatthe two equations must have been equivalent.

So the lines are actually the same.

So in this case there are infinitely many solutions:every point on the line satisfies both equations.

Page 665: Finite Math Lecture Slides

Here’s another flavor of weirdness.

x + y = 2y = 2− x

This time substitution produces

x + (2− x) = 22 = 2.

This is an identity.

The interpretation is thatthe two equations must have been equivalent.

So the lines are actually the same.

So in this case there are infinitely many solutions:every point on the line satisfies both equations.

Page 666: Finite Math Lecture Slides

Here’s another flavor of weirdness.

x + y = 2y = 2− x

This time substitution produces

x + (2− x) = 22 = 2.

This is an identity.

The interpretation is thatthe two equations must have been equivalent.

So the lines are actually the same.

So in this case there are infinitely many solutions:every point on the line satisfies both equations.

Page 667: Finite Math Lecture Slides

Here’s another flavor of weirdness.

x + y = 2y = 2− x

This time substitution produces

x + (2− x) = 22 = 2.

This is an identity.

The interpretation is thatthe two equations must have been equivalent.

So the lines are actually the same.

So in this case there are infinitely many solutions:every point on the line satisfies both equations.

Page 668: Finite Math Lecture Slides

Here’s another flavor of weirdness.

x + y = 2y = 2− x

This time substitution produces

x + (2− x) = 22 = 2.

This is an identity.

The interpretation is thatthe two equations must have been equivalent.

So the lines are actually the same.

So in this case there are infinitely many solutions:every point on the line satisfies both equations.

Page 669: Finite Math Lecture Slides

x + y = 2y = 2− x

Infinitely many solutions.

Even in this case we can do a little more:

provide a recipe for producing as many solutions as desired.

Note that the second equation gives y as a function of x .

y = 2− x

This is a recipe for y in terms of x .

If we think of x as the input, then y is the output.

x is called the free variable or arbitrary parameter.

Page 670: Finite Math Lecture Slides

x + y = 2y = 2− x

Infinitely many solutions.

Even in this case we can do a little more:

provide a recipe for producing as many solutions as desired.

Note that the second equation gives y as a function of x .

y = 2− x

This is a recipe for y in terms of x .

If we think of x as the input, then y is the output.

x is called the free variable or arbitrary parameter.

Page 671: Finite Math Lecture Slides

x + y = 2y = 2− x

Infinitely many solutions.

Even in this case we can do a little more:

provide a recipe for producing as many solutions as desired.

Note that the second equation gives y as a function of x .

y = 2− x

This is a recipe for y in terms of x .

If we think of x as the input, then y is the output.

x is called the free variable or arbitrary parameter.

Page 672: Finite Math Lecture Slides

x + y = 2y = 2− x

Infinitely many solutions.

Even in this case we can do a little more:

provide a recipe for producing as many solutions as desired.

Note that the second equation gives y as a function of x .

y = 2− x

This is a recipe for y in terms of x .

If we think of x as the input, then y is the output.

x is called the free variable or arbitrary parameter.

Page 673: Finite Math Lecture Slides

x + y = 2y = 2− x

Infinitely many solutions.

Even in this case we can do a little more:

provide a recipe for producing as many solutions as desired.

Note that the second equation gives y as a function of x .

y = 2− x

This is a recipe for y in terms of x .

If we think of x as the input, then y is the output.

x is called the free variable or arbitrary parameter.

Page 674: Finite Math Lecture Slides

x + y = 2y = 2− x

Infinitely many solutions.

Even in this case we can do a little more:

provide a recipe for producing as many solutions as desired.

Note that the second equation gives y as a function of x .

y = 2− x

This is a recipe for y in terms of x .

If we think of x as the input, then y is the output.

x is called the free variable or arbitrary parameter.

Page 675: Finite Math Lecture Slides

x + y = 2y = 2− x

Infinitely many solutions.

Even in this case we can do a little more:

provide a recipe for producing as many solutions as desired.

Note that the second equation gives y as a function of x .

y = 2− x

This is a recipe for y in terms of x .

If we think of x as the input, then y is the output.

x is called the free variable or arbitrary parameter.

Page 676: Finite Math Lecture Slides

Reduction Method

Reduction Method = Elimination Method - Cleverness

Or alternately,

Reduction is Elimination by brute force.

This is not pretty.

Page 677: Finite Math Lecture Slides

Reduction Method

Reduction Method = Elimination Method - Cleverness

Or alternately,

Reduction is Elimination by brute force.

This is not pretty.

Page 678: Finite Math Lecture Slides

Reduction Method

Reduction Method = Elimination Method - Cleverness

Or alternately,

Reduction is Elimination by brute force.

This is not pretty.

Page 679: Finite Math Lecture Slides

Example2x + 3y = 43x + 4y = 5

Let’s change that 2x into a 1x .

12(2x + 3y = 4) x + 3

2y = 2=⇒

3x + 4y = 5 3x + 4y = 5

Note that we always write both equations together,but we modify only one at a time.

Page 680: Finite Math Lecture Slides

Example2x + 3y = 43x + 4y = 5

Let’s change that 2x into a 1x .

12(2x + 3y = 4) x + 3

2y = 2=⇒

3x + 4y = 5 3x + 4y = 5

Note that we always write both equations together,but we modify only one at a time.

Page 681: Finite Math Lecture Slides

Example2x + 3y = 43x + 4y = 5

Let’s change that 2x into a 1x .

12(2x + 3y = 4) x + 3

2y = 2=⇒

3x + 4y = 5 3x + 4y = 5

Note that we always write both equations together,but we modify only one at a time.

Page 682: Finite Math Lecture Slides

Example2x + 3y = 43x + 4y = 5

Let’s change that 2x into a 1x .

12(2x + 3y = 4) x + 3

2y = 2=⇒

3x + 4y = 5 3x + 4y = 5

Note that we always write both equations together,but we modify only one at a time.

Page 683: Finite Math Lecture Slides

x + 32y = 2

3x + 4y = 5

Now let’s eliminate the x .

−3(x + 32y = 2) −3x − 9

2y = −6=⇒

3x + 4y = 5 3x + 82y = 5

−12y = −1

Let’s regard the result as a new improved equation 2,and let’s keep the simpler old equation 1.

x + 32y = 2

−12y = −1

Page 684: Finite Math Lecture Slides

x + 32y = 2

3x + 4y = 5

Now let’s eliminate the x .

−3(x + 32y = 2) −3x − 9

2y = −6=⇒

3x + 4y = 5 3x + 82y = 5

−12y = −1

Let’s regard the result as a new improved equation 2,and let’s keep the simpler old equation 1.

x + 32y = 2

−12y = −1

Page 685: Finite Math Lecture Slides

x + 32y = 2

3x + 4y = 5

Now let’s eliminate the x .

−3(x + 32y = 2) −3x − 9

2y = −6=⇒

3x + 4y = 5 3x + 82y = 5

−12y = −1

Let’s regard the result as a new improved equation 2,and let’s keep the simpler old equation 1.

x + 32y = 2

−12y = −1

Page 686: Finite Math Lecture Slides

x + 32y = 2

3x + 4y = 5

Now let’s eliminate the x .

−3(x + 32y = 2) −3x − 9

2y = −6=⇒

3x + 4y = 5 3x + 82y = 5

−12y = −1

Let’s regard the result as a new improved equation 2,and let’s keep the simpler old equation 1.

x + 32y = 2

−12y = −1

Page 687: Finite Math Lecture Slides

x + 32y = 2

−12y = −1

Now let’s finish solving for y .

x + 32y = 2 x + 3

2y = 2=⇒

−2(−1

2y = −1) y = 2

Page 688: Finite Math Lecture Slides

x + 32y = 2

−12y = −1

Now let’s finish solving for y .

x + 32y = 2 x + 3

2y = 2=⇒

−2(−1

2y = −1) y = 2

Page 689: Finite Math Lecture Slides

x + 32y = 2

y = 2

Finally, let’s eliminate y from equation 1.

x + 32y = 2 x + 3

2y = 2=⇒

−32(y = 2) −3

2y = −3

x = −1

So the complete simplification (and solution) is

x = −1y = 2.

Page 690: Finite Math Lecture Slides

x + 32y = 2

y = 2

Finally, let’s eliminate y from equation 1.

x + 32y = 2 x + 3

2y = 2=⇒

−32(y = 2) −3

2y = −3

x = −1

So the complete simplification (and solution) is

x = −1y = 2.

Page 691: Finite Math Lecture Slides

x + 32y = 2

y = 2

Finally, let’s eliminate y from equation 1.

x + 32y = 2 x + 3

2y = 2=⇒

−32(y = 2) −3

2y = −3

x = −1

So the complete simplification (and solution) is

x = −1y = 2.

Page 692: Finite Math Lecture Slides

x + 32y = 2

y = 2

Finally, let’s eliminate y from equation 1.

x + 32y = 2 x + 3

2y = 2=⇒

−32(y = 2) −3

2y = −3

x = −1

So the complete simplification (and solution) is

x = −1y = 2.

Page 693: Finite Math Lecture Slides

Note there were only two things we had to know to do:·→ Multiply an equation by a constant.+→ Add a multiple of one equation to another.

You can also do:

↔ Reverse the equations.

Page 694: Finite Math Lecture Slides

Note there were only two things we had to know to do:·→ Multiply an equation by a constant.+→ Add a multiple of one equation to another.

You can also do:

↔ Reverse the equations.

Page 695: Finite Math Lecture Slides

Now let’s work on speed.

Shorthand notation: Instead of

2x + 3y = 43x + 4y = 5

we’ll write [2 3 43 4 5

].

Page 696: Finite Math Lecture Slides

[2 3 43 4 5

].

The first step: get a 1 in upper left.

1/2·→ R1 :

[1 3/2 23 4 5

]

Now use the 1 to cancel the 3.

−3R1+→ R2 :

[1 3/2 20 −1/2 −1

]Now get a 1 in the lower right.

−2·→ R2 :

[1 3/2 20 1 2

]Finally use the 1 in the lower right to cancel the 3/2.

−3/2R2+→ R1 :

[1 0 −10 1 2

]

Page 697: Finite Math Lecture Slides

[2 3 43 4 5

].

The first step: get a 1 in upper left.

1/2·→ R1 :

[1 3/2 23 4 5

]Now use the 1 to cancel the 3.

−3R1+→ R2 :

[1 3/2 20 −1/2 −1

]

Now get a 1 in the lower right.

−2·→ R2 :

[1 3/2 20 1 2

]Finally use the 1 in the lower right to cancel the 3/2.

−3/2R2+→ R1 :

[1 0 −10 1 2

]

Page 698: Finite Math Lecture Slides

[2 3 43 4 5

].

The first step: get a 1 in upper left.

1/2·→ R1 :

[1 3/2 23 4 5

]Now use the 1 to cancel the 3.

−3R1+→ R2 :

[1 3/2 20 −1/2 −1

]Now get a 1 in the lower right.

−2·→ R2 :

[1 3/2 20 1 2

]

Finally use the 1 in the lower right to cancel the 3/2.

−3/2R2+→ R1 :

[1 0 −10 1 2

]

Page 699: Finite Math Lecture Slides

[2 3 43 4 5

].

The first step: get a 1 in upper left.

1/2·→ R1 :

[1 3/2 23 4 5

]Now use the 1 to cancel the 3.

−3R1+→ R2 :

[1 3/2 20 −1/2 −1

]Now get a 1 in the lower right.

−2·→ R2 :

[1 3/2 20 1 2

]Finally use the 1 in the lower right to cancel the 3/2.

−3/2R2+→ R1 :

[1 0 −10 1 2

]

Page 700: Finite Math Lecture Slides

Note that the goal is to get it looking like[1 0 ∗0 1 ∗

].

We do it one column at a time, left to right.

In each column,

first get the 1,

then use the 1 to get the 0s.

Page 701: Finite Math Lecture Slides

Note that the goal is to get it looking like[1 0 ∗0 1 ∗

].

We do it one column at a time, left to right.

In each column,

first get the 1,

then use the 1 to get the 0s.

Page 702: Finite Math Lecture Slides

Note that the goal is to get it looking like[1 0 ∗0 1 ∗

].

We do it one column at a time, left to right.

In each column,

first get the 1,

then use the 1 to get the 0s.

Page 703: Finite Math Lecture Slides

Operations

You use the three operations·→,

+→, ↔ to get what you want.

1. To get a 1,·→ usually;↔ sometimes.

2. to get a 0, use+→, using 1s to make 0s.

Page 704: Finite Math Lecture Slides

Operations

You use the three operations·→,

+→, ↔ to get what you want.

1. To get a 1,

·→ usually;↔ sometimes.

2. to get a 0, use+→, using 1s to make 0s.

Page 705: Finite Math Lecture Slides

Operations

You use the three operations·→,

+→, ↔ to get what you want.

1. To get a 1,·→ usually;

↔ sometimes.

2. to get a 0, use+→, using 1s to make 0s.

Page 706: Finite Math Lecture Slides

Operations

You use the three operations·→,

+→, ↔ to get what you want.

1. To get a 1,·→ usually;↔ sometimes.

2. to get a 0, use+→, using 1s to make 0s.

Page 707: Finite Math Lecture Slides

Operations

You use the three operations·→,

+→, ↔ to get what you want.

1. To get a 1,·→ usually;↔ sometimes.

2. to get a 0, use+→, using 1s to make 0s.

Page 708: Finite Math Lecture Slides

Example: Solve the system

2x + 2y + 6z = −1x + y + 2z = 1

2x + 3y + z = 2

using matrix methods.

Solution: First translate the problem into matrix form. 2 2 6 −11 1 2 12 3 1 2

.

Page 709: Finite Math Lecture Slides

Example: Solve the system

2x + 2y + 6z = −1x + y + 2z = 1

2x + 3y + z = 2

using matrix methods.

Solution: First translate the problem into matrix form. 2 2 6 −11 1 2 12 3 1 2

.

Page 710: Finite Math Lecture Slides

2 2 6 −11 1 2 12 3 1 2

.Now we need a 1 in the upper left.

We could use 1/2·→ R1,

but that creates a fraction, and there is an alternative.

R1 ↔ R2 :

1 1 2 12 2 6 −12 3 1 2

.

Page 711: Finite Math Lecture Slides

2 2 6 −11 1 2 12 3 1 2

.Now we need a 1 in the upper left.

We could use 1/2·→ R1,

but that creates a fraction, and there is an alternative.

R1 ↔ R2 :

1 1 2 12 2 6 −12 3 1 2

.

Page 712: Finite Math Lecture Slides

2 2 6 −11 1 2 12 3 1 2

.Now we need a 1 in the upper left.

We could use 1/2·→ R1,

but that creates a fraction, and there is an alternative.

R1 ↔ R2 :

1 1 2 12 2 6 −12 3 1 2

.

Page 713: Finite Math Lecture Slides

1 1 2 12 2 6 −12 3 1 2

Now get 0s down the rest of the first column.

−2R1+→ R2 :

1 1 2 10 0 2 −32 3 1 2

−2R1+→ R3 :

1 1 2 10 0 2 −30 1 −3 0

Page 714: Finite Math Lecture Slides

1 1 2 12 2 6 −12 3 1 2

Now get 0s down the rest of the first column.

−2R1+→ R2 :

1 1 2 10 0 2 −32 3 1 2

−2R1+→ R3 :

1 1 2 10 0 2 −30 1 −3 0

Page 715: Finite Math Lecture Slides

1 1 2 12 2 6 −12 3 1 2

Now get 0s down the rest of the first column.

−2R1+→ R2 :

1 1 2 10 0 2 −32 3 1 2

−2R1+→ R3 :

1 1 2 10 0 2 −30 1 −3 0

Page 716: Finite Math Lecture Slides

1 1 2 10 0 2 −30 1 −3 0

Now we need a 1 in the 2,2 position.

Normally we’d use·→, but that won’t work here.

So instead we use

R2 ↔ R3 :

1 1 2 10 1 −3 00 0 2 −3

.Now finish the second column:

−1R2+→ R1 :

1 0 5 10 1 −3 00 0 2 −3

.

Page 717: Finite Math Lecture Slides

1 1 2 10 0 2 −30 1 −3 0

Now we need a 1 in the 2,2 position.

Normally we’d use·→, but that won’t work here.

So instead we use

R2 ↔ R3 :

1 1 2 10 1 −3 00 0 2 −3

.

Now finish the second column:

−1R2+→ R1 :

1 0 5 10 1 −3 00 0 2 −3

.

Page 718: Finite Math Lecture Slides

1 1 2 10 0 2 −30 1 −3 0

Now we need a 1 in the 2,2 position.

Normally we’d use·→, but that won’t work here.

So instead we use

R2 ↔ R3 :

1 1 2 10 1 −3 00 0 2 −3

.Now finish the second column:

−1R2+→ R1 :

1 0 5 10 1 −3 00 0 2 −3

.

Page 719: Finite Math Lecture Slides

1 0 5 10 1 −3 00 0 2 −3

.Finally the third column,

where we need a 1 in the 3,3 position.

1/2·→ R3 :

1 0 5 10 1 −3 00 0 1 −3/2

.

Page 720: Finite Math Lecture Slides

1 0 5 10 1 −3 00 0 2 −3

.Finally the third column,

where we need a 1 in the 3,3 position.

1/2·→ R3 :

1 0 5 10 1 −3 00 0 1 −3/2

.

Page 721: Finite Math Lecture Slides

1 0 5 10 1 −3 00 0 1 −3/2

Then clear out the upper entries.

3R3+→ R2 :

1 0 5 10 1 0 −9/20 0 1 −3/2

−5R3+→ R1 :

1 0 0 17/20 1 0 −9/20 0 1 −3/2

So we get the solution

x = 17/2y = −9/2

z = −3/2.

Page 722: Finite Math Lecture Slides

1 0 5 10 1 −3 00 0 1 −3/2

Then clear out the upper entries.

3R3+→ R2 :

1 0 5 10 1 0 −9/20 0 1 −3/2

−5R3+→ R1 :

1 0 0 17/20 1 0 −9/20 0 1 −3/2

So we get the solution

x = 17/2y = −9/2

z = −3/2.

Page 723: Finite Math Lecture Slides

1 0 5 10 1 −3 00 0 1 −3/2

Then clear out the upper entries.

3R3+→ R2 :

1 0 5 10 1 0 −9/20 0 1 −3/2

−5R3+→ R1 :

1 0 0 17/20 1 0 −9/20 0 1 −3/2

So we get the solution

x = 17/2y = −9/2

z = −3/2.

Page 724: Finite Math Lecture Slides

1 0 5 10 1 −3 00 0 1 −3/2

Then clear out the upper entries.

3R3+→ R2 :

1 0 5 10 1 0 −9/20 0 1 −3/2

−5R3+→ R1 :

1 0 0 17/20 1 0 −9/20 0 1 −3/2

So we get the solution

x = 17/2y = −9/2

z = −3/2.

Page 725: Finite Math Lecture Slides

Example: Here is a variation on the last problem.

2x + 2y + 6z = −1x + y + 2z = 1

2x + 2y + z = 2

Start off the same way.

R1 ↔ R2 :

1 1 2 12 2 6 −12 2 1 2

− 2R1+→ R2 :

1 1 2 10 0 2 −32 2 1 2

−2R1+→ R3 :

1 1 2 10 0 2 −30 0 −3 0

Page 726: Finite Math Lecture Slides

Example: Here is a variation on the last problem.

2x + 2y + 6z = −1x + y + 2z = 1

2x + 2y + z = 2

Start off the same way.

R1 ↔ R2 :

1 1 2 12 2 6 −12 2 1 2

− 2R1+→ R2 :

1 1 2 10 0 2 −32 2 1 2

−2R1+→ R3 :

1 1 2 10 0 2 −30 0 −3 0

Page 727: Finite Math Lecture Slides

Example: Here is a variation on the last problem.

2x + 2y + 6z = −1x + y + 2z = 1

2x + 2y + z = 2

Start off the same way.

R1 ↔ R2 :

1 1 2 12 2 6 −12 2 1 2

− 2R1+→ R2 :

1 1 2 10 0 2 −32 2 1 2

−2R1+→ R3 :

1 1 2 10 0 2 −30 0 −3 0

Page 728: Finite Math Lecture Slides

Example: Here is a variation on the last problem.

2x + 2y + 6z = −1x + y + 2z = 1

2x + 2y + z = 2

Start off the same way.

R1 ↔ R2 :

1 1 2 12 2 6 −12 2 1 2

− 2R1+→ R2 :

1 1 2 10 0 2 −32 2 1 2

−2R1+→ R3 :

1 1 2 10 0 2 −30 0 −3 0

Page 729: Finite Math Lecture Slides

1 1 2 10 0 2 −30 0 −3 0

Now we have a another problem.

There’s no way to make progress on column 2.

So we have no choice but to skip it and move on to column 3.

But where should the 1 go?

Just below the previous 1 in column 1.

1/2·→ R2 :

1 1 2 10 0 1 −3/20 0 −3 0

Those are called leading ones.

Page 730: Finite Math Lecture Slides

1 1 2 10 0 2 −30 0 −3 0

Now we have a another problem.

There’s no way to make progress on column 2.

So we have no choice but to skip it and move on to column 3.

But where should the 1 go?

Just below the previous 1 in column 1.

1/2·→ R2 :

1 1 2 10 0 1 −3/20 0 −3 0

Those are called leading ones.

Page 731: Finite Math Lecture Slides

1 1 2 10 0 2 −30 0 −3 0

Now we have a another problem.

There’s no way to make progress on column 2.

So we have no choice but to skip it and move on to column 3.

But where should the 1 go?

Just below the previous 1 in column 1.

1/2·→ R2 :

1 1 2 10 0 1 −3/20 0 −3 0

Those are called leading ones.

Page 732: Finite Math Lecture Slides

1 1 2 10 0 2 −30 0 −3 0

Now we have a another problem.

There’s no way to make progress on column 2.

So we have no choice but to skip it and move on to column 3.

But where should the 1 go?

Just below the previous 1 in column 1.

1/2·→ R2 :

1 1 2 10 0 1 −3/20 0 −3 0

Those are called leading ones.

Page 733: Finite Math Lecture Slides

1 1 2 10 0 2 −30 0 −3 0

Now we have a another problem.

There’s no way to make progress on column 2.

So we have no choice but to skip it and move on to column 3.

But where should the 1 go?

Just below the previous 1 in column 1.

1/2·→ R2 :

1 1 2 10 0 1 −3/20 0 −3 0

Those are called leading ones.

Page 734: Finite Math Lecture Slides

1 1 2 10 0 2 −30 0 −3 0

Now we have a another problem.

There’s no way to make progress on column 2.

So we have no choice but to skip it and move on to column 3.

But where should the 1 go?

Just below the previous 1 in column 1.

1/2·→ R2 :

1 1 2 10 0 1 −3/20 0 −3 0

Those are called leading ones.

Page 735: Finite Math Lecture Slides

1 1 2 10 0 1 −3/20 0 −3 0

Now the 0s.

−2R2+→ R1 :

1 1 0 40 0 1 −3/20 0 −3 0

3R2+→ R3 :

1 1 0 40 0 1 −3/20 0 0 −9/2

Page 736: Finite Math Lecture Slides

1 1 2 10 0 1 −3/20 0 −3 0

Now the 0s.

−2R2+→ R1 :

1 1 0 40 0 1 −3/20 0 −3 0

3R2+→ R3 :

1 1 0 40 0 1 −3/20 0 0 −9/2

Page 737: Finite Math Lecture Slides

1 1 0 40 0 1 −3/20 0 0 −9/2

Now we’re done with the row reduction,

so we can translate back to equation form.

x + y = 4z = −3/20 = −9/2

That last equation is trouble.

Conclusion: The system has no solution.

Page 738: Finite Math Lecture Slides

1 1 0 40 0 1 −3/20 0 0 −9/2

Now we’re done with the row reduction,

so we can translate back to equation form.

x + y = 4z = −3/20 = −9/2

That last equation is trouble.

Conclusion: The system has no solution.

Page 739: Finite Math Lecture Slides

1 1 0 40 0 1 −3/20 0 0 −9/2

Now we’re done with the row reduction,

so we can translate back to equation form.

x + y = 4z = −3/20 = −9/2

That last equation is trouble.

Conclusion: The system has no solution.

Page 740: Finite Math Lecture Slides

1 1 0 40 0 1 −3/20 0 0 −9/2

Now we’re done with the row reduction,

so we can translate back to equation form.

x + y = 4z = −3/20 = −9/2

That last equation is trouble.

Conclusion: The system has no solution.

Page 741: Finite Math Lecture Slides

Suppose, hypothetically, that −9/2 had been 0 instead. 1 1 0 40 0 1 −3/20 0 0 0

Then we getx + y = 4

z = −3/20 = 0

and the third equation is OK.

On the other hand,it doesn’t tell us anything useful,so let’s ignore it.

Page 742: Finite Math Lecture Slides

Suppose, hypothetically, that −9/2 had been 0 instead. 1 1 0 40 0 1 −3/20 0 0 0

Then we get

x + y = 4z = −3/20 = 0

and the third equation is OK.

On the other hand,it doesn’t tell us anything useful,so let’s ignore it.

Page 743: Finite Math Lecture Slides

Suppose, hypothetically, that −9/2 had been 0 instead. 1 1 0 40 0 1 −3/20 0 0 0

Then we get

x + y = 4z = −3/20 = 0

and the third equation is OK.

On the other hand,it doesn’t tell us anything useful,so let’s ignore it.

Page 744: Finite Math Lecture Slides

x + y = 4z = −3/2

Note that we now have more variables than equations.

That usually means infinitely many solutions.

Solve for the leading variables.

x = 4− yz = −3/2

We say that y is arbitrary or free.

It can take any value you want.

But once you decide on a value for y ,then x and z are determined by the equations.

Page 745: Finite Math Lecture Slides

x + y = 4z = −3/2

Note that we now have more variables than equations.

That usually means infinitely many solutions.

Solve for the leading variables.

x = 4− yz = −3/2

We say that y is arbitrary or free.

It can take any value you want.

But once you decide on a value for y ,then x and z are determined by the equations.

Page 746: Finite Math Lecture Slides

x + y = 4z = −3/2

Note that we now have more variables than equations.

That usually means infinitely many solutions.

Solve for the leading variables.

x = 4− yz = −3/2

We say that y is arbitrary or free.

It can take any value you want.

But once you decide on a value for y ,then x and z are determined by the equations.

Page 747: Finite Math Lecture Slides

x + y = 4z = −3/2

Note that we now have more variables than equations.

That usually means infinitely many solutions.

Solve for the leading variables.

x = 4− yz = −3/2

We say that y is arbitrary or free.

It can take any value you want.

But once you decide on a value for y ,then x and z are determined by the equations.

Page 748: Finite Math Lecture Slides

x = 4− yz = −3/2

For example, suppose y = 17.

Thenx = 4− 17 = −13

giving the complete solution

x = −13y = 17

z = −3/2.

Page 749: Finite Math Lecture Slides

x = 4− yz = −3/2

For example, suppose y = 17.Then

x = 4− 17 = −13

giving the complete solution

x = −13y = 17

z = −3/2.

Page 750: Finite Math Lecture Slides

x = 4− yz = −3/2

For example, suppose y = 17.Then

x = 4− 17 = −13

giving the complete solution

x = −13y = 17

z = −3/2.

Page 751: Finite Math Lecture Slides

Row Reduced Form

To recap the last example:

Sometimes you cannot achieve your goal of 1 0 0 ∗0 1 0 ∗0 0 1 ∗

.

In that case you have to settle for row-reduced form.

1. Each row has a leading 1;

2. every column with a leading 1 has 0s elsewhere;

3. leading 1s do not zigzag.

Page 752: Finite Math Lecture Slides

Row Reduced Form

To recap the last example:

Sometimes you cannot achieve your goal of 1 0 0 ∗0 1 0 ∗0 0 1 ∗

.In that case you have to settle for row-reduced form.

1. Each row has a leading 1;

2. every column with a leading 1 has 0s elsewhere;

3. leading 1s do not zigzag.

Page 753: Finite Math Lecture Slides

Row Reduced Form

To recap the last example:

Sometimes you cannot achieve your goal of 1 0 0 ∗0 1 0 ∗0 0 1 ∗

.In that case you have to settle for row-reduced form.

1. Each row has a leading 1;

2. every column with a leading 1 has 0s elsewhere;

3. leading 1s do not zigzag.

Page 754: Finite Math Lecture Slides

Row-reduced: [1 0 2 30 1 4 5

][

1 2 0 30 0 1 4

][

1 2 3 4]

Not row-reduced: [0 1 4 51 0 2 3

][

1 1 0 00 1 0 1

][−1 0 2 3

0 2 4 5

]

Page 755: Finite Math Lecture Slides

Row-reduced: [1 0 2 30 1 4 5

][

1 2 0 30 0 1 4

][

1 2 3 4]

Not row-reduced: [0 1 4 51 0 2 3

]

[1 1 0 00 1 0 1

][−1 0 2 3

0 2 4 5

]

Page 756: Finite Math Lecture Slides

Row-reduced: [1 0 2 30 1 4 5

][

1 2 0 30 0 1 4

][

1 2 3 4]

Not row-reduced: [0 1 4 51 0 2 3

][

1 1 0 00 1 0 1

]

[−1 0 2 3

0 2 4 5

]

Page 757: Finite Math Lecture Slides

Row-reduced: [1 0 2 30 1 4 5

][

1 2 0 30 0 1 4

][

1 2 3 4]

Not row-reduced: [0 1 4 51 0 2 3

][

1 1 0 00 1 0 1

][−1 0 2 3

0 2 4 5

]

Page 758: Finite Math Lecture Slides

Having gotten your matrx in row-reduced form:

1. check for nonsense like 0 = 5, which means “no solution”;

2. if the equations all look good, then solve for the leadingvariables;

3. the remaining variables (now on the right) are free.

Page 759: Finite Math Lecture Slides

Chapter 1: Matrices

A matrix is a rectangular array of numbers.[1 2 34 5 6

]

The size of a matrix is described by the number of rows andcolumns.

These numbers are called the dimensions of the matrix.

(Rows are horizontal; columns are vertical.)

This matrix is 2× 3. (# rows × # columns)

Page 760: Finite Math Lecture Slides

Chapter 1: Matrices

A matrix is a rectangular array of numbers.[1 2 34 5 6

]The size of a matrix is described by the number of rows andcolumns.

These numbers are called the dimensions of the matrix.

(Rows are horizontal; columns are vertical.)

This matrix is 2× 3. (# rows × # columns)

Page 761: Finite Math Lecture Slides

Matrices are usually denoted by capital letters.

A =

[1 2 34 5 6

]

Individual entries in a matrix are denoted by the correspondinglower-case letter, with subscripts to indicate the position.

a1,1 = 1 a1,2 = 2 a1,3 = 3a2,1 = 4 a2,2 = 5 a2,3 = 6

The commas can be omitted from the subscripts if it doesn’tcause confusion.

a11 = 1 a12 = 2 a13 = 3a21 = 4 a22 = 5 a23 = 6

Page 762: Finite Math Lecture Slides

Matrices are usually denoted by capital letters.

A =

[1 2 34 5 6

]Individual entries in a matrix are denoted by the corresponding

lower-case letter, with subscripts to indicate the position.

a1,1 = 1 a1,2 = 2 a1,3 = 3a2,1 = 4 a2,2 = 5 a2,3 = 6

The commas can be omitted from the subscripts if it doesn’tcause confusion.

a11 = 1 a12 = 2 a13 = 3a21 = 4 a22 = 5 a23 = 6

Page 763: Finite Math Lecture Slides

Matrices are usually denoted by capital letters.

A =

[1 2 34 5 6

]Individual entries in a matrix are denoted by the corresponding

lower-case letter, with subscripts to indicate the position.

a1,1 = 1 a1,2 = 2 a1,3 = 3a2,1 = 4 a2,2 = 5 a2,3 = 6

The commas can be omitted from the subscripts if it doesn’tcause confusion.

a11 = 1 a12 = 2 a13 = 3a21 = 4 a22 = 5 a23 = 6

Page 764: Finite Math Lecture Slides

A vector is a skinny matrix:

a matrix with only 1 row or only 1 column.

row vector: [1 2 3]

column vector:

123

Page 765: Finite Math Lecture Slides

Matrix Operations: +,−

Total no-brainers.

Example:[1 2 34 5 6

]−[−1 1 7

1 3 −4

]=

[2 1 −43 2 10

]

If the dimensions don’t match, the result is undefined.

Example: [1 24 5

]+

[−1 1 −7

1 3 −4

]= undefined

Page 766: Finite Math Lecture Slides

Matrix Operations: +,−

Total no-brainers.

Example:[1 2 34 5 6

]−[−1 1 7

1 3 −4

]=

[2 1 −43 2 10

]

If the dimensions don’t match, the result is undefined.

Example: [1 24 5

]+

[−1 1 −7

1 3 −4

]= undefined

Page 767: Finite Math Lecture Slides

Matrix Operations: +,−

Total no-brainers.

Example:[1 2 34 5 6

]−[−1 1 7

1 3 −4

]=

[2 1 −43 2 10

]

If the dimensions don’t match, the result is undefined.

Example: [1 24 5

]+

[−1 1 −7

1 3 −4

]= undefined

Page 768: Finite Math Lecture Slides

Matrix Operations: Scalar ·

There are 2 kinds of matrix multiplication, this is theeasy one.

Example:

3

[1 −2−3 4

]=

[3 −6−9 12

]An ordinary number can be called a scalar.

Scalar multiplication is always defined.

Page 769: Finite Math Lecture Slides

Matrix Operations: Scalar ·

There are 2 kinds of matrix multiplication, this is theeasy one.

Example:

3

[1 −2−3 4

]=

[3 −6−9 12

]

An ordinary number can be called a scalar.

Scalar multiplication is always defined.

Page 770: Finite Math Lecture Slides

Matrix Operations: Scalar ·

There are 2 kinds of matrix multiplication, this is theeasy one.

Example:

3

[1 −2−3 4

]=

[3 −6−9 12

]An ordinary number can be called a scalar.

Scalar multiplication is always defined.

Page 771: Finite Math Lecture Slides

Linear CombinationsIt’s common to combine addition and scalar multiplication.

Example: Suppose that

A =

[2 0−1 3

]B =

[−3 5

2 −1

].

What is 2A− 3B?

Solution:

2A− 3B = 2

[2 0−1 3

]− 3

[−3 5

2 −1

]

=

[4 0−2 6

]−

[−9 15

6 −3

]

=

[13 −15−8 9

].

Page 772: Finite Math Lecture Slides

Linear CombinationsIt’s common to combine addition and scalar multiplication.

Example: Suppose that

A =

[2 0−1 3

]B =

[−3 5

2 −1

].

What is 2A− 3B?

Solution:

2A− 3B = 2

[2 0−1 3

]− 3

[−3 5

2 −1

]

=

[4 0−2 6

]−

[−9 15

6 −3

]

=

[13 −15−8 9

].

Page 773: Finite Math Lecture Slides

Linear CombinationsIt’s common to combine addition and scalar multiplication.

Example: Suppose that

A =

[2 0−1 3

]B =

[−3 5

2 −1

].

What is 2A− 3B?

Solution:

2A− 3B = 2

[2 0−1 3

]− 3

[−3 5

2 −1

]

=

[4 0−2 6

]−

[−9 15

6 −3

]

=

[13 −15−8 9

].

Page 774: Finite Math Lecture Slides

Linear CombinationsIt’s common to combine addition and scalar multiplication.

Example: Suppose that

A =

[2 0−1 3

]B =

[−3 5

2 −1

].

What is 2A− 3B?

Solution:

2A− 3B = 2

[2 0−1 3

]− 3

[−3 5

2 −1

]

=

[4 0−2 6

]−

[−9 15

6 −3

]

=

[13 −15−8 9

].

Page 775: Finite Math Lecture Slides

Example: Suppose that

A =

[2 0−1 3

]B =

[−3 5

2 −1

].

What is 2A− 3B?

Alternate Solution:

2A− 3B = 2

[2 0−1 3

]+ −3

[−3 5

2 −1

]

=

[4 0−2 6

]+

[9 −15−6 3

]

=

[13 −15−8 9

].

Page 776: Finite Math Lecture Slides

Example: Suppose that

A =

[2 0−1 3

]B =

[−3 5

2 −1

].

What is 2A− 3B?

Alternate Solution:

2A− 3B = 2

[2 0−1 3

]+ −3

[−3 5

2 −1

]

=

[4 0−2 6

]+

[9 −15−6 3

]

=

[13 −15−8 9

].

Page 777: Finite Math Lecture Slides

Example: Suppose that

A =

[2 0−1 3

]B =

[−3 5

2 −1

].

What is 2A− 3B?

Alternate Solution:

2A− 3B = 2

[2 0−1 3

]+ −3

[−3 5

2 −1

]

=

[4 0−2 6

]+

[9 −15−6 3

]

=

[13 −15−8 9

].

Page 778: Finite Math Lecture Slides

Matrix Multiplication

This one is confusing.

Imagine that vectors are cannons,and numbers are cannon balls.

like this: 2−1

2

[4 2 3

]→ .

Page 779: Finite Math Lecture Slides

Matrix Multiplication

This one is confusing.

Imagine that vectors are cannons,and numbers are cannon balls.

like this: 2−1

2

[4 2 3

]→ .

Page 780: Finite Math Lecture Slides

Matrix Multiplication

This one is confusing.

Imagine that vectors are cannons,and numbers are cannon balls.

Think of this problem

[4 2 3

−2−1

2

Page 781: Finite Math Lecture Slides

Matrix Multiplication

This one is confusing.

Imagine that vectors are cannons,and numbers are cannon balls.

like this: 2−1

2

[4 2 3

]→ .

Page 782: Finite Math Lecture Slides

Matrix Multiplication

This one is confusing.

Imagine that vectors are cannons,and numbers are cannon balls.

like this: 2−1

2

[4 2 3

]→ .

Page 783: Finite Math Lecture Slides

−2

−1

2

[4 2 3

]→

Page 784: Finite Math Lecture Slides

−2

−1

[4 2

]

Page 785: Finite Math Lecture Slides

−2

−1

[4 2

]→

Page 786: Finite Math Lecture Slides

−2

−1

[4 2

]→ 6

Page 787: Finite Math Lecture Slides

−2

[4

]6

Page 788: Finite Math Lecture Slides

−2

[4

]→

Page 789: Finite Math Lecture Slides

−2

[4

]→ −2 + 6

Page 790: Finite Math Lecture Slides

[ ]−2 + 6

Page 791: Finite Math Lecture Slides

[ ]→

Page 792: Finite Math Lecture Slides

[ ]→ −8− 2 + 6

Page 793: Finite Math Lecture Slides

Thus, [4 2 3

−2−1

2

=[−4

]

Example:

[−3 −1 0

1−5

8

= ?

Solution: 1−5

8

↓[

−3 −1 0]→

[−3 · 1− 1(−5) + 0 · 8

]=[

2]

Page 794: Finite Math Lecture Slides

Thus, [4 2 3

−2−1

2

=[−4

]Example:

[−3 −1 0

1−5

8

= ?

Solution: 1−5

8

↓[

−3 −1 0]→

[−3 · 1− 1(−5) + 0 · 8

]=[

2]

Page 795: Finite Math Lecture Slides

Thus, [4 2 3

−2−1

2

=[−4

]Example:

[−3 −1 0

1−5

8

= ?

Solution: 1−5

8

↓[

−3 −1 0]→

[−3 · 1− 1(−5) + 0 · 8

]=[

2]

Page 796: Finite Math Lecture Slides

Thus, [4 2 3

−2−1

2

=[−4

]Example:

[−3 −1 0

1−5

8

= ?

Solution: 1−5

8

↓[

−3 −1 0]→

[−3 · 1− 1(−5) + 0 · 8

]

=[

2]

Page 797: Finite Math Lecture Slides

Thus, [4 2 3

−2−1

2

=[−4

]Example:

[−3 −1 0

1−5

8

= ?

Solution: 1−5

8

↓[

−3 −1 0]→

[−3 · 1− 1(−5) + 0 · 8

]=[

2]

Page 798: Finite Math Lecture Slides

Note that both vectors must be of the same length!

Example: [1 2

] 456

= Undefined

Page 799: Finite Math Lecture Slides

The same idea works for double barreled cannons.[−1 2

3 −4

]↓ ↓[

2 −3]→

[ ]

Page 800: Finite Math Lecture Slides

Fire! [−1 2

−4

]↓[

2]→

[−9

]

Page 801: Finite Math Lecture Slides

Fire! [2−4

]↓[ ]

→[−11

]

Page 802: Finite Math Lecture Slides

Reload. [2−4

]↓[

2 −3]→

[−11

]

Page 803: Finite Math Lecture Slides

Fire, and fire again. [ ]↓[ ]

→[−11 16

]

Page 804: Finite Math Lecture Slides

So we get

[2 −3

] [ −1 23 −4

]=[−11 16

].

Page 805: Finite Math Lecture Slides

In fact this works for almost any sizes of matrix:

The only restriction is that

the length of the rows (2nd dimension) in the left matrix=

the length of the columns (1st dimension) in the right matrix.

Example: [1 2 3

] 4 56 78 9

Can we multiply a 1× 3 matrix by a 3× 2 matrix?Yes, because

the 2nd dimension of the first=

the 1st dimension of the second.

Moreover the result is 1× 2.

(1× 3)(3× 2) = (1×63)( 63× 2) = 1× 2

Page 806: Finite Math Lecture Slides

In fact this works for almost any sizes of matrix:

The only restriction is that

the length of the rows (2nd dimension) in the left matrix=

the length of the columns (1st dimension) in the right matrix.

Example: [1 2 3

] 4 56 78 9

Can we multiply a 1× 3 matrix by a 3× 2 matrix?

Yes, because

the 2nd dimension of the first=

the 1st dimension of the second.

Moreover the result is 1× 2.

(1× 3)(3× 2) = (1×63)( 63× 2) = 1× 2

Page 807: Finite Math Lecture Slides

In fact this works for almost any sizes of matrix:

The only restriction is that

the length of the rows (2nd dimension) in the left matrix=

the length of the columns (1st dimension) in the right matrix.

Example: [1 2 3

] 4 56 78 9

Can we multiply a 1× 3 matrix by a 3× 2 matrix?Yes, because

the 2nd dimension of the first=

the 1st dimension of the second.

Moreover the result is 1× 2.

(1× 3)(3× 2) = (1×63)( 63× 2) = 1× 2

Page 808: Finite Math Lecture Slides

In fact this works for almost any sizes of matrix:

The only restriction is that

the length of the rows (2nd dimension) in the left matrix=

the length of the columns (1st dimension) in the right matrix.

Example: [1 2 3

] 4 56 78 9

Can we multiply a 1× 3 matrix by a 3× 2 matrix?Yes, because

the 2nd dimension of the first=

the 1st dimension of the second.

Moreover the result is 1× 2.

(1× 3)(3× 2) = (1×63)( 63× 2) = 1× 2

Page 809: Finite Math Lecture Slides

If both dimensions are the same we say the matrix is square.[−2 1

4 −3

]↓ ↓[

2 −30 1

]→→

[ ]

Page 810: Finite Math Lecture Slides

First row on left against first column on right.[−2 1

4 −3

]↓[

2 −30 1

]→

[ ]

Page 811: Finite Math Lecture Slides

Fire both rounds. [1−3

]↓[

0 1

]→

[−16

]

Page 812: Finite Math Lecture Slides

Second row on left against second column on right.[1−3

]↓[

0 1

]→

[−16

]

Page 813: Finite Math Lecture Slides

Fire both rounds. [ ]↓[ ]

[−16

−3

]

Page 814: Finite Math Lecture Slides

Reload. [−2 1

4 −3

][

2 −30 1

] [−16

−3

]

Page 815: Finite Math Lecture Slides

First row on left against second column on right.[−2 1

4 −3

]↓[

2 −30 1

]→

[−16

−3

]

Page 816: Finite Math Lecture Slides

Fire. [−2

4

]↓[

0 1

]→

[−16 11

−3

]

Page 817: Finite Math Lecture Slides

Second row on left against first column on right.[−2

4

]↓[

0 1

]→

[−16 11

−3

]

Page 818: Finite Math Lecture Slides

Fire. [ ]↓[ ]

[−16 11

4 −3

]

So [2 −30 1

] [−2 1

4 −3

]=

[−16 11

4 −3

].

Page 819: Finite Math Lecture Slides

Fire. [ ]↓[ ]

[−16 11

4 −3

]So [

2 −30 1

] [−2 1

4 −3

]=

[−16 11

4 −3

].

Page 820: Finite Math Lecture Slides

Example: [3 −11 2

] [−1 −5

3 2

]= ?

Solution: [−1 −5

3 2

][

3 −11 2

] [−6 −17

5 −1

]Note: [

31

](−1) +

[−1

2

](3) =

[−6

5

][

31

](−5) +

[−1

2

](2) =

[−17−1

]So the each column in the answer is a linear combination of thecolumns in the left factor.

Page 821: Finite Math Lecture Slides

Example: [3 −11 2

] [−1 −5

3 2

]= ?

Solution: [−1 −5

3 2

][

3 −11 2

] [−6 −17

5 −1

]

Note: [31

](−1) +

[−1

2

](3) =

[−6

5

][

31

](−5) +

[−1

2

](2) =

[−17−1

]So the each column in the answer is a linear combination of thecolumns in the left factor.

Page 822: Finite Math Lecture Slides

Example: [3 −11 2

] [−1 −5

3 2

]= ?

Solution: [−1 −5

3 2

][

3 −11 2

] [−6 −17

5 −1

]Note: [

31

](−1) +

[−1

2

](3) =

[−6

5

][

31

](−5) +

[−1

2

](2) =

[−17−1

]So the each column in the answer is a linear combination of thecolumns in the left factor.

Page 823: Finite Math Lecture Slides

Matrix multiplication is not commutative!

This means that usually

AB 6= BA.

This is very unlike ordinary arithmetic.

You must take care with the order in which matrices are multiplied!

Page 824: Finite Math Lecture Slides

Matrix multiplication is not commutative!

This means that usually

AB 6= BA.

This is very unlike ordinary arithmetic.

You must take care with the order in which matrices are multiplied!

Page 825: Finite Math Lecture Slides

Matrix multiplication is not commutative!

This means that usually

AB 6= BA.

This is very unlike ordinary arithmetic.

You must take care with the order in which matrices are multiplied!

Page 826: Finite Math Lecture Slides

Special Matrices

In ordinary arithmetic there are two special numbers:

0, the additive identity, for which

0 + x = x ;

and 1, the multiplicative identity, for which

1 · x = x .

Page 827: Finite Math Lecture Slides

There are special matrices with the same properties:

the zero matrix 0 with the property that

0 + X = X ;

and the identity matrix I with the property that

I · X = X and X · I = X .

Page 828: Finite Math Lecture Slides

All the entries of the zero matrix 0 are 0.

Example: Suppose that

X =

[1 2 34 5 6

].

Since X is 2× 3, the corresponding zero matrix should be

0 =

[0 0 00 0 0

].

Then0 + X = X + 0 = X .

Page 829: Finite Math Lecture Slides

All the entries of the zero matrix 0 are 0.

Example: Suppose that

X =

[1 2 34 5 6

].

Since X is 2× 3, the corresponding zero matrix should be

0 =

[0 0 00 0 0

].

Then0 + X = X + 0 = X .

Page 830: Finite Math Lecture Slides

All the entries of the zero matrix 0 are 0.

Example: Suppose that

X =

[1 2 34 5 6

].

Since X is 2× 3, the corresponding zero matrix should be

0 =

[0 0 00 0 0

].

Then0 + X = X + 0 = X .

Page 831: Finite Math Lecture Slides

The identity matrix is trickier.

It also comes in different sizes, but is always square.

I1 =[

1],

I2 =

[1 00 1

],

I3 =

1 0 00 1 00 0 1

,etc.

Page 832: Finite Math Lecture Slides

Example: Suppose that

X =

[1 2 34 5 6

].

Then

I2 X = X and X I3 = X .(2× 2) (2× 3) (2× 3) (3× 3)

Notice that we need a different sized I on each side of X .

But since the size of I is determined by its neighbor,we can omit the subscript.

IX = X and X I = X .

Page 833: Finite Math Lecture Slides

Example: Suppose that

X =

[1 2 34 5 6

].

Then

I2 X = X and X I3 = X .(2× 2) (2× 3) (2× 3) (3× 3)

Notice that we need a different sized I on each side of X .

But since the size of I is determined by its neighbor,we can omit the subscript.

IX = X and X I = X .

Page 834: Finite Math Lecture Slides

Matrix Inverses

In ordinary algebra the multiplicative inverse is used constantly.For example,

2x = 612 · 2x = 1

2 · 61x = 3

x = 3.

Wouldn’t it be nice if we had the same for matrices?

Page 835: Finite Math Lecture Slides

[1 23 4

] [xy

]=

[56

]

[−2 1

32 −1

2

] [1 23 4

] [xy

]=

[−2 1

32 −1

2

] [56

][

1 00 1

] [xy

]=

[−4

92

][

xy

]=

[−4

92

]

Page 836: Finite Math Lecture Slides

[1 23 4

] [xy

]=

[56

][−2 1

32 −1

2

] [1 23 4

] [xy

]=

[−2 1

32 −1

2

] [56

]

[1 00 1

] [xy

]=

[−4

92

][

xy

]=

[−4

92

]

Page 837: Finite Math Lecture Slides

[1 23 4

] [xy

]=

[56

][−2 1

32 −1

2

] [1 23 4

] [xy

]=

[−2 1

32 −1

2

] [56

][

1 00 1

] [xy

]=

[−4

92

]

[xy

]=

[−4

92

]

Page 838: Finite Math Lecture Slides

[1 23 4

] [xy

]=

[56

][−2 1

32 −1

2

] [1 23 4

] [xy

]=

[−2 1

32 −1

2

] [56

][

1 00 1

] [xy

]=

[−4

92

][

xy

]=

[−4

92

]

Page 839: Finite Math Lecture Slides

[−2 1

32 −1

2

]is called the inverse matrix of

[1 23 4

]because

[−2 1

32 −1

2

] [1 23 4

]=

[1 00 1

]and [

1 23 4

] [−2 1

32 −1

2

]=

[1 00 1

].

As shorthand we can write[1 23 4

]−1=

[−2 1

32 −1

2

].

Note that also [−2 1

32 −1

2

]−1=

[1 23 4

].

Page 840: Finite Math Lecture Slides

[−2 1

32 −1

2

]is called the inverse matrix of

[1 23 4

]because

[−2 1

32 −1

2

] [1 23 4

]=

[1 00 1

]and [

1 23 4

] [−2 1

32 −1

2

]=

[1 00 1

].

As shorthand we can write[1 23 4

]−1=

[−2 1

32 −1

2

].

Note that also [−2 1

32 −1

2

]−1=

[1 23 4

].

Page 841: Finite Math Lecture Slides

[−2 1

32 −1

2

]is called the inverse matrix of

[1 23 4

]because

[−2 1

32 −1

2

] [1 23 4

]=

[1 00 1

]and [

1 23 4

] [−2 1

32 −1

2

]=

[1 00 1

].

As shorthand we can write[1 23 4

]−1=

[−2 1

32 −1

2

].

Note that also [−2 1

32 −1

2

]−1=

[1 23 4

].

Page 842: Finite Math Lecture Slides

In general, ifAB = I = BA

then we say B is the inverse of A.

Almost every square matrix has an inverse.

Only square matrices can have inverses,

but not every one does.

If a matrix does not have an inverse,we say it is singular or noninvertible.

Examples:

[1 2−2 −4

],

[1 2 34 5 6

],

[1 00 0

],

1 2 34 5 67 8 9

Page 843: Finite Math Lecture Slides

In general, ifAB = I = BA

then we say B is the inverse of A.

Almost every square matrix has an inverse.

Only square matrices can have inverses,

but not every one does.

If a matrix does not have an inverse,we say it is singular or noninvertible.

Examples:

[1 2−2 −4

],

[1 2 34 5 6

],

[1 00 0

],

1 2 34 5 67 8 9

Page 844: Finite Math Lecture Slides

How do you calculate the inverse of a matrix?

Row reduction.

Each of those row operations are equivalent to a multiplication.

3·→ R1 :

3 0 00 1 00 0 1

·

R2 ↔ R3 :

1 0 00 0 10 1 0

·

5R2+→ R3 :

1 0 00 1 00 5 1

·

Page 845: Finite Math Lecture Slides

How do you calculate the inverse of a matrix?

Row reduction.

Each of those row operations are equivalent to a multiplication.

3·→ R1 :

3 0 00 1 00 0 1

·

R2 ↔ R3 :

1 0 00 0 10 1 0

·

5R2+→ R3 :

1 0 00 1 00 5 1

·

Page 846: Finite Math Lecture Slides

How do you calculate the inverse of a matrix?

Row reduction.

Each of those row operations are equivalent to a multiplication.

3·→ R1 :

3 0 00 1 00 0 1

·

R2 ↔ R3 :

1 0 00 0 10 1 0

·

5R2+→ R3 :

1 0 00 1 00 5 1

·

Page 847: Finite Math Lecture Slides

How do you calculate the inverse of a matrix?

Row reduction.

Each of those row operations are equivalent to a multiplication.

3·→ R1 :

3 0 00 1 00 0 1

·

R2 ↔ R3 :

1 0 00 0 10 1 0

·

5R2+→ R3 :

1 0 00 1 00 5 1

·

Page 848: Finite Math Lecture Slides

So row reduction is equivalent to multiplication.

That means if −1 2 34 5 67 8 9

RR−→

1 0 00 1 00 0 1

then then same result can be gotten by matrix multiplication:

M7M6M5M4M3M2M1

−1 0 12 −1 21 0 1

=

1 0 00 1 00 0 1

with the Ms equivalent to the row reduction.

Then −1 0 12 −1 21 0 1

−1 = M7M6M5M4M3M2M1.

Page 849: Finite Math Lecture Slides

So row reduction is equivalent to multiplication.

That means if −1 2 34 5 67 8 9

RR−→

1 0 00 1 00 0 1

then then same result can be gotten by matrix multiplication:

M7M6M5M4M3M2M1

−1 0 12 −1 21 0 1

=

1 0 00 1 00 0 1

with the Ms equivalent to the row reduction.

Then −1 0 12 −1 21 0 1

−1 = M7M6M5M4M3M2M1.

Page 850: Finite Math Lecture Slides

So row reduction is equivalent to multiplication.

That means if −1 2 34 5 67 8 9

RR−→

1 0 00 1 00 0 1

then then same result can be gotten by matrix multiplication:

M7M6M5M4M3M2M1

−1 0 12 −1 21 0 1

=

1 0 00 1 00 0 1

with the Ms equivalent to the row reduction.

Then −1 0 12 −1 21 0 1

−1 = M7M6M5M4M3M2M1.

Page 851: Finite Math Lecture Slides

The only problem is:

how do we keep track of the Ms while doing the row reduction?

Simple trick:Row reduce −1 0 1 1 0 0

2 −1 2 0 1 01 0 1 0 0 1

instead.

Then the right side keeps track of the row operations performed onthe left side.

Page 852: Finite Math Lecture Slides

Calculating the Matrix Inverse

If A is a square matrix, then to calculate its inverse:

1. Set up the augmented matrix[A I

].

2. Row reduce.

3. If you succeed in getting [I B

]then B = A−1.

4. If you fail to get the identity on the left, then A is notinvertible.

Page 853: Finite Math Lecture Slides

Example: Find the matrix inverse of

A =

[1 23 4

].

Solution: Start off with the augmented matrix[1 2 1 03 4 0 1

]then row reduce.

Page 854: Finite Math Lecture Slides

Example: Find the matrix inverse of

A =

[1 23 4

].

Solution: Start off with the augmented matrix[1 2 1 03 4 0 1

]then row reduce.

Page 855: Finite Math Lecture Slides

[1 2 1 03 4 0 1

]

−3R1+→ R2 :

[1 2 1 00 −2 −3 1

]

−1/2·→ R2 :

[1 2 1 00 1 3/2 −1/2

]

−2R2+→ R1 :

[1 0 −2 10 1 3/2 −1/2

]So

A−1 =

[−2 13/2 −1/2

]You can check your answer: is AA−1 = I?

Page 856: Finite Math Lecture Slides

[1 2 1 03 4 0 1

]

−3R1+→ R2 :

[1 2 1 00 −2 −3 1

]

−1/2·→ R2 :

[1 2 1 00 1 3/2 −1/2

]

−2R2+→ R1 :

[1 0 −2 10 1 3/2 −1/2

]So

A−1 =

[−2 13/2 −1/2

]You can check your answer: is AA−1 = I?

Page 857: Finite Math Lecture Slides

[1 2 1 03 4 0 1

]

−3R1+→ R2 :

[1 2 1 00 −2 −3 1

]

−1/2·→ R2 :

[1 2 1 00 1 3/2 −1/2

]

−2R2+→ R1 :

[1 0 −2 10 1 3/2 −1/2

]

So

A−1 =

[−2 13/2 −1/2

]You can check your answer: is AA−1 = I?

Page 858: Finite Math Lecture Slides

[1 2 1 03 4 0 1

]

−3R1+→ R2 :

[1 2 1 00 −2 −3 1

]

−1/2·→ R2 :

[1 2 1 00 1 3/2 −1/2

]

−2R2+→ R1 :

[1 0 −2 10 1 3/2 −1/2

]So

A−1 =

[−2 13/2 −1/2

]

You can check your answer: is AA−1 = I?

Page 859: Finite Math Lecture Slides

[1 2 1 03 4 0 1

]

−3R1+→ R2 :

[1 2 1 00 −2 −3 1

]

−1/2·→ R2 :

[1 2 1 00 1 3/2 −1/2

]

−2R2+→ R1 :

[1 0 −2 10 1 3/2 −1/2

]So

A−1 =

[−2 13/2 −1/2

]You can check your answer: is AA−1 = I?

Page 860: Finite Math Lecture Slides

Solving Systems with Inverses

Note that the matrix equation −1 2 34 5 67 8 9

xyz

=

236

is equivalent to the system of equations

−x + 2y + 3z = 34x + 5y + 6z = 67x + 8y + 9z = 9.

So matrix inverses can also be used to solve systems.

Page 861: Finite Math Lecture Slides

Example: Solve the following system.

2x + z = 2−y + 2z = −1

x + z = 3.

Solution: First translate the system into matrix equation form. 2 0 10 −1 21 0 1

xyz

=

2−1

3

Page 862: Finite Math Lecture Slides

Example: Solve the following system.

2x + z = 2−y + 2z = −1

x + z = 3.

Solution: First translate the system into matrix equation form. 2 0 10 −1 21 0 1

xyz

=

2−1

3

Page 863: Finite Math Lecture Slides

2 0 10 −1 21 0 1

xyz

=

2−1

3

Now compute the inverse matrix. 2 0 1 1 0 0

0 −1 2 0 1 01 0 1 0 0 1

R1 ↔ R3

1 0 1 0 0 10 −1 2 0 1 02 0 1 1 0 0

−2R1+→ R3

1 0 1 0 0 10 −1 2 0 1 00 0 −1 1 0 −2

Page 864: Finite Math Lecture Slides

2 0 10 −1 21 0 1

xyz

=

2−1

3

Now compute the inverse matrix. 2 0 1 1 0 0

0 −1 2 0 1 01 0 1 0 0 1

R1 ↔ R3

1 0 1 0 0 10 −1 2 0 1 02 0 1 1 0 0

−2R1+→ R3

1 0 1 0 0 10 −1 2 0 1 00 0 −1 1 0 −2

Page 865: Finite Math Lecture Slides

2 0 10 −1 21 0 1

xyz

=

2−1

3

Now compute the inverse matrix. 2 0 1 1 0 0

0 −1 2 0 1 01 0 1 0 0 1

R1 ↔ R3

1 0 1 0 0 10 −1 2 0 1 02 0 1 1 0 0

−2R1+→ R3

1 0 1 0 0 10 −1 2 0 1 00 0 −1 1 0 −2

Page 866: Finite Math Lecture Slides

1 0 1 0 0 10 −1 2 0 1 00 0 −1 1 0 −2

−1·→ R2

1 0 1 0 0 10 1 −2 0 −1 00 0 −1 1 0 −2

−1·→ R3

1 0 1 0 0 10 1 −2 0 −1 00 0 1 −1 0 2

−1R3+→ R1

1 0 0 1 0 −10 1 −2 0 −1 00 0 1 −1 0 2

2R3+→ R2

1 0 0 1 0 −10 1 0 −2 −1 40 0 1 −1 0 2

Page 867: Finite Math Lecture Slides

1 0 1 0 0 10 −1 2 0 1 00 0 −1 1 0 −2

−1·→ R2

1 0 1 0 0 10 1 −2 0 −1 00 0 −1 1 0 −2

−1·→ R3

1 0 1 0 0 10 1 −2 0 −1 00 0 1 −1 0 2

−1R3+→ R1

1 0 0 1 0 −10 1 −2 0 −1 00 0 1 −1 0 2

2R3+→ R2

1 0 0 1 0 −10 1 0 −2 −1 40 0 1 −1 0 2

Page 868: Finite Math Lecture Slides

1 0 1 0 0 10 −1 2 0 1 00 0 −1 1 0 −2

−1·→ R2

1 0 1 0 0 10 1 −2 0 −1 00 0 −1 1 0 −2

−1·→ R3

1 0 1 0 0 10 1 −2 0 −1 00 0 1 −1 0 2

−1R3+→ R1

1 0 0 1 0 −10 1 −2 0 −1 00 0 1 −1 0 2

2R3+→ R2

1 0 0 1 0 −10 1 0 −2 −1 40 0 1 −1 0 2

Page 869: Finite Math Lecture Slides

1 0 1 0 0 10 −1 2 0 1 00 0 −1 1 0 −2

−1·→ R2

1 0 1 0 0 10 1 −2 0 −1 00 0 −1 1 0 −2

−1·→ R3

1 0 1 0 0 10 1 −2 0 −1 00 0 1 −1 0 2

−1R3+→ R1

1 0 0 1 0 −10 1 −2 0 −1 00 0 1 −1 0 2

2R3+→ R2

1 0 0 1 0 −10 1 0 −2 −1 40 0 1 −1 0 2

Page 870: Finite Math Lecture Slides

1 0 1 0 0 10 −1 2 0 1 00 0 −1 1 0 −2

−1·→ R2

1 0 1 0 0 10 1 −2 0 −1 00 0 −1 1 0 −2

−1·→ R3

1 0 1 0 0 10 1 −2 0 −1 00 0 1 −1 0 2

−1R3+→ R1

1 0 0 1 0 −10 1 −2 0 −1 00 0 1 −1 0 2

2R3+→ R2

1 0 0 1 0 −10 1 0 −2 −1 40 0 1 −1 0 2

Page 871: Finite Math Lecture Slides

So the inverse is 1 0 −1−2 −1 4−1 0 2

.Now we can solve the matrix equation. 1 0 −1−2 −1 4−1 0 2

2 0 10 −1 21 0 1

xyz

=

1 0 −1−2 −1 4−1 0 2

2−1

3

xyz

=

−194

So the solution to the original system is

x = −1y = 9

z = 4.

Page 872: Finite Math Lecture Slides

So the inverse is 1 0 −1−2 −1 4−1 0 2

.Now we can solve the matrix equation. 1 0 −1−2 −1 4−1 0 2

2 0 10 −1 21 0 1

xyz

=

1 0 −1−2 −1 4−1 0 2

2−1

3

x

yz

=

−194

So the solution to the original system is

x = −1y = 9

z = 4.

Page 873: Finite Math Lecture Slides

So the inverse is 1 0 −1−2 −1 4−1 0 2

.Now we can solve the matrix equation. 1 0 −1−2 −1 4−1 0 2

2 0 10 −1 21 0 1

xyz

=

1 0 −1−2 −1 4−1 0 2

2−1

3

x

yz

=

−194

So the solution to the original system is

x = −1y = 9

z = 4.

Page 874: Finite Math Lecture Slides

Example: Solve the following system.

2x + z = 3−y + 2z = 2

x + z = 1.

Solution: First translate the system into matrix equation form. 2 0 10 −1 21 0 1

xyz

=

321

Now reuse the inverse from the last example.

Page 875: Finite Math Lecture Slides

Example: Solve the following system.

2x + z = 3−y + 2z = 2

x + z = 1.

Solution: First translate the system into matrix equation form. 2 0 10 −1 21 0 1

xyz

=

321

Now reuse the inverse from the last example.

Page 876: Finite Math Lecture Slides

Example: Solve the following system.

2x + z = 3−y + 2z = 2

x + z = 1.

Solution: First translate the system into matrix equation form. 2 0 10 −1 21 0 1

xyz

=

321

Now reuse the inverse from the last example.

Page 877: Finite Math Lecture Slides

1 0 −1−2 −1 4−1 0 2

2 0 10 −1 21 0 1

xyz

=

1 0 −1−2 −1 4−1 0 2

321

xyz

=

2−4−1

So the solution to the system is

x = 2y = −4

z = −1.

Page 878: Finite Math Lecture Slides

1 0 −1−2 −1 4−1 0 2

2 0 10 −1 21 0 1

xyz

=

1 0 −1−2 −1 4−1 0 2

321

x

yz

=

2−4−1

So the solution to the system is

x = 2y = −4

z = −1.

Page 879: Finite Math Lecture Slides

1 0 −1−2 −1 4−1 0 2

2 0 10 −1 21 0 1

xyz

=

1 0 −1−2 −1 4−1 0 2

321

x

yz

=

2−4−1

So the solution to the system is

x = 2y = −4

z = −1.

Page 880: Finite Math Lecture Slides

Augmented matrix versus Matrix Equation

A system can be solved either by

1. converting it to an augmented matrix and row reducing; or

2. converting it to a matrix equation and using a matrix inverse.

Method 1 is usually easier, and it’s more general:

it even handles cases where there are infinitely many solutions.

Method 2 only makes sense if you’ll reuse the inverse.

Page 881: Finite Math Lecture Slides

Augmented matrix versus Matrix Equation

A system can be solved either by

1. converting it to an augmented matrix and row reducing; or

2. converting it to a matrix equation and using a matrix inverse.

Method 1 is usually easier, and it’s more general:

it even handles cases where there are infinitely many solutions.

Method 2 only makes sense if you’ll reuse the inverse.

Page 882: Finite Math Lecture Slides

Augmented matrix versus Matrix Equation

A system can be solved either by

1. converting it to an augmented matrix and row reducing; or

2. converting it to a matrix equation and using a matrix inverse.

Method 1 is usually easier, and it’s more general:

it even handles cases where there are infinitely many solutions.

Method 2 only makes sense if you’ll reuse the inverse.

Page 883: Finite Math Lecture Slides

Recipes

Example: Suppose that a recipe for a dozen biscuits calls for 4cups of flour and 2 eggs; a recipe for a dozen pancakes calls for 5cups of flour and 1 egg. Now suppose you need to make 2 dozenbiscuits and 3 dozen pancakes. How much flour and how manyeggs do you need?

Solution: Define

R1 =

[42

]R2 =

[51

].

Then the flour and eggs required are given by

2R1 + 3R2 = 2

[42

]+ 3

[51

].

This can also be expressed as[4 52 1

] [23

].

Page 884: Finite Math Lecture Slides

RecipesExample: Suppose that a recipe for a dozen biscuits calls for 4cups of flour and 2 eggs; a recipe for a dozen pancakes calls for 5cups of flour and 1 egg. Now suppose you need to make 2 dozenbiscuits and 3 dozen pancakes. How much flour and how manyeggs do you need?

Solution: Define

R1 =

[42

]R2 =

[51

].

Then the flour and eggs required are given by

2R1 + 3R2 = 2

[42

]+ 3

[51

].

This can also be expressed as[4 52 1

] [23

].

Page 885: Finite Math Lecture Slides

RecipesExample: Suppose that a recipe for a dozen biscuits calls for 4cups of flour and 2 eggs; a recipe for a dozen pancakes calls for 5cups of flour and 1 egg. Now suppose you need to make 2 dozenbiscuits and 3 dozen pancakes. How much flour and how manyeggs do you need?

Solution: Define

R1 =

[42

]R2 =

[51

].

Then the flour and eggs required are given by

2R1 + 3R2 = 2

[42

]+ 3

[51

].

This can also be expressed as[4 52 1

] [23

].

Page 886: Finite Math Lecture Slides

RecipesExample: Suppose that a recipe for a dozen biscuits calls for 4cups of flour and 2 eggs; a recipe for a dozen pancakes calls for 5cups of flour and 1 egg. Now suppose you need to make 2 dozenbiscuits and 3 dozen pancakes. How much flour and how manyeggs do you need?

Solution: Define

R1 =

[42

]R2 =

[51

].

Then the flour and eggs required are given by

2R1 + 3R2 = 2

[42

]+ 3

[51

].

This can also be expressed as[4 52 1

] [23

].

Page 887: Finite Math Lecture Slides

RecipesExample: Suppose that a recipe for a dozen biscuits calls for 4cups of flour and 2 eggs; a recipe for a dozen pancakes calls for 5cups of flour and 1 egg. Now suppose you need to make 2 dozenbiscuits and 3 dozen pancakes. How much flour and how manyeggs do you need?

Solution: Define

R1 =

[42

]R2 =

[51

].

Then the flour and eggs required are given by

2R1 + 3R2 = 2

[42

]+ 3

[51

].

This can also be expressed as[4 52 1

] [23

].

Page 888: Finite Math Lecture Slides

A =

[4 52 1

]is called the production matrix.

X =

[23

]is called the production schedule.

R denotes the matrix of required flour and eggs, the resource vector.

The fundamental equation relating these is

R = AX .

In our case,

R = AX =

[4 52 1

] [23

]=

[23

7

].

This is called a linear production model.

Page 889: Finite Math Lecture Slides

A =

[4 52 1

]is called the production matrix.

X =

[23

]is called the production schedule.

R denotes the matrix of required flour and eggs, the resource vector.

The fundamental equation relating these is

R = AX .

In our case,

R = AX =

[4 52 1

] [23

]=

[23

7

].

This is called a linear production model.

Page 890: Finite Math Lecture Slides

A =

[4 52 1

]is called the production matrix.

X =

[23

]is called the production schedule.

R denotes the matrix of required flour and eggs, the resource vector.

The fundamental equation relating these is

R = AX .

In our case,

R = AX =

[4 52 1

] [23

]=

[23

7

].

This is called a linear production model.

Page 891: Finite Math Lecture Slides

Note in our problem

R =

[r1r2

]where

r1 = flour in cupsr2 = eggs

X =

[x1

x2

]where

x1 = biscuits in dozensx2 = pancakes in dozens.

What would be different if biscuits and pancakes were countedindividually?

Page 892: Finite Math Lecture Slides

Note in our problem

R =

[r1r2

]where

r1 = flour in cupsr2 = eggs

X =

[x1

x2

]where

x1 = biscuits in dozensx2 = pancakes in dozens.

What would be different if biscuits and pancakes were countedindividually?

Page 893: Finite Math Lecture Slides

R =

[r1r2

]where

r1 = flour in cupsr2 = eggs

X =

[x1

x2

]where

x1 = # biscuitsx2 = # pancakes

A =

[4/12

5/122/12

1/12

]=

[1/3

5/121/6

1/12

]

What if eggs were in dozens and flour in kilograms?(Assume 1 cup flour is 125 grams.)

Page 894: Finite Math Lecture Slides

R =

[r1r2

]where

r1 = flour in cupsr2 = eggs

X =

[x1

x2

]where

x1 = # biscuitsx2 = # pancakes

A =

[4/12

5/122/12

1/12

]=

[1/3

5/121/6

1/12

]

What if eggs were in dozens and flour in kilograms?(Assume 1 cup flour is 125 grams.)

Page 895: Finite Math Lecture Slides

R =

[r1r2

]where

r1 = flour in kg.r2 = eggs in dozens

X =

[x1

x2

]where

x1 = # biscuitsx2 = # pancakes

A =

[.125 · 1/3 .125 · 5/121/12 · 1/6 1/12 · 1/12

]=

[.042 .0521/72

1/144

]

Page 896: Finite Math Lecture Slides

Example: Solve for X in 1 23 45 6

[ x1

x2

]=

102437

.

Solution: Row reducing 1 2 103 4 245 6 37

yields 1 0 0

0 1 00 0 1

which translates into

x1 = 0x2 = 00 = 1

which has no solution.

Page 897: Finite Math Lecture Slides

Example: Solve for X in 1 23 45 6

[ x1

x2

]=

102437

.Solution: Row reducing 1 2 10

3 4 245 6 37

yields 1 0 0

0 1 00 0 1

which translates into

x1 = 0x2 = 00 = 1

which has no solution.

Page 898: Finite Math Lecture Slides

Example: Solve for X in 1 23 45 6

[ x1

x2

]=

102437

.Solution: Row reducing 1 2 10

3 4 245 6 37

yields 1 0 0

0 1 00 0 1

which translates into

x1 = 0x2 = 00 = 1

which has no solution.

Page 899: Finite Math Lecture Slides

Example: Solve for X in 1 23 45 6

[ x1

x2

]=

102438

.

Solution: Row reduction yields

1 0 40 1 30 0 0

,which translates into

x1 = 4x2 = 30 = 0.

Page 900: Finite Math Lecture Slides

Example: Solve for X in 1 23 45 6

[ x1

x2

]=

102438

.Solution: Row reduction yields

1 0 40 1 30 0 0

,

which translates into

x1 = 4x2 = 30 = 0.

Page 901: Finite Math Lecture Slides

Example: Solve for X in 1 23 45 6

[ x1

x2

]=

102438

.Solution: Row reduction yields

1 0 40 1 30 0 0

,which translates into

x1 = 4x2 = 30 = 0.

Page 902: Finite Math Lecture Slides

Example: Solve for X in[1 −2−2 4

] [x1

x2

]=

[3−6

].

Solution: Row reducing [1 −2 3−2 4 −6

]yields [

1 −2 30 0 0

],

which translates into

1x1 − 2x2 = 30 + 0 = 0.

Solving the first equation for x1 gives

x1 = 2x2 + 3

and we interpret x2 as a free parameter.

Page 903: Finite Math Lecture Slides

Example: Solve for X in[1 −2−2 4

] [x1

x2

]=

[3−6

].

Solution: Row reducing [1 −2 3−2 4 −6

]yields [

1 −2 30 0 0

],

which translates into

1x1 − 2x2 = 30 + 0 = 0.

Solving the first equation for x1 gives

x1 = 2x2 + 3

and we interpret x2 as a free parameter.

Page 904: Finite Math Lecture Slides

Example: Solve for X in[1 −2−2 4

] [x1

x2

]=

[3−6

].

Solution: Row reducing [1 −2 3−2 4 −6

]yields [

1 −2 30 0 0

],

which translates into

1x1 − 2x2 = 30 + 0 = 0.

Solving the first equation for x1 gives

x1 = 2x2 + 3

and we interpret x2 as a free parameter.

Page 905: Finite Math Lecture Slides

Example: Solve for X in[1 −2−2 4

] [x1

x2

]=

[3−6

].

Solution: Row reducing [1 −2 3−2 4 −6

]yields [

1 −2 30 0 0

],

which translates into

1x1 − 2x2 = 30 + 0 = 0.

Solving the first equation for x1 gives

x1 = 2x2 + 3

and we interpret x2 as a free parameter.

Page 906: Finite Math Lecture Slides

x1 = 2x2 + 3x2 = free

A free parameter is a variable that can take any value.

For example, suppose x2 = −17.

Then x1 = 2(−17) + 3 = −31.

And that is a valid solution to the initial problem:

1x1 − 2x2 = 3−2x1 + 4x2 = −6

1 · (−31) − 2 · (−17) = 3−2 · (−31) + 4 · (−17) = −6.

Page 907: Finite Math Lecture Slides

x1 = 2x2 + 3x2 = free

A free parameter is a variable that can take any value.

For example, suppose x2 = −17.

Then x1 = 2(−17) + 3 = −31.

And that is a valid solution to the initial problem:

1x1 − 2x2 = 3−2x1 + 4x2 = −6

1 · (−31) − 2 · (−17) = 3−2 · (−31) + 4 · (−17) = −6.

Page 908: Finite Math Lecture Slides

x1 = 2x2 + 3x2 = free

A free parameter is a variable that can take any value.

For example, suppose x2 = −17.

Then x1 = 2(−17) + 3 = −31.

And that is a valid solution to the initial problem:

1x1 − 2x2 = 3−2x1 + 4x2 = −6

1 · (−31) − 2 · (−17) = 3−2 · (−31) + 4 · (−17) = −6.

Page 909: Finite Math Lecture Slides

Example: Solve for X in[1 −2−2 4

] [x1

x2

]=

[36

].

Solution: Row reducing [1 −2 3−2 4 6

]yields [

1 −2 00 0 1

],

which translates into

1x1 − 2x2 = 00 + 0 = 1,

which has no solution.

Page 910: Finite Math Lecture Slides

Example: Solve for X in[1 −2−2 4

] [x1

x2

]=

[36

].

Solution: Row reducing [1 −2 3−2 4 6

]yields [

1 −2 00 0 1

],

which translates into

1x1 − 2x2 = 00 + 0 = 1,

which has no solution.

Page 911: Finite Math Lecture Slides

Example: Solve for X in[1 −2−2 4

] [x1

x2

]=

[36

].

Solution: Row reducing [1 −2 3−2 4 6

]yields [

1 −2 00 0 1

],

which translates into

1x1 − 2x2 = 00 + 0 = 1,

which has no solution.

Page 912: Finite Math Lecture Slides

Example: Solve for X in[1 −2−2 4

] [x1

x2

]=

[36

].

Solution: Row reducing [1 −2 3−2 4 6

]yields [

1 −2 00 0 1

],

which translates into

1x1 − 2x2 = 00 + 0 = 1,

which has no solution.

Page 913: Finite Math Lecture Slides

Example: Solve for X in

[1 2 34 5 6

] x1

x2

x3

=

[2124

].

Solution: Row reducing [1 2 3 214 5 6 24

]yields [

1 0 −1 −190 1 2 20

],

which translates into

x1 − x3 = −19x2 + 2x3 = 20.

Page 914: Finite Math Lecture Slides

Example: Solve for X in

[1 2 34 5 6

] x1

x2

x3

=

[2124

].

Solution: Row reducing [1 2 3 214 5 6 24

]yields [

1 0 −1 −190 1 2 20

],

which translates into

x1 − x3 = −19x2 + 2x3 = 20.

Page 915: Finite Math Lecture Slides

Example: Solve for X in

[1 2 34 5 6

] x1

x2

x3

=

[2124

].

Solution: Row reducing [1 2 3 214 5 6 24

]yields [

1 0 −1 −190 1 2 20

],

which translates into

x1 − x3 = −19x2 + 2x3 = 20.

Page 916: Finite Math Lecture Slides

x1 − x3 = −19x2 + 2x3 = 20.

Here we will solve for x1 and x2

x1 = x3 − 19x2 = −2x3 + 20

and interpret x3 as a free variable.

For example, if x3 = 5 then

x1 = 5− 19 = −14x2 = −2(5) + 20 = 10x3 = 5

is a solution.

Page 917: Finite Math Lecture Slides

x1 − x3 = −19x2 + 2x3 = 20.

Here we will solve for x1 and x2

x1 = x3 − 19x2 = −2x3 + 20

and interpret x3 as a free variable.

For example, if x3 = 5 then

x1 = 5− 19 = −14x2 = −2(5) + 20 = 10x3 = 5

is a solution.

Page 918: Finite Math Lecture Slides

Number of Solutions

General Observations:

1. zeros on left & nonzero on right =⇒ no solution; otherwise

2. #rows = #variables =⇒ unique solution;

3. #rows < #variables =⇒ infinitely many solutions.

Page 919: Finite Math Lecture Slides

Number of Solutions

General Observations:

1. zeros on left & nonzero on right =⇒ no solution; otherwise

2. #rows = #variables =⇒ unique solution;

3. #rows < #variables =⇒ infinitely many solutions.

Page 920: Finite Math Lecture Slides

Number of Solutions

General Observations:

1. zeros on left & nonzero on right =⇒ no solution; otherwise

2. #rows = #variables =⇒ unique solution;

3. #rows < #variables =⇒ infinitely many solutions.

Page 921: Finite Math Lecture Slides

Number of Solutions

General Observations:

1. zeros on left & nonzero on right =⇒ no solution; otherwise

2. #rows = #variables =⇒ unique solution;

3. #rows < #variables =⇒ infinitely many solutions.

Page 922: Finite Math Lecture Slides

Row reduction produces reduced row echelon form. 1 0 40 1 30 0 0

.RREF is the closest to I.

RREF satisfies the following.

1. Each row has a leading 1;

2. each column with a leading 1 has 0’s elsewhere;

3. the leading 1’s form a staircase (no zigzags).

Page 923: Finite Math Lecture Slides

Row reduction produces reduced row echelon form. 1 0 40 1 30 0 0

.RREF is the closest to I.

RREF satisfies the following.

1. Each row has a leading 1;

2. each column with a leading 1 has 0’s elsewhere;

3. the leading 1’s form a staircase (no zigzags).

Page 924: Finite Math Lecture Slides

Row reduction produces reduced row echelon form. 1 0 40 1 30 0 0

.RREF is the closest to I.

RREF satisfies the following.

1. Each row has a leading 1;

2. each column with a leading 1 has 0’s elsewhere;

3. the leading 1’s form a staircase (no zigzags).

Page 925: Finite Math Lecture Slides

Row reduction produces reduced row echelon form. 1 0 40 1 30 0 0

.RREF is the closest to I.

RREF satisfies the following.

1. Each row has a leading 1;

2. each column with a leading 1 has 0’s elsewhere;

3. the leading 1’s form a staircase (no zigzags).

Page 926: Finite Math Lecture Slides

Leontiev ModelsNow we apply linear production models to self-sustainingeconomies.

Think of an Amish oat farmer who plows with horses.

He uses horse manure to fertilize his oat fields,and oats to feed his horses.

Suppose that it takes 1/10 ton of oats and half a ton of manure togrow one ton of oats.And it takes one ton of oats to produce a ton of manure.

Then ifx1 = # tons of oatsx2 = # tons of manure

we get

A =

[0.1 10.5 0

].

Page 927: Finite Math Lecture Slides

Leontiev ModelsNow we apply linear production models to self-sustainingeconomies.

Think of an Amish oat farmer who plows with horses.

He uses horse manure to fertilize his oat fields,and oats to feed his horses.

Suppose that it takes 1/10 ton of oats and half a ton of manure togrow one ton of oats.And it takes one ton of oats to produce a ton of manure.

Then ifx1 = # tons of oatsx2 = # tons of manure

we get

A =

[0.1 10.5 0

].

Page 928: Finite Math Lecture Slides

Leontiev ModelsNow we apply linear production models to self-sustainingeconomies.

Think of an Amish oat farmer who plows with horses.

He uses horse manure to fertilize his oat fields,and oats to feed his horses.

Suppose that it takes 1/10 ton of oats and half a ton of manure togrow one ton of oats.

And it takes one ton of oats to produce a ton of manure.

Then ifx1 = # tons of oatsx2 = # tons of manure

we get

A =

[0.1 10.5 0

].

Page 929: Finite Math Lecture Slides

Leontiev ModelsNow we apply linear production models to self-sustainingeconomies.

Think of an Amish oat farmer who plows with horses.

He uses horse manure to fertilize his oat fields,and oats to feed his horses.

Suppose that it takes 1/10 ton of oats and half a ton of manure togrow one ton of oats.And it takes one ton of oats to produce a ton of manure.

Then ifx1 = # tons of oatsx2 = # tons of manure

we get

A =

[0.1 10.5 0

].

Page 930: Finite Math Lecture Slides

x1 = # tons of oatsx2 = # tons of manure

OutputsOats Manure

Inp

uts Oats 0.1 1Manure 0.5 0

A =

[0.1 10.5 0

]

Page 931: Finite Math Lecture Slides

So if

X =

[x1x2

]then the resources consumed are

R = AX =

[0.1 10.5 0

] [x1x2

].

Now the farmer would like to sell what he can:

excess oats to Quaker,excess manure to other farmers.

Page 932: Finite Math Lecture Slides

So if

X =

[x1x2

]then the resources consumed are

R = AX =

[0.1 10.5 0

] [x1x2

].

Now the farmer would like to sell what he can:

excess oats to Quaker,excess manure to other farmers.

Page 933: Finite Math Lecture Slides

How much does he have at the end of the harvestthat doesn’t need himself for next year?

D = X − AX

D is called the demand vectorand denotes the excess of each product that can be soldto meet external Demand.

The equation above is the fundamental one for Leontiev theory.

Page 934: Finite Math Lecture Slides

How much does he have at the end of the harvestthat doesn’t need himself for next year?

D = X − AX

D is called the demand vectorand denotes the excess of each product that can be soldto meet external Demand.

The equation above is the fundamental one for Leontiev theory.

Page 935: Finite Math Lecture Slides

How much does he have at the end of the harvestthat doesn’t need himself for next year?

D = X − AX

D is called the demand vectorand denotes the excess of each product that can be soldto meet external Demand.

The equation above is the fundamental one for Leontiev theory.

Page 936: Finite Math Lecture Slides

The Fundamental Equation of Leontiev Theory

This equation can be written in three different ways,and each has its own uses.

D = X − AX (1)

D = (I− A)X (2)(I− A)−1 = X (3)

1. Form (1) is easy to understand;

2. form (2) can be solved for X using row reduction;

3. form (3) has been solved using a matrix inverse.

Page 937: Finite Math Lecture Slides

The Fundamental Equation of Leontiev Theory

This equation can be written in three different ways,and each has its own uses.

D = X − AX (1)D = (I− A)X (2)

(I− A)−1 = X (3)

1. Form (1) is easy to understand;

2. form (2) can be solved for X using row reduction;

3. form (3) has been solved using a matrix inverse.

Page 938: Finite Math Lecture Slides

The Fundamental Equation of Leontiev Theory

This equation can be written in three different ways,and each has its own uses.

D = X − AX (1)D = (I− A)X (2)

(I− A)−1 = X (3)

1. Form (1) is easy to understand;

2. form (2) can be solved for X using row reduction;

3. form (3) has been solved using a matrix inverse.

Page 939: Finite Math Lecture Slides

The Fundamental Equation of Leontiev Theory

This equation can be written in three different ways,and each has its own uses.

D = X − AX (1)D = (I− A)X (2)

(I− A)−1 = X (3)

1. Form (1) is easy to understand;

2. form (2) can be solved for X using row reduction;

3. form (3) has been solved using a matrix inverse.

Page 940: Finite Math Lecture Slides

Back to the Amish oat farmer:

I− A =

[1 00 1

]−[

0.1 10.5 0

]=

[0.9 −1−0.5 1

].

Suppose the farmer is currently growing 100 tons of oats and“producing” 60 tons of horse manure. How much will he be ableto sell?

X =

[100

60

],

D = (I− A)X

=

[0.9 −1−0.5 1

] [100

60

]

=

[3010

]

Page 941: Finite Math Lecture Slides

Back to the Amish oat farmer:

I− A =

[1 00 1

]−[

0.1 10.5 0

]=

[0.9 −1−0.5 1

].

Suppose the farmer is currently growing 100 tons of oats and“producing” 60 tons of horse manure. How much will he be ableto sell?

X =

[100

60

],

D = (I− A)X

=

[0.9 −1−0.5 1

] [100

60

]

=

[3010

]

Page 942: Finite Math Lecture Slides

Back to the Amish oat farmer:

I− A =

[1 00 1

]−[

0.1 10.5 0

]=

[0.9 −1−0.5 1

].

Suppose the farmer is currently growing 100 tons of oats and“producing” 60 tons of horse manure. How much will he be ableto sell?

X =

[100

60

],

D = (I− A)X

=

[0.9 −1−0.5 1

] [100

60

]

=

[3010

]

Page 943: Finite Math Lecture Slides

Back to the Amish oat farmer:

I− A =

[1 00 1

]−[

0.1 10.5 0

]=

[0.9 −1−0.5 1

].

Suppose the farmer is currently growing 100 tons of oats and“producing” 60 tons of horse manure. How much will he be ableto sell?

X =

[100

60

],

D = (I− A)X

=

[0.9 −1−0.5 1

] [100

60

]

=

[3010

]

Page 944: Finite Math Lecture Slides

Back to the Amish oat farmer:

I− A =

[1 00 1

]−[

0.1 10.5 0

]=

[0.9 −1−0.5 1

].

Suppose the farmer is currently growing 100 tons of oats and“producing” 60 tons of horse manure. How much will he be ableto sell?

X =

[100

60

],

D = (I− A)X

=

[0.9 −1−0.5 1

] [100

60

]

=

[3010

]

Page 945: Finite Math Lecture Slides

Harder Question: Suppose the farmer wants to be able to sell 50tons of oats and 15 tons of manure. How much of each should heproduce?

Solution: Here D is given but X is the unknown. We can use eitherform (2) or form (3). Here I use form (2).

D =

[5015

],

D = (I− A)X[5015

]=

[0.9 −1−0.5 1

]X

This translates into the augmented matrix[0.9 −1 50−0.5 1 15

].

Page 946: Finite Math Lecture Slides

Harder Question: Suppose the farmer wants to be able to sell 50tons of oats and 15 tons of manure. How much of each should heproduce?

Solution: Here D is given but X is the unknown. We can use eitherform (2) or form (3). Here I use form (2).

D =

[5015

],

D = (I− A)X[5015

]=

[0.9 −1−0.5 1

]X

This translates into the augmented matrix[0.9 −1 50−0.5 1 15

].

Page 947: Finite Math Lecture Slides

Harder Question: Suppose the farmer wants to be able to sell 50tons of oats and 15 tons of manure. How much of each should heproduce?

Solution: Here D is given but X is the unknown. We can use eitherform (2) or form (3). Here I use form (2).

D =

[5015

],

D = (I− A)X[5015

]=

[0.9 −1−0.5 1

]X

This translates into the augmented matrix[0.9 −1 50−0.5 1 15

].

Page 948: Finite Math Lecture Slides

Harder Question: Suppose the farmer wants to be able to sell 50tons of oats and 15 tons of manure. How much of each should heproduce?

Solution: Here D is given but X is the unknown. We can use eitherform (2) or form (3). Here I use form (2).

D =

[5015

],

D = (I− A)X[5015

]=

[0.9 −1−0.5 1

]X

This translates into the augmented matrix[0.9 −1 50−0.5 1 15

].

Page 949: Finite Math Lecture Slides

[0.9 −1 50−0.5 1 15

].

Now row reduce.

R1 ↔ R2 :

[−0.5 1 15

0.9 −1 50

]

−2·→ R1 :

[1 −2 −30

0.9 −1 50

]

−0.9R1+→ R2 :

[1 −2 −300 0.8 77

]

5/4·→ R2 :

[1 −2 −300 1 385/4

]

2R2+→ R1 :

[1 0 445/20 1 385/4

]

Page 950: Finite Math Lecture Slides

[0.9 −1 50−0.5 1 15

].

Now row reduce.

R1 ↔ R2 :

[−0.5 1 15

0.9 −1 50

]

−2·→ R1 :

[1 −2 −30

0.9 −1 50

]

−0.9R1+→ R2 :

[1 −2 −300 0.8 77

]

5/4·→ R2 :

[1 −2 −300 1 385/4

]

2R2+→ R1 :

[1 0 445/20 1 385/4

]

Page 951: Finite Math Lecture Slides

[0.9 −1 50−0.5 1 15

].

Now row reduce.

R1 ↔ R2 :

[−0.5 1 15

0.9 −1 50

]

−2·→ R1 :

[1 −2 −30

0.9 −1 50

]

−0.9R1+→ R2 :

[1 −2 −300 0.8 77

]

5/4·→ R2 :

[1 −2 −300 1 385/4

]

2R2+→ R1 :

[1 0 445/20 1 385/4

]

Page 952: Finite Math Lecture Slides

[0.9 −1 50−0.5 1 15

].

Now row reduce.

R1 ↔ R2 :

[−0.5 1 15

0.9 −1 50

]

−2·→ R1 :

[1 −2 −30

0.9 −1 50

]

−0.9R1+→ R2 :

[1 −2 −300 0.8 77

]

5/4·→ R2 :

[1 −2 −300 1 385/4

]

2R2+→ R1 :

[1 0 445/20 1 385/4

]

Page 953: Finite Math Lecture Slides

[0.9 −1 50−0.5 1 15

].

Now row reduce.

R1 ↔ R2 :

[−0.5 1 15

0.9 −1 50

]

−2·→ R1 :

[1 −2 −30

0.9 −1 50

]

−0.9R1+→ R2 :

[1 −2 −300 0.8 77

]

5/4·→ R2 :

[1 −2 −300 1 385/4

]

2R2+→ R1 :

[1 0 445/20 1 385/4

]

Page 954: Finite Math Lecture Slides

[1 0 445/20 1 385/4

]So the solution is

# tons of oats = x1 = 445/2 = 222.5

# tons of manure = x2 = 385/4 = 96.25.

Page 955: Finite Math Lecture Slides

Linear Programming

Bad news: this chapter is all about story problems.

Good news: the only math you need is high school algebra.

Bad news: solving a single problem is a 3 step process that cantake 20 minutes.

Good news: all 3 steps are easy and most do well on theseproblems.

Page 956: Finite Math Lecture Slides

Linear Programming

Bad news: this chapter is all about story problems.

Good news: the only math you need is high school algebra.

Bad news: solving a single problem is a 3 step process that cantake 20 minutes.

Good news: all 3 steps are easy and most do well on theseproblems.

Page 957: Finite Math Lecture Slides

Linear Programming

Bad news: this chapter is all about story problems.

Good news: the only math you need is high school algebra.

Bad news: solving a single problem is a 3 step process that cantake 20 minutes.

Good news: all 3 steps are easy and most do well on theseproblems.

Page 958: Finite Math Lecture Slides

Linear Programming

Bad news: this chapter is all about story problems.

Good news: the only math you need is high school algebra.

Bad news: solving a single problem is a 3 step process that cantake 20 minutes.

Good news: all 3 steps are easy and most do well on theseproblems.

Page 959: Finite Math Lecture Slides

Example: Suppose you’re taking a dozen 10 year olds out to seeIce Age IV. You figure you’ll need 44 ounces of popcorn and 56ounces of soda to see them through the movie. The snack baroffers two snack combos that look interesting: the Gulpalicious,which offers a 12 oz. popcorn with an 8 oz. soda for $8, and theSlurpalicious, which offers an 8 oz. popcorn and a 12 oz. soda for$7. How many of each combo should you buy in order to get allthe popcorn and soda you need at the lowest price?

Strategy: There’s a lot of info here.

First identify exactly what the question is asking for.

Page 960: Finite Math Lecture Slides

Example: Suppose you’re taking a dozen 10 year olds out to seeIce Age IV. You figure you’ll need 44 ounces of popcorn and 56ounces of soda to see them through the movie. The snack baroffers two snack combos that look interesting: the Gulpalicious,which offers a 12 oz. popcorn with an 8 oz. soda for $8, and theSlurpalicious, which offers an 8 oz. popcorn and a 12 oz. soda for$7. How many of each combo should you buy in order to get allthe popcorn and soda you need at the lowest price?

Strategy: There’s a lot of info here.

First identify exactly what the question is asking for.

Page 961: Finite Math Lecture Slides

Example: Suppose you’re taking a dozen 10 year olds out to seeIce Age IV. You figure you’ll need 44 ounces of popcorn and 56ounces of soda to see them through the movie. The snack baroffers two snack combos that look interesting: the Gulpalicious,which offers a 12 oz. popcorn with an 8 oz. soda for $8, and theSlurpalicious, which offers an 8 oz. popcorn and a 12 oz. soda for$7. How many of each combo should you buy in order to get allthe popcorn and soda you need at the lowest price?

Strategy: So the answer should be two numbers:

x = # Gulpaliciousesy = # Slurpaliciouses.

Page 962: Finite Math Lecture Slides

Example: Suppose you’re taking a dozen 10 year olds out to seeIce Age IV. You figure you’ll need 44 ounces of popcorn and 56ounces of soda to see them through the movie. The snack baroffers two snack combos that look interesting: the Gulpalicious,which offers a 12 oz. popcorn with an 8 oz. soda for $8, and theSlurpalicious, which offers an 8 oz. popcorn and a 12 oz. soda for$7. How many of each combo should you buy in order to get allthe popcorn and soda you need at the lowest price?

Strategy: So the answer should be two numbers:

x = # Gulpaliciousesy = # Slurpaliciouses.

Page 963: Finite Math Lecture Slides

Example: Suppose you’re taking a dozen 10 year olds out to seeIce Age IV. You figure you’ll need 44 ounces of popcorn and 56ounces of soda to see them through the movie. The snack baroffers two snack combos that look interesting: the Gulpalicious,which offers a 12 oz. popcorn with an 8 oz. soda for $8, and theSlurpalicious, which offers an 8 oz. popcorn and a 12 oz. soda for$7. How many of each combo should you buy in order to get allthe popcorn and soda you need at the lowest price?

Strategy: In all these problems there will be some quantity you’retrying to optimize.

You’re not given a definite target for this quantity,you just want to do as well as you can.

What is that quantity here?

Page 964: Finite Math Lecture Slides

Example: Suppose you’re taking a dozen 10 year olds out to seeIce Age IV. You figure you’ll need 44 ounces of popcorn and 56ounces of soda to see them through the movie. The snack baroffers two snack combos that look interesting: the Gulpalicious,which offers a 12 oz. popcorn with an 8 oz. soda for $8, and theSlurpalicious, which offers an 8 oz. popcorn and a 12 oz. soda for$7. How many of each combo should you buy in order to get allthe popcorn and soda you need at the lowest price?

Strategy: In all these problems there will be some quantity you’retrying to optimize.

You’re not given a definite target for this quantity,you just want to do as well as you can.

What is that quantity here?

Page 965: Finite Math Lecture Slides

Example: Suppose you’re taking a dozen 10 year olds out to seeIce Age IV. You figure you’ll need 44 ounces of popcorn and 56ounces of soda to see them through the movie. The snack baroffers two snack combos that look interesting: the Gulpalicious,which offers a 12 oz. popcorn with an 8 oz. soda for $8, and theSlurpalicious, which offers an 8 oz. popcorn and a 12 oz. soda for$7. How many of each combo should you buy in order to get allthe popcorn and soda you need at the lowest price?

Strategy: In all these problems there will be some quantity you’retrying to optimize.

You’re not given a definite target for this quantity,you just want to do as well as you can.

What is that quantity here?

Page 966: Finite Math Lecture Slides

Example: Suppose you’re taking a dozen 10 year olds out to seeIce Age IV. You figure you’ll need 44 ounces of popcorn and 56ounces of soda to see them through the movie. The snack baroffers two snack combos that look interesting: the Gulpalicious,which offers a 12 oz. popcorn with an 8 oz. soda for $8, and theSlurpalicious, which offers an 8 oz. popcorn and a 12 oz. soda for$7. How many of each combo should you buy in order to get allthe popcorn and soda you need at the lowest price?

Strategy: Here we want the lowest price possible.

This is called the objective function,because it’s actual value depends on x and y .

Here the objective is

minimize 8x + 7y .

Page 967: Finite Math Lecture Slides

Example: Suppose you’re taking a dozen 10 year olds out to seeIce Age IV. You figure you’ll need 44 ounces of popcorn and 56ounces of soda to see them through the movie. The snack baroffers two snack combos that look interesting: the Gulpalicious,which offers a 12 oz. popcorn with an 8 oz. soda for $8, and theSlurpalicious, which offers an 8 oz. popcorn and a 12 oz. soda for$7. How many of each combo should you buy in order to get allthe popcorn and soda you need at the lowest price?

Strategy: Here we want the lowest price possible.

This is called the objective function,because it’s actual value depends on x and y .

Here the objective is

minimize 8x + 7y .

Page 968: Finite Math Lecture Slides

Example: Suppose you’re taking a dozen 10 year olds out to seeIce Age IV. You figure you’ll need 44 ounces of popcorn and 56ounces of soda to see them through the movie. The snack baroffers two snack combos that look interesting: the Gulpalicious,which offers a 12 oz. popcorn with an 8 oz. soda for $8, and theSlurpalicious, which offers an 8 oz. popcorn and a 12 oz. soda for$7. How many of each combo should you buy in order to get allthe popcorn and soda you need at the lowest price?

Strategy: Here we want the lowest price possible.

This is called the objective function,because it’s actual value depends on x and y .

Here the objective is

minimize 8x + 7y .

Page 969: Finite Math Lecture Slides

Example: Suppose you’re taking a dozen 10 year olds out to seeIce Age IV. You figure you’ll need 44 ounces of popcorn and 56ounces of soda to see them through the movie. The snack baroffers two snack combos that look interesting: the Gulpalicious,which offers a 12 oz. popcorn with an 8 oz. soda for $8, and theSlurpalicious, which offers an 8 oz. popcorn and a 12 oz. soda for$7. How many of each combo should you buy in order to get allthe popcorn and soda you need at the lowest price?

Strategy: There are other quantities for which we do have targets.

The precise amounts of popcorn and soda depends on x and y ,but each must satisfy at least the given requirements.

# oz.s popcorn ≥ 44# oz.s soda ≥ 56

Page 970: Finite Math Lecture Slides

Example: Suppose you’re taking a dozen 10 year olds out to seeIce Age IV. You figure you’ll need 44 ounces of popcorn and 56ounces of soda to see them through the movie. The snack baroffers two snack combos that look interesting: the Gulpalicious,which offers a 12 oz. popcorn with an 8 oz. soda for $8, and theSlurpalicious, which offers an 8 oz. popcorn and a 12 oz. soda for$7. How many of each combo should you buy in order to get allthe popcorn and soda you need at the lowest price?

Strategy: There are other quantities for which we do have targets.

The precise amounts of popcorn and soda depends on x and y ,but each must satisfy at least the given requirements.

# oz.s popcorn ≥ 44# oz.s soda ≥ 56

Page 971: Finite Math Lecture Slides

Example: Suppose you’re taking a dozen 10 year olds out to seeIce Age IV. You figure you’ll need 44 ounces of popcorn and 56ounces of soda to see them through the movie. The snack baroffers two snack combos that look interesting: the Gulpalicious,which offers a 12 oz. popcorn with an 8 oz. soda for $8, and theSlurpalicious, which offers an 8 oz. popcorn and a 12 oz. soda for$7. How many of each combo should you buy in order to get allthe popcorn and soda you need at the lowest price?

Strategy: There are other quantities for which we do have targets.

The precise amounts of popcorn and soda depends on x and y ,but each must satisfy at least the given requirements.

# oz.s popcorn ≥ 44# oz.s soda ≥ 56

Page 972: Finite Math Lecture Slides

Example: Suppose you’re taking a dozen 10 year olds out to seeIce Age IV. You figure you’ll need 44 ounces of popcorn and 56ounces of soda to see them through the movie. The snack baroffers two snack combos that look interesting: the Gulpalicious,which offers a 12 oz. popcorn with an 8 oz. soda for $8, and theSlurpalicious, which offers an 8 oz. popcorn and a 12 oz. soda for$7. How many of each combo should you buy in order to get allthe popcorn and soda you need at the lowest price?

Strategy: We don’t know exactly how much of each we’ll get,but we can express the quantities in terms of x and y .

12x + 8y = # oz.s popcorn ≥ 448x + 12y = # oz.s soda ≥ 56

Page 973: Finite Math Lecture Slides

Example: Suppose you’re taking a dozen 10 year olds out to seeIce Age IV. You figure you’ll need 44 ounces of popcorn and 56ounces of soda to see them through the movie. The snack baroffers two snack combos that look interesting: the Gulpalicious,which offers a 12 oz. popcorn with an 8 oz. soda for $8, and theSlurpalicious, which offers an 8 oz. popcorn and a 12 oz. soda for$7. How many of each combo should you buy in order to get allthe popcorn and soda you need at the lowest price?

Strategy: Some people find it helpful in these “Materials &Products” problems to make a table.

Gulp. Slurp. Total

popcorn 12 oz. 8 oz. 44 oz.soda 8 oz. 12 oz. 56 oz.

Page 974: Finite Math Lecture Slides

Gulp. Slurp. Total

popcorn 12 oz. 8 oz. 44 oz.soda 8 oz. 12 oz. 56 oz.

From this table we get the inequalities

12x + 8y ≥ 448x + 12y ≥ 56.

The direction of the inequalities is becausethe numbers on the right are minimum requirements.

In other problems they may go the other way.

Page 975: Finite Math Lecture Slides

Gulp. Slurp. Total

popcorn 12 oz. 8 oz. 44 oz.soda 8 oz. 12 oz. 56 oz.

From this table we get the inequalities

12x + 8y ≥ 448x + 12y ≥ 56.

The direction of the inequalities is becausethe numbers on the right are minimum requirements.

In other problems they may go the other way.

Page 976: Finite Math Lecture Slides

Gulp. Slurp. Total

popcorn 12 oz. 8 oz. 44 oz.soda 8 oz. 12 oz. 56 oz.

From this table we get the inequalities

12x + 8y ≥ 448x + 12y ≥ 56.

The direction of the inequalities is becausethe numbers on the right are minimum requirements.

In other problems they may go the other way.

Page 977: Finite Math Lecture Slides

Example: Suppose you’re taking a dozen 10 year olds out to seeIce Age IV. You figure you’ll need 44 ounces of popcorn and 56ounces of soda to see them through the movie. The snack baroffers two snack combos that look interesting: the Gulpalicious,which offers a 12 oz. popcorn with an 8 oz. soda for $8, and theSlurpalicious, which offers an 8 oz. popcorn and a 12 oz. soda for$7. How many of each combo should you buy in order to get allthe popcorn and soda you need at the lowest price?

Strategy:

You can’t buy a negative number of snack combos.

Therefore x and y must satisfy the natural constraints:

x ≥ 0y ≥ 0.

The 4 inequalities together are called the constraints.

Page 978: Finite Math Lecture Slides

Example: Suppose you’re taking a dozen 10 year olds out to seeIce Age IV. You figure you’ll need 44 ounces of popcorn and 56ounces of soda to see them through the movie. The snack baroffers two snack combos that look interesting: the Gulpalicious,which offers a 12 oz. popcorn with an 8 oz. soda for $8, and theSlurpalicious, which offers an 8 oz. popcorn and a 12 oz. soda for$7. How many of each combo should you buy in order to get allthe popcorn and soda you need at the lowest price?

Strategy:

You can’t buy a negative number of snack combos.

Therefore x and y must satisfy the natural constraints:

x ≥ 0y ≥ 0.

The 4 inequalities together are called the constraints.

Page 979: Finite Math Lecture Slides

Putting together everything we’ve done so far, we get the following.

x = # Gulpaliciousesy = # Slurpaliciouses

We want tominimize 8x + 7y

subject to the constraints

x ≥ 0y ≥ 0

12x + 8y ≥ 448x + 12y ≥ 56.

This is called the mathematical formulation of the problem.

Page 980: Finite Math Lecture Slides

Putting together everything we’ve done so far, we get the following.

x = # Gulpaliciousesy = # Slurpaliciouses

We want tominimize 8x + 7y

subject to the constraints

x ≥ 0y ≥ 0

12x + 8y ≥ 448x + 12y ≥ 56.

This is called the mathematical formulation of the problem.

Page 981: Finite Math Lecture Slides

Formulation of a Linear Programming Problem

1. Define the variables, including the units of measure;

2. write out the objective function in terms of the variables, andspecify whether you want to maximize or minimize it;

3. write out the system of constraints, including the naturalconstraints if applicable.

Page 982: Finite Math Lecture Slides

Example: (7.1 #12) A purchasing agent for a college finds itnecessary to decide how much hard and soft chalk should bepurchased each month. He knows that a typical instructor will usetwo pieces of soft chalk or one piece of hard chalk for each class.In addition, he has observed that hard chalk always amounts to atleast one-quarter of the total used. Finally, his supplier has limitedhim to a purchase of at most 60 gross (8640 pieces) of soft chalk.There are 3600 classes to be taught each month. If soft chalk is$1.50 per gross and hard chalk is $3.50 per gross, how much ofeach should be purchased to meet the needs and to keep costs to aminimum?

Page 983: Finite Math Lecture Slides

Example: (7.1 #12) A purchasing agent for a college finds itnecessary to decide how much hard and soft chalk should bepurchased each month. He knows that a typical instructor will usetwo pieces of soft chalk or one piece of hard chalk for each class.In addition, he has observed that hard chalk always amounts to atleast one-quarter of the total used. Finally, his supplier has limitedhim to a purchase of at most 60 gross (8640 pieces) of soft chalk.There are 3600 classes to be taught each month. If soft chalk is$1.50 per gross and hard chalk is $3.50 per gross, how much ofeach should be purchased to meet the needs and to keep costs to aminimum?

Variables:x = # pieces of soft chalky = # pieces of hard chalk

Other units could be used, e.g. gross.

Page 984: Finite Math Lecture Slides

Example: (7.1 #12) A purchasing agent for a college finds itnecessary to decide how much hard and soft chalk should bepurchased each month. He knows that a typical instructor will usetwo pieces of soft chalk or one piece of hard chalk for each class.In addition, he has observed that hard chalk always amounts to atleast one-quarter of the total used. Finally, his supplier has limitedhim to a purchase of at most 60 gross (8640 pieces) of soft chalk.There are 3600 classes to be taught each month. If soft chalk is$1.50 per gross and hard chalk is $3.50 per gross, how much ofeach should be purchased to meet the needs and to keep costs to aminimum?

Objective Function: Since gross = dozen dozen = 144,

minimize Cost = 1.5( x

144

)+ 3.5

( y

144

)=

(1.5

144

)x +

(3.5

144

)y .

Page 985: Finite Math Lecture Slides

Example: (7.1 #12) A purchasing agent for a college finds itnecessary to decide how much hard and soft chalk should bepurchased each month. He knows that a typical instructor will usetwo pieces of soft chalk or one piece of hard chalk for each class.In addition, he has observed that hard chalk always amounts to atleast one-quarter of the total used. Finally, his supplier has limitedhim to a purchase of at most 60 gross (8640 pieces) of soft chalk.There are 3600 classes to be taught each month. If soft chalk is$1.50 per gross and hard chalk is $3.50 per gross, how much ofeach should be purchased to meet the needs and to keep costs to aminimum?

Constraints: The natural constraints apply here:

x ≥ 0y ≥ 0.

Page 986: Finite Math Lecture Slides

Example: (7.1 #12) A purchasing agent for a college finds itnecessary to decide how much hard and soft chalk should bepurchased each month. He knows that a typical instructor will usetwo pieces of soft chalk or one piece of hard chalk for each class.In addition, he has observed that hard chalk always amounts to atleast one-quarter of the total used. Finally, his supplier has limitedhim to a purchase of at most 60 gross (8640 pieces) of soft chalk.There are 3600 classes to be taught each month. If soft chalk is$1.50 per gross and hard chalk is $3.50 per gross, how much ofeach should be purchased to meet the needs and to keep costs to aminimum?

Constraints:

classes per piece soft chalk = 1/2classes per piece hard chalk = 1

# classes with all soft chalk = 1/2x# classes with all hard chalk = 1y

total # classes with all chalk = 1/2x + y ≥ 3600

Page 987: Finite Math Lecture Slides

Example: (7.1 #12) A purchasing agent for a college finds itnecessary to decide how much hard and soft chalk should bepurchased each month. He knows that a typical instructor will usetwo pieces of soft chalk or one piece of hard chalk for each class.In addition, he has observed that hard chalk always amounts to atleast one-quarter of the total used. Finally, his supplier has limitedhim to a purchase of at most 60 gross (8640 pieces) of soft chalk.There are 3600 classes to be taught each month. If soft chalk is$1.50 per gross and hard chalk is $3.50 per gross, how much ofeach should be purchased to meet the needs and to keep costs to aminimum?

Constraints:

classes per piece soft chalk = 1/2classes per piece hard chalk = 1

# classes with all soft chalk = 1/2x# classes with all hard chalk = 1y

total # classes with all chalk = 1/2x + y ≥ 3600

Page 988: Finite Math Lecture Slides

Example: (7.1 #12) A purchasing agent for a college finds itnecessary to decide how much hard and soft chalk should bepurchased each month. He knows that a typical instructor will usetwo pieces of soft chalk or one piece of hard chalk for each class.In addition, he has observed that hard chalk always amounts to atleast one-quarter of the total used. Finally, his supplier has limitedhim to a purchase of at most 60 gross (8640 pieces) of soft chalk.There are 3600 classes to be taught each month. If soft chalk is$1.50 per gross and hard chalk is $3.50 per gross, how much ofeach should be purchased to meet the needs and to keep costs to aminimum?

Constraints:

classes per piece soft chalk = 1/2classes per piece hard chalk = 1

# classes with all soft chalk = 1/2x# classes with all hard chalk = 1y

total # classes with all chalk = 1/2x + y ≥ 3600

Page 989: Finite Math Lecture Slides

Example: (7.1 #12) A purchasing agent for a college finds itnecessary to decide how much hard and soft chalk should bepurchased each month. He knows that a typical instructor will usetwo pieces of soft chalk or one piece of hard chalk for each class.In addition, he has observed that hard chalk always amounts to atleast one-quarter of the total used. Finally, his supplier has limitedhim to a purchase of at most 60 gross (8640 pieces) of soft chalk.There are 3600 classes to be taught each month. If soft chalk is$1.50 per gross and hard chalk is $3.50 per gross, how much ofeach should be purchased to meet the needs and to keep costs to aminimum?

Constraints:

total chalk used =

x + yone-quarter the total chalk used = 1

4(x + y)

y ≥ 14(x + y)

Page 990: Finite Math Lecture Slides

Example: (7.1 #12) A purchasing agent for a college finds itnecessary to decide how much hard and soft chalk should bepurchased each month. He knows that a typical instructor will usetwo pieces of soft chalk or one piece of hard chalk for each class.In addition, he has observed that hard chalk always amounts to atleast one-quarter of the total used. Finally, his supplier has limitedhim to a purchase of at most 60 gross (8640 pieces) of soft chalk.There are 3600 classes to be taught each month. If soft chalk is$1.50 per gross and hard chalk is $3.50 per gross, how much ofeach should be purchased to meet the needs and to keep costs to aminimum?

Constraints:

total chalk used = x + y

one-quarter the total chalk used = 14(x + y)

y ≥ 14(x + y)

Page 991: Finite Math Lecture Slides

Example: (7.1 #12) A purchasing agent for a college finds itnecessary to decide how much hard and soft chalk should bepurchased each month. He knows that a typical instructor will usetwo pieces of soft chalk or one piece of hard chalk for each class.In addition, he has observed that hard chalk always amounts to atleast one-quarter of the total used. Finally, his supplier has limitedhim to a purchase of at most 60 gross (8640 pieces) of soft chalk.There are 3600 classes to be taught each month. If soft chalk is$1.50 per gross and hard chalk is $3.50 per gross, how much ofeach should be purchased to meet the needs and to keep costs to aminimum?

Constraints:

total chalk used = x + yone-quarter the total chalk used =

14(x + y)

y ≥ 14(x + y)

Page 992: Finite Math Lecture Slides

Example: (7.1 #12) A purchasing agent for a college finds itnecessary to decide how much hard and soft chalk should bepurchased each month. He knows that a typical instructor will usetwo pieces of soft chalk or one piece of hard chalk for each class.In addition, he has observed that hard chalk always amounts to atleast one-quarter of the total used. Finally, his supplier has limitedhim to a purchase of at most 60 gross (8640 pieces) of soft chalk.There are 3600 classes to be taught each month. If soft chalk is$1.50 per gross and hard chalk is $3.50 per gross, how much ofeach should be purchased to meet the needs and to keep costs to aminimum?

Constraints:

total chalk used = x + yone-quarter the total chalk used = 1

4(x + y)

y ≥ 14(x + y)

Page 993: Finite Math Lecture Slides

Example: (7.1 #12) A purchasing agent for a college finds itnecessary to decide how much hard and soft chalk should bepurchased each month. He knows that a typical instructor will usetwo pieces of soft chalk or one piece of hard chalk for each class.In addition, he has observed that hard chalk always amounts to atleast one-quarter of the total used. Finally, his supplier has limitedhim to a purchase of at most 60 gross (8640 pieces) of soft chalk.There are 3600 classes to be taught each month. If soft chalk is$1.50 per gross and hard chalk is $3.50 per gross, how much ofeach should be purchased to meet the needs and to keep costs to aminimum?

Constraints:

total chalk used = x + yone-quarter the total chalk used = 1

4(x + y)

y ≥

14(x + y)

Page 994: Finite Math Lecture Slides

Example: (7.1 #12) A purchasing agent for a college finds itnecessary to decide how much hard and soft chalk should bepurchased each month. He knows that a typical instructor will usetwo pieces of soft chalk or one piece of hard chalk for each class.In addition, he has observed that hard chalk always amounts to atleast one-quarter of the total used. Finally, his supplier has limitedhim to a purchase of at most 60 gross (8640 pieces) of soft chalk.There are 3600 classes to be taught each month. If soft chalk is$1.50 per gross and hard chalk is $3.50 per gross, how much ofeach should be purchased to meet the needs and to keep costs to aminimum?

Constraints:

total chalk used = x + yone-quarter the total chalk used = 1

4(x + y)

y ≥ 14(x + y)

Page 995: Finite Math Lecture Slides

Example: (7.1 #12) A purchasing agent for a college finds itnecessary to decide how much hard and soft chalk should bepurchased each month. He knows that a typical instructor will usetwo pieces of soft chalk or one piece of hard chalk for each class.In addition, he has observed that hard chalk always amounts to atleast one-quarter of the total used. Finally, his supplier has limitedhim to a purchase of at most 60 gross (8640 pieces) of soft chalk.There are 3600 classes to be taught each month. If soft chalk is$1.50 per gross and hard chalk is $3.50 per gross, how much ofeach should be purchased to meet the needs and to keep costs to aminimum?

Constraints:x ≤ 8640

Page 996: Finite Math Lecture Slides

Variables:x = # pieces of soft chalky = # pieces of hard chalk

Objective Function:

minimize

(1.5

144

)x +

(3.5

144

)y

Constraints:x ≥ 0y ≥ 0

1/2x + y ≥ 3600y ≥ 1

4(x + y)x ≤ 8640

Page 997: Finite Math Lecture Slides

Graphing InequalitiesTo “graph the inequality” x + 2y ≤ 4 meansto shade the region of the plane containing the points that satisfy it.

Here is the graph of the equation x + 2y = 4.

-4

-3

-2

-1

0

1

2

3

4

5

-4 -3 -2 -1 0 1 2 3 4 5

All the points onthe line satisfythe inequality.

Page 998: Finite Math Lecture Slides

Graphing InequalitiesTo “graph the inequality” x + 2y ≤ 4 meansto shade the region of the plane containing the points that satisfy it.

Here is the graph of the equation x + 2y = 4.

-4

-3

-2

-1

0

1

2

3

4

5

-4 -3 -2 -1 0 1 2 3 4 5

All the points onthe line satisfythe inequality.

Page 999: Finite Math Lecture Slides

What other points satisfy x + 2y ≤ 4?

Let’s try out a few.

(2, 3) : 2 + 2(3) ≤ 4 is false, so no.

(2,−3) : 2 + 2(−3) ≤ 4 is true, so yes.(−3, 2) : −3 + 2(2) ≤ 4 is true, so yes.(−1, 4) : −1 + 2(4) ≤ 4 is false, so no.

(−3,−2) : −3 + 2(−2) ≤ 4 is true, so yes.(1, 1) : 1 + 2(1) ≤ 4 is true, so yes.

-4

-3

-2

-1

0

1

2

3

4

5

-4 -3 -2 -1 0 1 2 3 4 5

There’s a pattern here.

Page 1000: Finite Math Lecture Slides

What other points satisfy x + 2y ≤ 4?

Let’s try out a few.

(2, 3) : 2 + 2(3) ≤ 4 is false, so no.(2,−3) : 2 + 2(−3) ≤ 4 is true, so yes.

(−3, 2) : −3 + 2(2) ≤ 4 is true, so yes.(−1, 4) : −1 + 2(4) ≤ 4 is false, so no.

(−3,−2) : −3 + 2(−2) ≤ 4 is true, so yes.(1, 1) : 1 + 2(1) ≤ 4 is true, so yes.

-4

-3

-2

-1

0

1

2

3

4

5

-4 -3 -2 -1 0 1 2 3 4 5

There’s a pattern here.

Page 1001: Finite Math Lecture Slides

What other points satisfy x + 2y ≤ 4?

Let’s try out a few.

(2, 3) : 2 + 2(3) ≤ 4 is false, so no.(2,−3) : 2 + 2(−3) ≤ 4 is true, so yes.(−3, 2) : −3 + 2(2) ≤ 4 is true, so yes.

(−1, 4) : −1 + 2(4) ≤ 4 is false, so no.(−3,−2) : −3 + 2(−2) ≤ 4 is true, so yes.

(1, 1) : 1 + 2(1) ≤ 4 is true, so yes.

-4

-3

-2

-1

0

1

2

3

4

5

-4 -3 -2 -1 0 1 2 3 4 5

There’s a pattern here.

Page 1002: Finite Math Lecture Slides

What other points satisfy x + 2y ≤ 4?

Let’s try out a few.

(2, 3) : 2 + 2(3) ≤ 4 is false, so no.(2,−3) : 2 + 2(−3) ≤ 4 is true, so yes.(−3, 2) : −3 + 2(2) ≤ 4 is true, so yes.(−1, 4) : −1 + 2(4) ≤ 4 is false, so no.

(−3,−2) : −3 + 2(−2) ≤ 4 is true, so yes.(1, 1) : 1 + 2(1) ≤ 4 is true, so yes.

-4

-3

-2

-1

0

1

2

3

4

5

-4 -3 -2 -1 0 1 2 3 4 5

There’s a pattern here.

Page 1003: Finite Math Lecture Slides

What other points satisfy x + 2y ≤ 4?

Let’s try out a few.

(2, 3) : 2 + 2(3) ≤ 4 is false, so no.(2,−3) : 2 + 2(−3) ≤ 4 is true, so yes.(−3, 2) : −3 + 2(2) ≤ 4 is true, so yes.(−1, 4) : −1 + 2(4) ≤ 4 is false, so no.

(−3,−2) : −3 + 2(−2) ≤ 4 is true, so yes.

(1, 1) : 1 + 2(1) ≤ 4 is true, so yes.

-4

-3

-2

-1

0

1

2

3

4

5

-4 -3 -2 -1 0 1 2 3 4 5

There’s a pattern here.

Page 1004: Finite Math Lecture Slides

What other points satisfy x + 2y ≤ 4?

Let’s try out a few.

(2, 3) : 2 + 2(3) ≤ 4 is false, so no.(2,−3) : 2 + 2(−3) ≤ 4 is true, so yes.(−3, 2) : −3 + 2(2) ≤ 4 is true, so yes.(−1, 4) : −1 + 2(4) ≤ 4 is false, so no.

(−3,−2) : −3 + 2(−2) ≤ 4 is true, so yes.(1, 1) : 1 + 2(1) ≤ 4 is true, so yes.

-4

-3

-2

-1

0

1

2

3

4

5

-4 -3 -2 -1 0 1 2 3 4 5

There’s a pattern here.

Page 1005: Finite Math Lecture Slides

What other points satisfy x + 2y ≤ 4?

Let’s try out a few.

(2, 3) : 2 + 2(3) ≤ 4 is false, so no.(2,−3) : 2 + 2(−3) ≤ 4 is true, so yes.(−3, 2) : −3 + 2(2) ≤ 4 is true, so yes.(−1, 4) : −1 + 2(4) ≤ 4 is false, so no.

(−3,−2) : −3 + 2(−2) ≤ 4 is true, so yes.(1, 1) : 1 + 2(1) ≤ 4 is true, so yes.

-4

-3

-2

-1

0

1

2

3

4

5

-4 -3 -2 -1 0 1 2 3 4 5

There’s a pattern here.

Page 1006: Finite Math Lecture Slides

-4

-3

-2

-1

0

1

2

3

4

5

-4 -3 -2 -1 0 1 2 3 4 5

The line x + 2y = 4 divides the plane into two half-planes.

In one half-plane all the points satisfy x + 2y ≤ 4;in the other half-plane none do.

So once we have graphed the line,it only remains to determine which half-plane should be shaded.

Page 1007: Finite Math Lecture Slides

-4

-3

-2

-1

0

1

2

3

4

5

-4 -3 -2 -1 0 1 2 3 4 5

The line x + 2y = 4 divides the plane into two half-planes.

In one half-plane all the points satisfy x + 2y ≤ 4;in the other half-plane none do.

So once we have graphed the line,it only remains to determine which half-plane should be shaded.

Page 1008: Finite Math Lecture Slides

-4

-3

-2

-1

0

1

2

3

4

5

-4 -3 -2 -1 0 1 2 3 4 5

The line x + 2y = 4 divides the plane into two half-planes.

In one half-plane all the points satisfy x + 2y ≤ 4;in the other half-plane none do.

So once we have graphed the line,it only remains to determine which half-plane should be shaded.

Page 1009: Finite Math Lecture Slides

There are two methods for doing that.

The first method is the test point method:test any point which is not on the line itself.

Any of the six points we looked at would do.

For example, (2, 3) does not satisfy the inequality;

therefore (2, 3) is in the wrong half-plane;

-4

-3

-2

-1

0

1

2

3

4

5

-4 -3 -2 -1 0 1 2 3 4 5

therefore we shade theother half-plane.

Page 1010: Finite Math Lecture Slides

There are two methods for doing that.

The first method is the test point method:test any point which is not on the line itself.

Any of the six points we looked at would do.

For example, (2, 3) does not satisfy the inequality;

therefore (2, 3) is in the wrong half-plane;

-4

-3

-2

-1

0

1

2

3

4

5

-4 -3 -2 -1 0 1 2 3 4 5

therefore we shade theother half-plane.

Page 1011: Finite Math Lecture Slides

There are two methods for doing that.

The first method is the test point method:test any point which is not on the line itself.

Any of the six points we looked at would do.

For example, (2, 3) does not satisfy the inequality;

therefore (2, 3) is in the wrong half-plane;

-4

-3

-2

-1

0

1

2

3

4

5

-4 -3 -2 -1 0 1 2 3 4 5

therefore we shade theother half-plane.

Page 1012: Finite Math Lecture Slides

The other method for determining which half-plane to shadeis to solve the inequality for one of the variables.

x + 2y ≤ 4x ≤ 4− 2y

This means that x is less than equal to the other side:

the variable x gets smaller towards the left,

-4

-3

-2

-1

0

1

2

3

4

5

-4 -3 -2 -1 0 1 2 3 4 5

right

left

so we should shade the leftside of the line x +2y = 4.

Page 1013: Finite Math Lecture Slides

The other method for determining which half-plane to shadeis to solve the inequality for one of the variables.

x + 2y ≤ 4x ≤ 4− 2y

This means that x is less than equal to the other side:

the variable x gets smaller towards the left,

-4

-3

-2

-1

0

1

2

3

4

5

-4 -3 -2 -1 0 1 2 3 4 5

right

left

so we should shade the leftside of the line x +2y = 4.

Page 1014: Finite Math Lecture Slides

The other method for determining which half-plane to shadeis to solve the inequality for one of the variables.

x + 2y ≤ 4x ≤ 4− 2y

This means that x is less than equal to the other side:

the variable x gets smaller towards the left,

-4

-3

-2

-1

0

1

2

3

4

5

-4 -3 -2 -1 0 1 2 3 4 5

right

left

so we should shade the leftside of the line x +2y = 4.

Page 1015: Finite Math Lecture Slides

The other method for determining which half-plane to shadeis to solve the inequality for one of the variables.

x + 2y ≤ 4x ≤ 4− 2y

This means that x is less than equal to the other side:

the variable x gets smaller towards the left,

-4

-3

-2

-1

0

1

2

3

4

5

-4 -3 -2 -1 0 1 2 3 4 5

right

left

so we should shade the leftside of the line x +2y = 4.

Page 1016: Finite Math Lecture Slides

The other method for determining which half-plane to shadeis to solve the inequality for one of the variables.

x + 2y ≤ 4x ≤ 4− 2y

This means that x is less than equal to the other side:

the variable x gets smaller towards the left,

-4

-3

-2

-1

0

1

2

3

4

5

-4 -3 -2 -1 0 1 2 3 4 5

right

left

so we should shade the leftside of the line x +2y = 4.

Page 1017: Finite Math Lecture Slides

Remember the algebra about inequalities:Multiplication by a negative reverses the inequality!

For example,−2x + 4y ≥ 6

− 2x ≥ 6− 4y

x ≤ −3 + 2y .

Page 1018: Finite Math Lecture Slides

Remember the algebra about inequalities:Multiplication by a negative reverses the inequality!

For example,−2x + 4y ≥ 6

− 2x ≥ 6− 4y

x ≤ −3 + 2y .

Page 1019: Finite Math Lecture Slides

Remember the algebra about inequalities:Multiplication by a negative reverses the inequality!

For example,−2x + 4y ≥ 6

− 2x ≥ 6− 4y

x ≤ −3 + 2y .

Page 1020: Finite Math Lecture Slides

Remember the algebra about inequalities:Multiplication by a negative reverses the inequality!

For example,−2x + 4y ≥ 6

− 2x ≥ 6− 4y

x ≤ −3 + 2y .

Page 1021: Finite Math Lecture Slides

Now let’s complicate things.

Let’s graph the system of inequalities

x + 2y ≤ 4 Iy ≤ 7− 3x II.

We’ve already done I, so let’s look at II.

The line y = 7− 3x has slope −3 and y -intercept at (0, 7).

-1

0

1

2

3

4

5

6

7

8

-1 0 1 2 3 4 5

We can use (0, 0) as a testpoint: it makes the in-equality true, so we shadethe lower left half-plane.

Page 1022: Finite Math Lecture Slides

Now let’s complicate things.

Let’s graph the system of inequalities

x + 2y ≤ 4 Iy ≤ 7− 3x II.

We’ve already done I, so let’s look at II.

The line y = 7− 3x has slope −3 and y -intercept at (0, 7).

-1

0

1

2

3

4

5

6

7

8

-1 0 1 2 3 4 5

We can use (0, 0) as a testpoint: it makes the in-equality true, so we shadethe lower left half-plane.

Page 1023: Finite Math Lecture Slides

Inequality I:

-1

0

1

2

3

4

5

6

7

8

-1 0 1 2 3 4 5

Page 1024: Finite Math Lecture Slides

Inequalities I and II together:

-1

0

1

2

3

4

5

6

7

8

-1 0 1 2 3 4 5

Page 1025: Finite Math Lecture Slides

Final Answer:

-1

0

1

2

3

4

5

6

7

8

-1 0 1 2 3 4 5

Page 1026: Finite Math Lecture Slides

The final answer is the region that satisfies all the inequalitiesfrom the original system.

There are two methods for putting the separate inequalitiestogether.

1. Use a different color for each—then the final answer is thedarkest region;

2. use each inequality to successively eliminate part of theplane—the the final answer is what is left over.

Page 1027: Finite Math Lecture Slides

The final answer is the region that satisfies all the inequalitiesfrom the original system.

There are two methods for putting the separate inequalitiestogether.

1. Use a different color for each—then the final answer is thedarkest region;

2. use each inequality to successively eliminate part of theplane—the the final answer is what is left over.

Page 1028: Finite Math Lecture Slides

Here’s the last problem done using successive elimination.Inequality I (red means eliminated):

-1

0

1

2

3

4

5

6

7

8

-1 0 1 2 3 4 5

Page 1029: Finite Math Lecture Slides

Inequality II (the final answer is white):

-1

0

1

2

3

4

5

6

7

8

-1 0 1 2 3 4 5

Page 1030: Finite Math Lecture Slides

We’re interested in using these graphs to solve LP problemsso our systems will usually include the natural constraints.

Example:x ≥ 0y ≥ 0

x + 2y ≤ 4 Iy ≤ 7− 3x II

The addition of the natural constraints produces a slight change:

since x and y must both be nonnegative,the answer includes only points from the first quadrant.

Page 1031: Finite Math Lecture Slides

We’re interested in using these graphs to solve LP problemsso our systems will usually include the natural constraints.

Example:x ≥ 0y ≥ 0

x + 2y ≤ 4 Iy ≤ 7− 3x II

The addition of the natural constraints produces a slight change:

since x and y must both be nonnegative,the answer includes only points from the first quadrant.

Page 1032: Finite Math Lecture Slides

-1

0

1

2

3

4

5

6

7

8

-1 0 1 2 3 4 5

I

II

In the context of LP problems, this is called the feasible set.

Page 1033: Finite Math Lecture Slides

-1

0

1

2

3

4

5

6

7

8

-1 0 1 2 3 4 5

I

II

In the context of LP problems, this is called the feasible set.

Page 1034: Finite Math Lecture Slides

We need to know a little bit more about the feasible set:

We also need to know the coordinates of the corners.

-1

0

1

2

3

4

-1 0 1 2 3 4

I

II

A B

C

D

A is obviously (0, 0).B and D are also easy:they’re intercepts of thelines.

Page 1035: Finite Math Lecture Slides

-1

0

1

2

3

4

-1 0 1 2 3 4

I

II

A B

C

D

B is the x-interceptof II: y = 7− 3x .

So

0 = 7− 3x7/3 = x ,

and B = (7/3, 0).

Similarly, D is the y -intercept of I: x + 2y = 4.

0 + 2y = 4y = 2D = (0, 2)

Page 1036: Finite Math Lecture Slides

-1

0

1

2

3

4

-1 0 1 2 3 4

I

II

A B

C

D

B is the x-interceptof II: y = 7− 3x .

So

0 = 7− 3x7/3 = x ,

and B = (7/3, 0).

Similarly, D is the y -intercept of I: x + 2y = 4.

0 + 2y = 4y = 2D = (0, 2)

Page 1037: Finite Math Lecture Slides

-1

0

1

2

3

4

-1 0 1 2 3 4

I

II

A B

C

D

B is the x-interceptof II: y = 7− 3x .

So

0 = 7− 3x7/3 = x ,

and B = (7/3, 0).

Similarly, D is the y -intercept of I: x + 2y = 4.

0 + 2y = 4y = 2D = (0, 2)

Page 1038: Finite Math Lecture Slides

-1

0

1

2

3

4

-1 0 1 2 3 4

I

II

A B

C

D

B is the x-interceptof II: y = 7− 3x .

So

0 = 7− 3x7/3 = x ,

and B = (7/3, 0).

Similarly, D is the y -intercept of I: x + 2y = 4.

0 + 2y = 4y = 2D = (0, 2)

Page 1039: Finite Math Lecture Slides

-1

0

1

2

3

4

-1 0 1 2 3 4

I

II

A B

C

D

C is the only hard one.It is formed by the intersec-tion of lines I and II, so wemust solve a system of lin-ear equations to find it:

x + 2y = 4y = 7− 3x .

By substitution,

x + 2(7− 3x) = 4−5x + 14 = 4

−5x = −10x = 2y = 7− 3(2) = 1.

Page 1040: Finite Math Lecture Slides

-1

0

1

2

3

4

-1 0 1 2 3 4

I

II

A B

C

D

C is the only hard one.It is formed by the intersec-tion of lines I and II, so wemust solve a system of lin-ear equations to find it:

x + 2y = 4y = 7− 3x .

By substitution,

x + 2(7− 3x) = 4−5x + 14 = 4

−5x = −10x = 2

y = 7− 3(2) = 1.

Page 1041: Finite Math Lecture Slides

-1

0

1

2

3

4

-1 0 1 2 3 4

I

II

A B

C

D

C is the only hard one.It is formed by the intersec-tion of lines I and II, so wemust solve a system of lin-ear equations to find it:

x + 2y = 4y = 7− 3x .

By substitution,

x + 2(7− 3x) = 4−5x + 14 = 4

−5x = −10x = 2y = 7− 3(2) = 1.

Page 1042: Finite Math Lecture Slides

So C = (2, 1). Now we have all the corners.

A (0, 0)B (7/3, 0)C (2, 1)D (0, 2)

This table will be crucial in actually solving LP problems.

Page 1043: Finite Math Lecture Slides

Solving LP Problems

Now suppose our objective is to maximize the value of 3x + 2y onthe feasible set. Here is the previous feasible set together with thevalues of 3x + 2y indicated by color: green for high values and redfor low values.

Of all the points in thefeasible set, which is thegreenest?

Suppose we were lookingfor the minimum?

Page 1044: Finite Math Lecture Slides

Solving LP Problems

Now suppose our objective is to maximize the value of 3x + 2y onthe feasible set. Here is the previous feasible set together with thevalues of 3x + 2y indicated by color: green for high values and redfor low values.

Of all the points in thefeasible set, which is thegreenest?

Suppose we were lookingfor the minimum?

Page 1045: Finite Math Lecture Slides

Solving LP Problems

Now suppose the objective function is x − y .Remember: green for high values and red for low values.

Of all the points in thefeasible set, which is thegreenest?

Which is the reddest?

Page 1046: Finite Math Lecture Slides

Solving LP Problems

Now suppose the objective function is x − y .Remember: green for high values and red for low values.

Of all the points in thefeasible set, which is thegreenest?

Which is the reddest?

Page 1047: Finite Math Lecture Slides

It seems that both the max and min values are at corner points.

In fact this always true if the objective function is linear .

With a linear objective function,the color gradient is straight.

So to solve an LP problem,all you have to do is to compare values of the objective functionat the corner points of the feasible set:one of those points must be the answer.

Page 1048: Finite Math Lecture Slides

It seems that both the max and min values are at corner points.

In fact this always true if the objective function is linear .

With a linear objective function,the color gradient is straight.

So to solve an LP problem,all you have to do is to compare values of the objective functionat the corner points of the feasible set:one of those points must be the answer.

Page 1049: Finite Math Lecture Slides

It seems that both the max and min values are at corner points.

In fact this always true if the objective function is linear .

With a linear objective function,the color gradient is straight.

So to solve an LP problem,all you have to do is to compare values of the objective functionat the corner points of the feasible set:one of those points must be the answer.

Page 1050: Finite Math Lecture Slides

Point 3x + 2y x − y

A (0, 0) 0 0B (7/3, 0) 7 7/3C (2, 1) 8 1D (0, 2) 4 −2

Page 1051: Finite Math Lecture Slides

This problem contains an additional complication.

Example: Find the max and min of 3x + 2y subject to theconstraints

x ≥ 0 y ≥ 0I x − 2y ≥ −6 II x − 3y ≤ 2.

Graph:

-1

0

1

2

3

4

-1 0 1 2 3 4

I

IIA B

C

The corners B and C arejust intercepts.

A = (0, 0)B = (2, 0)C = (0, 3)

Note that the feasible setcontinues forever to the up-per right.

Page 1052: Finite Math Lecture Slides

This problem contains an additional complication.

Example: Find the max and min of 3x + 2y subject to theconstraints

x ≥ 0 y ≥ 0I x − 2y ≥ −6 II x − 3y ≤ 2.

Graph:

-1

0

1

2

3

4

-1 0 1 2 3 4

I

IIA B

C

The corners B and C arejust intercepts.

A = (0, 0)B = (2, 0)C = (0, 3)

Note that the feasible setcontinues forever to the up-per right.

Page 1053: Finite Math Lecture Slides

This problem contains an additional complication.

Example: Find the max and min of 3x + 2y subject to theconstraints

x ≥ 0 y ≥ 0I x − 2y ≥ −6 II x − 3y ≤ 2.

Graph:

-1

0

1

2

3

4

-1 0 1 2 3 4

I

IIA B

C

The corners B and C arejust intercepts.

A = (0, 0)B = (2, 0)C = (0, 3)

Note that the feasible setcontinues forever to the up-per right.

Page 1054: Finite Math Lecture Slides

Here the objective function 3x + 2y is overlaid. See a problem?

There is no maximum, be-cause the objective func-tion just keeps gettingbigger toward the upperright.

There is a minimum at (0, 0).

How could we get those answers without using a computer tograph the objective function?

Page 1055: Finite Math Lecture Slides

Here the objective function 3x + 2y is overlaid. See a problem?

There is no maximum, be-cause the objective func-tion just keeps gettingbigger toward the upperright.

There is a minimum at (0, 0).

How could we get those answers without using a computer tograph the objective function?

Page 1056: Finite Math Lecture Slides

Here the objective function 3x + 2y is overlaid. See a problem?

There is no maximum, be-cause the objective func-tion just keeps gettingbigger toward the upperright.

There is a minimum at (0, 0).

How could we get those answers without using a computer tograph the objective function?

Page 1057: Finite Math Lecture Slides

Here the objective function 3x + 2y is overlaid. See a problem?

There is no maximum, be-cause the objective func-tion just keeps gettingbigger toward the upperright.

There is a minimum at (0, 0).

How could we get those answers without using a computer tograph the objective function?

Page 1058: Finite Math Lecture Slides

This feasible set is said to be unbounded.

-1

0

1

2

3

4

5

6

-1 0 1 2 3 4 5 6

I

IIA B

C

D

E

When the feasible setis unbounded we needadditional sample points,one from each unboundededge.

I’ll call D and E U-points.

They can be anywhere inside the unbounded edges.

Page 1059: Finite Math Lecture Slides

This feasible set is said to be unbounded.

-1

0

1

2

3

4

5

6

-1 0 1 2 3 4 5 6

I

IIA B

C

D

E

When the feasible setis unbounded we needadditional sample points,one from each unboundededge.

I’ll call D and E U-points.

They can be anywhere inside the unbounded edges.

Page 1060: Finite Math Lecture Slides

-1

0

1

2

3

4

5

6

-1 0 1 2 3 4 5 6

I

IIA B

C

D

E

Corners

A (0, 0)B (2, 0)C (0, 3)

U-points

DE

[h]

To get coordinates for D, notethat:

(a) D is on line I: x − 2y = −6;

(b) D is to the upper right of C .

We can use (b) to get one co-ordinate;then use (a) to get the other.

Page 1061: Finite Math Lecture Slides

-1

0

1

2

3

4

5

6

-1 0 1 2 3 4 5 6

I

IIA B

C

D

E

Corners

A (0, 0)B (2, 0)C (0, 3)

U-points

DE

[h]

To get coordinates for D, notethat:

(a) D is on line I: x − 2y = −6;

(b) D is to the upper right of C .We can use (b) to get one co-ordinate;then use (a) to get the other.

Page 1062: Finite Math Lecture Slides

[h]

Corners

A (0, 0)B (2, 0)C (0, 3)

U-points

DE

(a) D is on line I: x − 2y = −6;

(b) D is to the upper right of C .

By (b) we could use e.g. either x = 1or y = 4.

I’ll use the latter because it makesthe algebra easier.

By (a),x − 2(4) = −6

x = 2.

So for D we can use (2, 4).

Page 1063: Finite Math Lecture Slides

[h]

Corners

A (0, 0)B (2, 0)C (0, 3)

U-points

DE

(a) D is on line I: x − 2y = −6;

(b) D is to the upper right of C .

By (b) we could use e.g. either x = 1or y = 4.

I’ll use the latter because it makesthe algebra easier.

By (a),x − 2(4) = −6

x = 2.

So for D we can use (2, 4).

Page 1064: Finite Math Lecture Slides

For E the method is the same.

[h]

Corners

A (0, 0)B (2, 0)C (0, 3)

U-points

D (2, 4)E

(a) E is on line II: x − 3y = 2;

(b) E is to the upper right of B.

By (b) we could use e.g. either x = 3or y = 1.

I’ll use the latter because it makesthe algebra easier.

By (a),x − 3(1) = 2

x = 5.

So for E we can use (5, 1).

Page 1065: Finite Math Lecture Slides

For E the method is the same.

[h]

Corners

A (0, 0)B (2, 0)C (0, 3)

U-points

D (2, 4)E

(a) E is on line II: x − 3y = 2;

(b) E is to the upper right of B.

By (b) we could use e.g. either x = 3or y = 1.

I’ll use the latter because it makesthe algebra easier.

By (a),x − 3(1) = 2

x = 5.

So for E we can use (5, 1).

Page 1066: Finite Math Lecture Slides

For E the method is the same.

[h]

Corners

A (0, 0)B (2, 0)C (0, 3)

U-points

D (2, 4)E

(a) E is on line II: x − 3y = 2;

(b) E is to the upper right of B.

By (b) we could use e.g. either x = 3or y = 1.

I’ll use the latter because it makesthe algebra easier.

By (a),x − 3(1) = 2

x = 5.

So for E we can use (5, 1).

Page 1067: Finite Math Lecture Slides

Now complete the table.

Corners 3x + 2y

A (0, 0) 0B (2, 0) 6C (0, 3) 6

U-points

D (2, 4) 14E (5, 1) 17

The max value is at a U-point;the min value is at a corner.

The rule is:

1. if the best value is at a corner, that’s the answer;

2. if the best value is at a U-point, there is no answer.

Page 1068: Finite Math Lecture Slides

Now complete the table.

Corners 3x + 2y

A (0, 0) 0B (2, 0) 6C (0, 3) 6

U-points

D (2, 4) 14E (5, 1) 17

The max value is at a U-point;the min value is at a corner.

The rule is:

1. if the best value is at a corner, that’s the answer;

2. if the best value is at a U-point, there is no answer.

Page 1069: Finite Math Lecture Slides

Here’s the whole graph with all sample points and the objectivefunction overlaid.

Corners 3x + 2y

A (0, 0) 0B (2, 0) 6C (0, 3) 6

U-points

D (2, 4) 14E (5, 1) 17

The fact that the U-points take the highest values indicates thatthe values will increase forever.

Page 1070: Finite Math Lecture Slides

Here’s the same feasible set with a different objective function,y − x .

Corners y − x

A (0, 0) 0B (2, 0) −2C (0, 3) 3

U-points

D (2, 4) 2E (5, 1) −4

Here the max is 3 but there is no min.

Page 1071: Finite Math Lecture Slides

Here’s the same feasible set with a different objective function,y − x .

Corners y − x

A (0, 0) 0B (2, 0) −2C (0, 3) 3

U-points

D (2, 4) 2E (5, 1) −4

Here the max is 3 but there is no min.

Page 1072: Finite Math Lecture Slides

Markov Chains

A Markov Chain is a system that can be in different states atdifferent times.

Example: Weather: Rainy, Sunny, Cloudy; times: days.

In this case the weather is observed every day, and the state of theweather is assigned to one of the three states and recorded.

As this continues a chain of successive states is recorded.

R → S → S → C → . . .

Note that in this case the weather one day might have someinfluence on the next day’s weather.

Page 1073: Finite Math Lecture Slides

Markov Chains

A Markov Chain is a system that can be in different states atdifferent times.

Example: Weather: Rainy, Sunny, Cloudy; times: days.

In this case the weather is observed every day, and the state of theweather is assigned to one of the three states and recorded.

As this continues a chain of successive states is recorded.

R → S → S → C → . . .

Note that in this case the weather one day might have someinfluence on the next day’s weather.

Page 1074: Finite Math Lecture Slides

Markov Chains

A Markov Chain is a system that can be in different states atdifferent times.

Example: Weather: Rainy, Sunny, Cloudy; times: days.

In this case the weather is observed every day, and the state of theweather is assigned to one of the three states and recorded.

As this continues a chain of successive states is recorded.

R → S → S → C → . . .

Note that in this case the weather one day might have someinfluence on the next day’s weather.

Page 1075: Finite Math Lecture Slides

Markov Chains

A Markov Chain is a system that can be in different states atdifferent times.

Example: Weather: Rainy, Sunny, Cloudy; times: days.

In this case the weather is observed every day, and the state of theweather is assigned to one of the three states and recorded.

As this continues a chain of successive states is recorded.

R → S → S → C → . . .

Note that in this case the weather one day might have someinfluence on the next day’s weather.

Page 1076: Finite Math Lecture Slides

Another way to describe a Markov Chain:it’s like a Bernoulli Process with two differences.

1. The trials can have more outcomes than just S or F .

2. The trials are not independent: one observation may affectthe next.

In a Bernoulli process the probabilities are always the same;

in a M.C. they may depend on the previous state.

Page 1077: Finite Math Lecture Slides

Another way to describe a Markov Chain:it’s like a Bernoulli Process with two differences.

1. The trials can have more outcomes than just S or F .

2. The trials are not independent: one observation may affectthe next.

In a Bernoulli process the probabilities are always the same;

in a M.C. they may depend on the previous state.

Page 1078: Finite Math Lecture Slides

Another way to describe a Markov Chain:it’s like a Bernoulli Process with two differences.

1. The trials can have more outcomes than just S or F .

2. The trials are not independent: one observation may affectthe next.

In a Bernoulli process the probabilities are always the same;

in a M.C. they may depend on the previous state.

Page 1079: Finite Math Lecture Slides

Another way to describe a Markov Chain:it’s like a Bernoulli Process with two differences.

1. The trials can have more outcomes than just S or F .

2. The trials are not independent: one observation may affectthe next.

In a Bernoulli process the probabilities are always the same;

in a M.C. they may depend on the previous state.

Page 1080: Finite Math Lecture Slides

Transition DiagramsA good visual representation of a M.C. is by

a tree diagram with loops.Example: Suppose daily observations of the Dow Jones Average aremade and classified as either Rising, Falling, or Level.

Page 1081: Finite Math Lecture Slides

As with a tree diagram, we can also indicate the conditionalprobabilities of moving along a particular edge.

For example, the .7 from R to L means:if today the DJ was rising,there is a 70% chance that tomorrow it will be level.

Page 1082: Finite Math Lecture Slides

As with a tree diagram, we can also indicate the conditionalprobabilities of moving along a particular edge.

For example, the .7 from R to L means:if today the DJ was rising,there is a 70% chance that tomorrow it will be level.

Page 1083: Finite Math Lecture Slides

We can trim off any edges with 0 probability.

This looks nice, but it can be hard to find the number you want.

Page 1084: Finite Math Lecture Slides

We can trim off any edges with 0 probability.

This looks nice, but it can be hard to find the number you want.

Page 1085: Finite Math Lecture Slides

Here’s another way to present the same information.

EndR F L

Sta

rt R 0 .3 .7F .4 .5 .1L .2 .6 .2

So for example, if today the DJ is falling, the probability that itwill rise tomorrow is 40%.

This table is called the transition matrix of the M.C.

It’s usually denoted by P.

Page 1086: Finite Math Lecture Slides

Here’s another way to present the same information.

EndR F L

Sta

rt R 0 .3 .7F .4 .5 .1L .2 .6 .2

So for example, if today the DJ is falling, the probability that itwill rise tomorrow is

40%.

This table is called the transition matrix of the M.C.

It’s usually denoted by P.

Page 1087: Finite Math Lecture Slides

Here’s another way to present the same information.

EndR F L

Sta

rt R 0 .3 .7F .4 .5 .1L .2 .6 .2

So for example, if today the DJ is falling, the probability that itwill rise tomorrow is 40%.

This table is called the transition matrix of the M.C.

It’s usually denoted by P.

Page 1088: Finite Math Lecture Slides

Here’s another way to present the same information.

EndR F L

Sta

rt R 0 .3 .7F .4 .5 .1L .2 .6 .2

So for example, if today the DJ is falling, the probability that itwill rise tomorrow is 40%.

This table is called the transition matrix of the M.C.

It’s usually denoted by P.

Page 1089: Finite Math Lecture Slides

P =

0 .3 .7.4 .5 .1.2 .6 .2

Things to remember:

1. rows correspond to starting states;

2. columns correspond to ending states;

3. both have the states in the same order (here R, F , L, left toright or top to bottom);

4. rows always sum to 1;

5. shorthand: probability of a transition from state i to state j ispij .

For example, the probability of a transition from F to L is p23,which is the entry in the second row and third column of P.

Page 1090: Finite Math Lecture Slides

P =

0 .3 .7.4 .5 .1.2 .6 .2

Things to remember:

1. rows correspond to starting states;

2. columns correspond to ending states;

3. both have the states in the same order (here R, F , L, left toright or top to bottom);

4. rows always sum to 1;

5. shorthand: probability of a transition from state i to state j ispij .

For example, the probability of a transition from F to L is p23,which is the entry in the second row and third column of P.

Page 1091: Finite Math Lecture Slides

P =

0 .3 .7.4 .5 .1.2 .6 .2

Things to remember:

1. rows correspond to starting states;

2. columns correspond to ending states;

3. both have the states in the same order (here R, F , L, left toright or top to bottom);

4. rows always sum to 1;

5. shorthand: probability of a transition from state i to state j ispij .

For example, the probability of a transition from F to L is p23,which is the entry in the second row and third column of P.

Page 1092: Finite Math Lecture Slides

P =

0 .3 .7.4 .5 .1.2 .6 .2

Things to remember:

1. rows correspond to starting states;

2. columns correspond to ending states;

3. both have the states in the same order (here R, F , L, left toright or top to bottom);

4. rows always sum to 1;

5. shorthand: probability of a transition from state i to state j ispij .

For example, the probability of a transition from F to L is p23,which is the entry in the second row and third column of P.

Page 1093: Finite Math Lecture Slides

State 1 is Rise, State 2 is Fall, State 3 is Level.

P =

0 .3 .7.4 .5 .1.2 .6 .2

A lot of simple questions can be answered from P alone.

1. If tomorrow the DJ is level, what is the probability it will risethe day after?

20%

2. If the DJ fell today, what is most likely to happen tomorrow?Fall

3. If the DJ fell today, what is the probability of it falling thenext two days? 0.5 · 0.5 = 0.25

4. If the DJ fell today, what is the probability of it being levelthe next two days? 0.1 · 0.2 = 0.02

F.1−→ L

.2−→ L

Page 1094: Finite Math Lecture Slides

State 1 is Rise, State 2 is Fall, State 3 is Level.

P =

0 .3 .7.4 .5 .1.2 .6 .2

A lot of simple questions can be answered from P alone.

1. If tomorrow the DJ is level, what is the probability it will risethe day after? 20%

2. If the DJ fell today, what is most likely to happen tomorrow?

Fall

3. If the DJ fell today, what is the probability of it falling thenext two days? 0.5 · 0.5 = 0.25

4. If the DJ fell today, what is the probability of it being levelthe next two days? 0.1 · 0.2 = 0.02

F.1−→ L

.2−→ L

Page 1095: Finite Math Lecture Slides

State 1 is Rise, State 2 is Fall, State 3 is Level.

P =

0 .3 .7.4 .5 .1.2 .6 .2

A lot of simple questions can be answered from P alone.

1. If tomorrow the DJ is level, what is the probability it will risethe day after? 20%

2. If the DJ fell today, what is most likely to happen tomorrow?Fall

3. If the DJ fell today, what is the probability of it falling thenext two days?

0.5 · 0.5 = 0.25

4. If the DJ fell today, what is the probability of it being levelthe next two days? 0.1 · 0.2 = 0.02

F.1−→ L

.2−→ L

Page 1096: Finite Math Lecture Slides

State 1 is Rise, State 2 is Fall, State 3 is Level.

P =

0 .3 .7.4 .5 .1.2 .6 .2

A lot of simple questions can be answered from P alone.

1. If tomorrow the DJ is level, what is the probability it will risethe day after? 20%

2. If the DJ fell today, what is most likely to happen tomorrow?Fall

3. If the DJ fell today, what is the probability of it falling thenext two days? 0.5 · 0.5 = 0.25

4. If the DJ fell today, what is the probability of it being levelthe next two days?

0.1 · 0.2 = 0.02

F.1−→ L

.2−→ L

Page 1097: Finite Math Lecture Slides

State 1 is Rise, State 2 is Fall, State 3 is Level.

P =

0 .3 .7.4 .5 .1.2 .6 .2

A lot of simple questions can be answered from P alone.

1. If tomorrow the DJ is level, what is the probability it will risethe day after? 20%

2. If the DJ fell today, what is most likely to happen tomorrow?Fall

3. If the DJ fell today, what is the probability of it falling thenext two days? 0.5 · 0.5 = 0.25

4. If the DJ fell today, what is the probability of it being levelthe next two days? 0.1 · 0.2 = 0.02

F.1−→ L

.2−→ L

Page 1098: Finite Math Lecture Slides

State 1 is Rise, State 2 is Fall, State 3 is Level.

P =

0 .3 .7.4 .5 .1.2 .6 .2

Here’s a harder question.

If the DJ fell today, what is the probability it will be level two daysfrom now? In shorthand, p23(2) = ?

F?−→ ?

?−→ L

There are three cases:

F.4−→ R

.7−→ L F.5−→ F

.1−→ L F.1−→ L

.2−→ L

So the answer is 0.4 · 0.7 + 0.5 · 0.1 + 0.1 · 0.2.

Page 1099: Finite Math Lecture Slides

State 1 is Rise, State 2 is Fall, State 3 is Level.

P =

0 .3 .7.4 .5 .1.2 .6 .2

Here’s a harder question.

If the DJ fell today, what is the probability it will be level two daysfrom now? In shorthand, p23(2) = ?

F?−→ ?

?−→ L

There are three cases:

F.4−→ R

.7−→ L F.5−→ F

.1−→ L F.1−→ L

.2−→ L

So the answer is 0.4 · 0.7 + 0.5 · 0.1 + 0.1 · 0.2.

Page 1100: Finite Math Lecture Slides

State 1 is Rise, State 2 is Fall, State 3 is Level.

P =

0 .3 .7.4 .5 .1.2 .6 .2

Here’s a harder question.

If the DJ fell today, what is the probability it will be level two daysfrom now? In shorthand, p23(2) = ?

F?−→ ?

?−→ L

There are three cases:

F.4−→ R

.7−→ L F.5−→ F

.1−→ L F.1−→ L

.2−→ L

So the answer is 0.4 · 0.7 + 0.5 · 0.1 + 0.1 · 0.2.

Page 1101: Finite Math Lecture Slides

State 1 is Rise, State 2 is Fall, State 3 is Level.

P =

0 .3 .7.4 .5 .1.2 .6 .2

Here’s a harder question.

If the DJ fell today, what is the probability it will be level two daysfrom now? In shorthand, p23(2) = ?

F?−→ ?

?−→ L

There are three cases:

F.4−→ R

.7−→ L F.5−→ F

.1−→ L F.1−→ L

.2−→ L

So the answer is 0.4 · 0.7 + 0.5 · 0.1 + 0.1 · 0.2.

Page 1102: Finite Math Lecture Slides

State 1 is Rise, State 2 is Fall, State 3 is Level.

P =

0 .3 .7.4 .5 .1.2 .6 .2

Notice anything here?

0.4 · 0.7 + 0.5 · 0.1 + 0.1 · 0.2

It’s the product of the second row of P with the third column.

p23(2) =[.4 .5 .1

] .7.1.2

Page 1103: Finite Math Lecture Slides

State 1 is Rise, State 2 is Fall, State 3 is Level.

P =

0 .3 .7.4 .5 .1.2 .6 .2

Notice anything here?

0.4 · 0.7 + 0.5 · 0.1 + 0.1 · 0.2

It’s the product of the second row of P with the third column.

p23(2) =[.4 .5 .1

] .7.1.2

Page 1104: Finite Math Lecture Slides

I’ll write Ri for the i th row of P, and Cj for the j th column of P.

We have just seen that

R2C3 = p23(2).

How about this?R3C1 = p31(2)

In general,RiCj = pij(2).

Because of this, a nice pattern arises when we compute P · P.

Page 1105: Finite Math Lecture Slides

I’ll write Ri for the i th row of P, and Cj for the j th column of P.We have just seen that

R2C3 = p23(2).

How about this?R3C1 = p31(2)

In general,RiCj = pij(2).

Because of this, a nice pattern arises when we compute P · P.

Page 1106: Finite Math Lecture Slides

I’ll write Ri for the i th row of P, and Cj for the j th column of P.We have just seen that

R2C3 = p23(2).

How about this?R3C1 =

p31(2)

In general,RiCj = pij(2).

Because of this, a nice pattern arises when we compute P · P.

Page 1107: Finite Math Lecture Slides

I’ll write Ri for the i th row of P, and Cj for the j th column of P.We have just seen that

R2C3 = p23(2).

How about this?R3C1 = p31(2)

In general,RiCj = pij(2).

Because of this, a nice pattern arises when we compute P · P.

Page 1108: Finite Math Lecture Slides

I’ll write Ri for the i th row of P, and Cj for the j th column of P.We have just seen that

R2C3 = p23(2).

How about this?R3C1 = p31(2)

In general,RiCj = pij(2).

Because of this, a nice pattern arises when we compute P · P.

Page 1109: Finite Math Lecture Slides

I’ll write Ri for the i th row of P, and Cj for the j th column of P.We have just seen that

R2C3 = p23(2).

How about this?R3C1 = p31(2)

In general,RiCj = pij(2).

Because of this, a nice pattern arises when we compute P · P.

Page 1110: Finite Math Lecture Slides

Using the cannon method, P · P becomes[C1 C2 C3

]

R1

R2

R3

p11(2) p12(2) p13(2)

p21(2) p22(2) p23(2)

p31(2) p32(2) p33(2)

That is, P2 is the matrix of two-step transition probabilities.This can be denoted by P(2).

P(2) = P2

Page 1111: Finite Math Lecture Slides

Using the cannon method, P · P becomes[C1 C2 C3

]

R1

R2

R3

p11(2)

p12(2) p13(2)

p21(2) p22(2) p23(2)

p31(2) p32(2) p33(2)

That is, P2 is the matrix of two-step transition probabilities.This can be denoted by P(2).

P(2) = P2

Page 1112: Finite Math Lecture Slides

Using the cannon method, P · P becomes[C1 C2 C3

]

R1

R2

R3

p11(2) p12(2)

p13(2)

p21(2) p22(2) p23(2)

p31(2) p32(2) p33(2)

That is, P2 is the matrix of two-step transition probabilities.This can be denoted by P(2).

P(2) = P2

Page 1113: Finite Math Lecture Slides

Using the cannon method, P · P becomes[C1 C2 C3

]

R1

R2

R3

p11(2) p12(2) p13(2)

p21(2) p22(2) p23(2)

p31(2) p32(2) p33(2)

That is, P2 is the matrix of two-step transition probabilities.This can be denoted by P(2).

P(2) = P2

Page 1114: Finite Math Lecture Slides

Using the cannon method, P · P becomes[C1 C2 C3

]

R1

R2

R3

p11(2) p12(2) p13(2)

p21(2)

p22(2) p23(2)

p31(2) p32(2) p33(2)

That is, P2 is the matrix of two-step transition probabilities.This can be denoted by P(2).

P(2) = P2

Page 1115: Finite Math Lecture Slides

Using the cannon method, P · P becomes[C1 C2 C3

]

R1

R2

R3

p11(2) p12(2) p13(2)

p21(2) p22(2) p23(2)

p31(2) p32(2) p33(2)

That is, P2 is the matrix of two-step transition probabilities.This can be denoted by P(2).

P(2) = P2

Page 1116: Finite Math Lecture Slides

Using the cannon method, P · P becomes[C1 C2 C3

]

R1

R2

R3

p11(2) p12(2) p13(2)

p21(2) p22(2) p23(2)

p31(2) p32(2) p33(2)

That is, P2 is the matrix of two-step transition probabilities.This can be denoted by P(2).

P(2) = P2

Page 1117: Finite Math Lecture Slides

Summary

The probability of a transition from state i to state j in 2 steps is

pij(2) = the i , j entry in P2.

More generally,

The probability of a transition

from state ito state jin k steps is

pij(k) = the i , j entry in Pk .

Page 1118: Finite Math Lecture Slides

Summary

The probability of a transition from state i to state j in 2 steps is

pij(2) = the i , j entry in P2.

More generally,

The probability of a transition

from state ito state jin k steps is

pij(k) = the i , j entry in Pk .

Page 1119: Finite Math Lecture Slides

Example: Suppose IU is playing basketball against Purdue. State 1is “IU has just scored”; state 2 is “Purdue has just scored”. WhenIU scores, Purdue gets the ball, so in that case Purdue has a 70%chance of scoring next. When Purdue scores, IU gets the ball, andthen IU has an 80% of scoring next. Write out the transitionmatrix P and answer the following questions.

1. If Purdue has just scored, what is the probability that IUscores the next 2 baskets?

2. If Purdue has just scored, what is the probability that IUscores the basket after the next?

3. If Purdue has just scored, what is the probability that Purduescores the basket after the next?

4. If Purdue has just scored, what is the probability that IUscores the 4th basket from now?

Page 1120: Finite Math Lecture Slides

Example: Suppose IU is playing basketball against Purdue. State 1is “IU has just scored”; state 2 is “Purdue has just scored”. WhenIU scores, Purdue gets the ball, so in that case Purdue has a 70%chance of scoring next. When Purdue scores, IU gets the ball, andthen IU has an 80% of scoring next. Write out the transitionmatrix P and answer the following questions.

Transition matrix:

IU Purdue

IU

0.3 0.7Purdue 0.8 0.2

P =

[0.3 0.70.8 0.2

]

Page 1121: Finite Math Lecture Slides

Example: Suppose IU is playing basketball against Purdue. State 1is “IU has just scored”; state 2 is “Purdue has just scored”. WhenIU scores, Purdue gets the ball, so in that case Purdue has a 70%chance of scoring next. When Purdue scores, IU gets the ball, andthen IU has an 80% of scoring next. Write out the transitionmatrix P and answer the following questions.

Transition matrix:

IU Purdue

IU 0.3

0.7Purdue 0.8 0.2

P =

[0.3 0.70.8 0.2

]

Page 1122: Finite Math Lecture Slides

Example: Suppose IU is playing basketball against Purdue. State 1is “IU has just scored”; state 2 is “Purdue has just scored”. WhenIU scores, Purdue gets the ball, so in that case Purdue has a 70%chance of scoring next. When Purdue scores, IU gets the ball, andthen IU has an 80% of scoring next. Write out the transitionmatrix P and answer the following questions.

Transition matrix:

IU Purdue

IU 0.3 0.7Purdue

0.8 0.2

P =

[0.3 0.70.8 0.2

]

Page 1123: Finite Math Lecture Slides

Example: Suppose IU is playing basketball against Purdue. State 1is “IU has just scored”; state 2 is “Purdue has just scored”. WhenIU scores, Purdue gets the ball, so in that case Purdue has a 70%chance of scoring next. When Purdue scores, IU gets the ball, andthen IU has an 80% of scoring next. Write out the transitionmatrix P and answer the following questions.

Transition matrix:

IU Purdue

IU 0.3 0.7Purdue 0.8

0.2

P =

[0.3 0.70.8 0.2

]

Page 1124: Finite Math Lecture Slides

Example: Suppose IU is playing basketball against Purdue. State 1is “IU has just scored”; state 2 is “Purdue has just scored”. WhenIU scores, Purdue gets the ball, so in that case Purdue has a 70%chance of scoring next. When Purdue scores, IU gets the ball, andthen IU has an 80% of scoring next. Write out the transitionmatrix P and answer the following questions.

Transition matrix:

IU Purdue

IU 0.3 0.7Purdue 0.8 0.2

P =

[0.3 0.70.8 0.2

]

Page 1125: Finite Math Lecture Slides

States: IU, PU

P =

[0.3 0.70.8 0.2

]1: If Purdue has just scored, what is the probability that IU

scores the next 2 baskets?

Solution:PU

.8−→ IU.3−→ IU

p21 p11 = (.8)(.3) = .24

Page 1126: Finite Math Lecture Slides

States: IU, PU

P =

[0.3 0.70.8 0.2

]1: If Purdue has just scored, what is the probability that IU

scores the next 2 baskets?

Solution:PU

.8−→ IU.3−→ IU

p21 p11 = (.8)(.3) = .24

Page 1127: Finite Math Lecture Slides

States: IU, PU

P =

[0.3 0.70.8 0.2

]2: If Purdue has just scored, what is the probability that IU

scores the basket after the next?

Solution: We want p21(2), which is the 2, 1 entry in P2.

P2 =

[.65 .35.40 .60

]p21(2) = .40

Note that the rows in P2 also sum to 1.

Page 1128: Finite Math Lecture Slides

States: IU, PU

P =

[0.3 0.70.8 0.2

]2: If Purdue has just scored, what is the probability that IU

scores the basket after the next?

Solution: We want p21(2), which is the 2, 1 entry in P2.

P2 =

[.65 .35.40 .60

]p21(2) = .40

Note that the rows in P2 also sum to 1.

Page 1129: Finite Math Lecture Slides

States: IU, PU

P =

[0.3 0.70.8 0.2

]2: If Purdue has just scored, what is the probability that IU

scores the basket after the next?

Solution: We want p21(2), which is the 2, 1 entry in P2.

P2 =

[.65 .35.40 .60

]p21(2) = .40

Note that the rows in P2 also sum to 1.

Page 1130: Finite Math Lecture Slides

States: IU, PU

P =

[0.3 0.70.8 0.2

]2: If Purdue has just scored, what is the probability that IU

scores the basket after the next?

Solution: We want p21(2), which is the 2, 1 entry in P2.

P2 =

[.65 .35.40 .60

]p21(2) = .40

Note that the rows in P2 also sum to 1.

Page 1131: Finite Math Lecture Slides

States: IU, PU

P =

[0.3 0.70.8 0.2

]

P2 =

[.65 .35.40 .60

]3: If Purdue has just scored, what is the probability that Purdue

scores the basket after the next?

Solution: We want p22(2), which is the 2, 2 entry in P2.

p22(2) = .60

Alternatively, since this M.C. has only two states

p22(2) = 1− p21(2) = 1− .40 = .60,

using our previous answer.

Page 1132: Finite Math Lecture Slides

States: IU, PU

P =

[0.3 0.70.8 0.2

]

P2 =

[.65 .35.40 .60

]3: If Purdue has just scored, what is the probability that Purdue

scores the basket after the next?

Solution: We want p22(2), which is the 2, 2 entry in P2.

p22(2) = .60

Alternatively, since this M.C. has only two states

p22(2) = 1− p21(2) = 1− .40 = .60,

using our previous answer.

Page 1133: Finite Math Lecture Slides

States: IU, PU

P =

[0.3 0.70.8 0.2

]

P2 =

[.65 .35.40 .60

]3: If Purdue has just scored, what is the probability that Purdue

scores the basket after the next?

Solution: We want p22(2), which is the 2, 2 entry in P2.

p22(2) = .60

Alternatively, since this M.C. has only two states

p22(2) = 1− p21(2) = 1− .40 = .60,

using our previous answer.

Page 1134: Finite Math Lecture Slides

States: IU, PU

P =

[0.3 0.70.8 0.2

]

P2 =

[.65 .35.40 .60

]4: If Purdue has just scored, what is the probability that IU

scores the 4th basket from now?

Solution: We want p21(4), which is the 2, 1 entry in P4.

P4 = (P2)2

=

[.5625 .4375.5000 .5000

]p21(4) = .5

Page 1135: Finite Math Lecture Slides

States: IU, PU

P =

[0.3 0.70.8 0.2

]

P2 =

[.65 .35.40 .60

]4: If Purdue has just scored, what is the probability that IU

scores the 4th basket from now?

Solution: We want p21(4), which is the 2, 1 entry in P4.

P4 = (P2)2

=

[.5625 .4375.5000 .5000

]p21(4) = .5

Page 1136: Finite Math Lecture Slides

State Vectors

Even when we’re not sure which state a M.C. is currently in,we can still describe the situation with a state vector.

If a M.C. has two states,and we have no idea which state it is in now,we can say that its state vector X is

X =[

0.5 0.5].

The first number is probability of being in state 1,the second is the probability of being in state 2.

Note that the numbers in the state vector are probabilities,which must sum to 1.

Page 1137: Finite Math Lecture Slides

State Vectors

Even when we’re not sure which state a M.C. is currently in,we can still describe the situation with a state vector.

If a M.C. has two states,and we have no idea which state it is in now,we can say that its state vector X is

X =[

0.5 0.5].

The first number is probability of being in state 1,the second is the probability of being in state 2.

Note that the numbers in the state vector are probabilities,which must sum to 1.

Page 1138: Finite Math Lecture Slides

State Vectors

Even when we’re not sure which state a M.C. is currently in,we can still describe the situation with a state vector.

If a M.C. has two states,and we have no idea which state it is in now,we can say that its state vector X is

X =[

0.5 0.5].

The first number is probability of being in state 1,the second is the probability of being in state 2.

Note that the numbers in the state vector are probabilities,which must sum to 1.

Page 1139: Finite Math Lecture Slides

Example: Suppose a two state M.C. is twice as likely to now be instate 1 as to be in state 2. What is the state vector?

Solution:X =

[2x x

]Since the entries must sum to 1,

2x + x = 1

x = 1/3

X =[

2/31/3]

Page 1140: Finite Math Lecture Slides

Example: Suppose a two state M.C. is twice as likely to now be instate 1 as to be in state 2. What is the state vector?

Solution:X =

[2x x

]Since the entries must sum to 1,

2x + x = 1

x = 1/3

X =[

2/31/3]

Page 1141: Finite Math Lecture Slides

Example: Suppose a two state M.C. is twice as likely to now be instate 1 as to be in state 2. What is the state vector?

Solution:X =

[2x x

]

Since the entries must sum to 1,

2x + x = 1

x = 1/3

X =[

2/31/3]

Page 1142: Finite Math Lecture Slides

Example: Suppose a two state M.C. is twice as likely to now be instate 1 as to be in state 2. What is the state vector?

Solution:X =

[2x x

]Since the entries must sum to 1,

2x + x = 1

x = 1/3

X =[

2/31/3]

Page 1143: Finite Math Lecture Slides

Example: Suppose a two state M.C. is twice as likely to now be instate 1 as to be in state 2. What is the state vector?

Solution:X =

[2x x

]Since the entries must sum to 1,

2x + x = 1

x = 1/3

X =[

2/31/3]

Page 1144: Finite Math Lecture Slides

Example: Suppose a 4 state M.C. now has a 20% probability ofbeing in state 1, equal probabilities of being in states 2 or 3, andthree times the probability of being in state 4 as in state 3. Whatis its current state vector?

Solution:X =

[0.2 x x 3x

]Since the entries must sum to 1,

0.2 + x + x + 3x = 1

5x = 0.8

x = 0.16

X =[

0.2 0.16 0.16 0.48]

Page 1145: Finite Math Lecture Slides

Example: Suppose a 4 state M.C. now has a 20% probability ofbeing in state 1, equal probabilities of being in states 2 or 3, andthree times the probability of being in state 4 as in state 3. Whatis its current state vector?

Solution:X =

[0.2 x x 3x

]Since the entries must sum to 1,

0.2 + x + x + 3x = 1

5x = 0.8

x = 0.16

X =[

0.2 0.16 0.16 0.48]

Page 1146: Finite Math Lecture Slides

Example: Suppose a 4 state M.C. now has a 20% probability ofbeing in state 1, equal probabilities of being in states 2 or 3, andthree times the probability of being in state 4 as in state 3. Whatis its current state vector?

Solution:X =

[0.2 x x 3x

]

Since the entries must sum to 1,

0.2 + x + x + 3x = 1

5x = 0.8

x = 0.16

X =[

0.2 0.16 0.16 0.48]

Page 1147: Finite Math Lecture Slides

Example: Suppose a 4 state M.C. now has a 20% probability ofbeing in state 1, equal probabilities of being in states 2 or 3, andthree times the probability of being in state 4 as in state 3. Whatis its current state vector?

Solution:X =

[0.2 x x 3x

]Since the entries must sum to 1,

0.2 + x + x + 3x = 1

5x = 0.8

x = 0.16

X =[

0.2 0.16 0.16 0.48]

Page 1148: Finite Math Lecture Slides

Example: Suppose a 4 state M.C. now has a 20% probability ofbeing in state 1, equal probabilities of being in states 2 or 3, andthree times the probability of being in state 4 as in state 3. Whatis its current state vector?

Solution:X =

[0.2 x x 3x

]Since the entries must sum to 1,

0.2 + x + x + 3x = 1

5x = 0.8

x = 0.16

X =[

0.2 0.16 0.16 0.48]

Page 1149: Finite Math Lecture Slides

Any uncertainty about the current state is likely to be magnifiedfor future states, so we’ll need state vectors to describe futuresituations as well.

X0 = initial state vectorX1 = state vector after 1 stepX2 = state vector after 2 steps

...Xk = state vector after k steps

Page 1150: Finite Math Lecture Slides

Recall our previous result.

The probability of a transition

from state ito state jin k steps is

pij(k) = the i , j entry in Pk .

In English:every factor of P takes you one step further into the future.

Page 1151: Finite Math Lecture Slides

The same is true for the state vectors:

X1 = X0PX2 = X1P = X0P2

Xk+1 = XkP = X0Pk .

Example: Suppose a two state M.C. with transition matrix

P =

[0.4 0.60.2 0.8

]is equally likely initially to be in either state. What is theprobability it is in state 2 after two steps?

Solution:

X2 = X0P2 =[

0.5 0.5] [ 0.4 0.6

0.2 0.8

] [0.4 0.60.2 0.8

]

=[

0.3 0.7] [ 0.4 0.6

0.2 0.8

]=

[0.26 0.74

]

Page 1152: Finite Math Lecture Slides

The same is true for the state vectors:

X1 = X0PX2 = X1P = X0P2

Xk+1 = XkP = X0Pk .

Example: Suppose a two state M.C. with transition matrix

P =

[0.4 0.60.2 0.8

]is equally likely initially to be in either state. What is theprobability it is in state 2 after two steps?

Solution:

X2 = X0P2 =[

0.5 0.5] [ 0.4 0.6

0.2 0.8

] [0.4 0.60.2 0.8

]

=[

0.3 0.7] [ 0.4 0.6

0.2 0.8

]=

[0.26 0.74

]

Page 1153: Finite Math Lecture Slides

The same is true for the state vectors:

X1 = X0PX2 = X1P = X0P2

Xk+1 = XkP = X0Pk .

Example: Suppose a two state M.C. with transition matrix

P =

[0.4 0.60.2 0.8

]is equally likely initially to be in either state. What is theprobability it is in state 2 after two steps?

Solution:

X2 = X0P2 =[

0.5 0.5] [ 0.4 0.6

0.2 0.8

] [0.4 0.60.2 0.8

]

=

[0.3 0.7

] [ 0.4 0.60.2 0.8

]=

[0.26 0.74

]

Page 1154: Finite Math Lecture Slides

The same is true for the state vectors:

X1 = X0PX2 = X1P = X0P2

Xk+1 = XkP = X0Pk .

Example: Suppose a two state M.C. with transition matrix

P =

[0.4 0.60.2 0.8

]is equally likely initially to be in either state. What is theprobability it is in state 2 after two steps?

Solution:

X2 = X0P2 =[

0.5 0.5] [ 0.4 0.6

0.2 0.8

] [0.4 0.60.2 0.8

]

=[

0.3 0.7] [ 0.4 0.6

0.2 0.8

]=

[0.26 0.74

]

Page 1155: Finite Math Lecture Slides

Let’s continue this example.

P =

[0.4 0.60.2 0.8

]X0 =

[0.5 0.5

]X1 =

[0.3 0.7

]X2 =

[0.26 0.74

]X3 =

[0.252 0.748

]X32 =

[.2500000000000006 .7500000000000018

]X∞ =

[.25 .75

]

Page 1156: Finite Math Lecture Slides

Let’s continue this example.

P =

[0.4 0.60.2 0.8

]X0 =

[0.5 0.5

]X1 =

[0.3 0.7

]X2 =

[0.26 0.74

]X3 =

[0.252 0.748

]X32 =

[.2500000000000006 .7500000000000018

]X∞ =

[.25 .75

]

Page 1157: Finite Math Lecture Slides

Let’s continue this example.

P =

[0.4 0.60.2 0.8

]X0 =

[0.5 0.5

]X1 =

[0.3 0.7

]X2 =

[0.26 0.74

]X3 =

[0.252 0.748

]X32 =

[.2500000000000006 .7500000000000018

]X∞ =

[.25 .75

]

Page 1158: Finite Math Lecture Slides

X∞ =[.25 .75

]What does X∞ mean?

You could say it contains the probabilities of being in each stateover the last ∞− 32 steps of the process.

We call X∞ the vector of stable probabilitiesand we denote it by W ,because this is Finite Math after all.

You can think of W ascontaining the long-term probabilities of each state,the fraction of time the M.C. spends in each state in the long run.

Page 1159: Finite Math Lecture Slides

X∞ =[.25 .75

]What does X∞ mean?

You could say it contains the probabilities of being in each stateover the last ∞− 32 steps of the process.

We call X∞ the vector of stable probabilitiesand we denote it by W ,because this is Finite Math after all.

You can think of W ascontaining the long-term probabilities of each state,the fraction of time the M.C. spends in each state in the long run.

Page 1160: Finite Math Lecture Slides

X∞ =[.25 .75

]What does X∞ mean?

You could say it contains the probabilities of being in each stateover the last ∞− 32 steps of the process.

We call X∞ the vector of stable probabilitiesand we denote it by W ,because this is Finite Math after all.

You can think of W ascontaining the long-term probabilities of each state,the fraction of time the M.C. spends in each state in the long run.

Page 1161: Finite Math Lecture Slides

X∞ =[.25 .75

]What does X∞ mean?

You could say it contains the probabilities of being in each stateover the last ∞− 32 steps of the process.

We call X∞ the vector of stable probabilitiesand we denote it by W ,because this is Finite Math after all.

You can think of W ascontaining the long-term probabilities of each state,the fraction of time the M.C. spends in each state in the long run.

Page 1162: Finite Math Lecture Slides

How can we calculate W without using a computer?

Notice that W has two important characteristics:

1. it is a state vector, so its entries sum to 1;

2. it is stable, meaning WP = W .

The last property is because WP = X∞P represents the statevector after ∞+ 1 steps, which should be the same as X∞.

Each of these properties can be expressed in matrix equation form

Page 1163: Finite Math Lecture Slides

How can we calculate W without using a computer?

Notice that W has two important characteristics:

1. it is a state vector, so its entries sum to 1;

2. it is stable, meaning WP = W .

The last property is because WP = X∞P represents the statevector after ∞+ 1 steps, which should be the same as X∞.

Each of these properties can be expressed in matrix equation form

Page 1164: Finite Math Lecture Slides

How can we calculate W without using a computer?

Notice that W has two important characteristics:

1. it is a state vector, so its entries sum to 1;

2. it is stable, meaning WP = W .

The last property is because WP = X∞P represents the statevector after ∞+ 1 steps, which should be the same as X∞.

Each of these properties can be expressed in matrix equation form

Page 1165: Finite Math Lecture Slides

How can we calculate W without using a computer?

Notice that W has two important characteristics:

1. it is a state vector, so its entries sum to 1;

2. it is stable, meaning WP = W .

The last property is because WP = X∞P represents the statevector after ∞+ 1 steps, which should be the same as X∞.

Each of these properties can be expressed in matrix equation form

Page 1166: Finite Math Lecture Slides

1. W is a state vector, so its entries sum to 1.

Think of W as

W =[

w1 w2 w3 . . . wn

].

Then property 1 is equivalent to

w1 + w2 + w3 + · · ·+ wn = 1

[1 1 1 . . . 1

]

w1

w2

w3...

wn

= 1

Page 1167: Finite Math Lecture Slides

1. W is a state vector, so its entries sum to 1.

Think of W as

W =[

w1 w2 w3 . . . wn

].

Then property 1 is equivalent to

w1 + w2 + w3 + · · ·+ wn = 1

[1 1 1 . . . 1

]

w1

w2

w3...

wn

= 1

Page 1168: Finite Math Lecture Slides

1. W is a state vector, so its entries sum to 1.

Think of W as

W =[

w1 w2 w3 . . . wn

].

Then property 1 is equivalent to

w1 + w2 + w3 + · · ·+ wn = 1

[1 1 1 . . . 1

]

w1

w2

w3...

wn

= 1

Page 1169: Finite Math Lecture Slides

2. W is stable, meaning WP = W .

We’ll just do a little elementary algebra:

WP = W

WP −W = 0

(where 0 is the zero matrix)

W (P − I) = 0

Page 1170: Finite Math Lecture Slides

2. W is stable, meaning WP = W .

We’ll just do a little elementary algebra:

WP = W

WP −W = 0

(where 0 is the zero matrix)

W (

P − I) = 0

Page 1171: Finite Math Lecture Slides

2. W is stable, meaning WP = W .

We’ll just do a little elementary algebra:

WP = W

WP −W = 0

(where 0 is the zero matrix)

W (P − I) = 0

Page 1172: Finite Math Lecture Slides

W (P − I) = 0

Now there’s a slight problem:

In chapters 5 and 6 we learned about matrix methods for solvingsystems of equations.

In all those methods we were solving for a column vector,on the right.

For example, AX = B.

But here we are solving for W ,which is a row vector on the left.

Is everything we learned previously useless here?

Page 1173: Finite Math Lecture Slides

W (P − I) = 0

Now there’s a slight problem:

In chapters 5 and 6 we learned about matrix methods for solvingsystems of equations.

In all those methods we were solving for a column vector,on the right.

For example, AX = B.

But here we are solving for W ,which is a row vector on the left.

Is everything we learned previously useless here?

Page 1174: Finite Math Lecture Slides

There’s simple trick to save us:

turn everything sideways.

This is called transposing a matrix.

Example:

A =

[1 2 34 5 6

]

At =

1 42 53 6

Page 1175: Finite Math Lecture Slides

There’s simple trick to save us:

turn everything sideways.

This is called transposing a matrix.

Example:

A =

[1 2 34 5 6

]

At =

1 42 53 6

Page 1176: Finite Math Lecture Slides

There’s simple trick to save us:

turn everything sideways.

This is called transposing a matrix.

Example:

A =

[1 2 34 5 6

]

At =

1 42 53 6

Page 1177: Finite Math Lecture Slides

When you transpose matrices, you also have to reverse the order inwhich they are multiplied:

You’re switching the positions of the lower left and upper rightcannons. 7

89

[

1 2 34 5 6

]becomes 1 4

2 53 6

[

7 8 9]

Page 1178: Finite Math Lecture Slides

So we’ll just transpose the equation W (P − I) = 0:

(P − I)tW t = 0

Notice that the other equation can written in a similar way.

[1 1 1 . . . 1

]

w1

w2

w3...

wn

= 1

[1 1 1 . . . 1

]W t = 1

Page 1179: Finite Math Lecture Slides

So we’ll just transpose the equation W (P − I) = 0:

(P − I)tW t = 0

Notice that the other equation can written in a similar way.

[1 1 1 . . . 1

]

w1

w2

w3...

wn

= 1

[1 1 1 . . . 1

]W t = 1

Page 1180: Finite Math Lecture Slides

[1 1 1 . . . 1

]W t = 1

(P − I)tW t = 0

One great feature of matrix equations is that they’re stackable,like modular furniture.

1 1 1 . . . 1

(P − I)t

W t =

100...0

Page 1181: Finite Math Lecture Slides

1 1 1 . . . 1

(P − I)t

W t =

100...0

Now we can apply the method of chapter 5:convert to augmented matrix form

1 1 1 . . . 1 10

(P − I)t 00

and then row reduce.

Page 1182: Finite Math Lecture Slides

Method to Calculate W

To the find the vector W of stable probabilities for a M.C. withtranstion matrix P, form the augmented matrix

1 1 1 . . . 1 10

(P − I)t 00

and row reduce.

At the end you will have W t,i.e. W written as a column vector.

Page 1183: Finite Math Lecture Slides

Example: Find W for the M.C. in the previous example, with

P =

[0.4 0.60.2 0.8

]

Solution:

P − I =

[0.4 0.60.2 0.8

]−[

1 00 1

]

=

[−0.6 0.6

0.2 −0.2

]

(P − I)t =

[−0.6 0.2

0.6 −0.2

]

Page 1184: Finite Math Lecture Slides

Example: Find W for the M.C. in the previous example, with

P =

[0.4 0.60.2 0.8

]

Solution:

P − I =

[0.4 0.60.2 0.8

]−[

1 00 1

]

=

[−0.6 0.6

0.2 −0.2

]

(P − I)t =

[−0.6 0.2

0.6 −0.2

]

Page 1185: Finite Math Lecture Slides

Example: Find W for the M.C. in the previous example, with

P =

[0.4 0.60.2 0.8

]

Solution:

P − I =

[0.4 0.60.2 0.8

]−[

1 00 1

]

=

[−0.6 0.6

0.2 −0.2

]

(P − I)t =

[−0.6 0.2

0.6 −0.2

]

Page 1186: Finite Math Lecture Slides

(P − I)t =

[−0.6 0.2

0.6 −0.2

]Therefore the augmented matrix we need is 1 1 1

−0.6 0.2 00.6 −0.2 0

.

Now we row reduce.

0.6R1+→ R2 :

1 1 10 0.8 0.6

0.6 −0.2 0

− 0.6R1+→ R3 :

1 1 10 0.8 0.60 −0.8 −0.6

Page 1187: Finite Math Lecture Slides

(P − I)t =

[−0.6 0.2

0.6 −0.2

]Therefore the augmented matrix we need is 1 1 1

−0.6 0.2 00.6 −0.2 0

.Now we row reduce.

0.6R1+→ R2 :

1 1 10 0.8 0.6

0.6 −0.2 0

− 0.6R1+→ R3 :

1 1 10 0.8 0.60 −0.8 −0.6

Page 1188: Finite Math Lecture Slides

(P − I)t =

[−0.6 0.2

0.6 −0.2

]Therefore the augmented matrix we need is 1 1 1

−0.6 0.2 00.6 −0.2 0

.Now we row reduce.

0.6R1+→ R2 :

1 1 10 0.8 0.6

0.6 −0.2 0

− 0.6R1+→ R3 :

1 1 10 0.8 0.60 −0.8 −0.6

Page 1189: Finite Math Lecture Slides

1 1 10 0.8 0.60 −0.8 −0.6

Notice that row 3 is an exact multiple of row 2.Therefore we can cancel one of them.[

1 1 10 0.8 0.6

]

5/4·→ R2 :

[1 1 10 1 0.75

]

− 1R2+→ R1 :

[1 0 0.250 1 0.75

]So that

W =[

0.25 0.75].

Page 1190: Finite Math Lecture Slides

1 1 10 0.8 0.60 −0.8 −0.6

Notice that row 3 is an exact multiple of row 2.Therefore we can cancel one of them.[

1 1 10 0.8 0.6

]

5/4·→ R2 :

[1 1 10 1 0.75

]

− 1R2+→ R1 :

[1 0 0.250 1 0.75

]So that

W =[

0.25 0.75].

Page 1191: Finite Math Lecture Slides

1 1 10 0.8 0.60 −0.8 −0.6

Notice that row 3 is an exact multiple of row 2.Therefore we can cancel one of them.[

1 1 10 0.8 0.6

]

5/4·→ R2 :

[1 1 10 1 0.75

]

− 1R2+→ R1 :

[1 0 0.250 1 0.75

]

So thatW =

[0.25 0.75

].

Page 1192: Finite Math Lecture Slides

1 1 10 0.8 0.60 −0.8 −0.6

Notice that row 3 is an exact multiple of row 2.Therefore we can cancel one of them.[

1 1 10 0.8 0.6

]

5/4·→ R2 :

[1 1 10 1 0.75

]

− 1R2+→ R1 :

[1 0 0.250 1 0.75

]So that

W =[

0.25 0.75].

Page 1193: Finite Math Lecture Slides

Mysteriously, some students do not enjoy calculating stable vectors.

If you are repulsed by my method,have a look at the method shown in the book (section 8.3).

If you’re good at algebra you can solve for W byconverting WP = W into ordinary equations and throwing in

w1 + w2 + w3 + · · ·+ wn = 1

to get a system of equations you can then solve to get the the ws.

Page 1194: Finite Math Lecture Slides

Regular Markov ChainsI’ve misled you.There are M.C.s for which X∞ does not exist.

If this M.C. starts in state 1, then

X0 =[

1 0]

X1 =[

0 1]

X2 =[

1 0]

Xeven =[

1 0]

Xodd =[

0 1].

Page 1195: Finite Math Lecture Slides

Regular Markov ChainsI’ve misled you.There are M.C.s for which X∞ does not exist.

If this M.C. starts in state 1, then

X0 =[

1 0]

X1 =[

0 1]

X2 =[

1 0]

Xeven =[

1 0]

Xodd =[

0 1].

Page 1196: Finite Math Lecture Slides

Regular Markov ChainsI’ve misled you.There are M.C.s for which X∞ does not exist.

If this M.C. starts in state 1, then

X0 =[

1 0]

X1 =[

0 1]

X2 =[

1 0]

Xeven =[

1 0]

Xodd =[

0 1].

Page 1197: Finite Math Lecture Slides

Regular Markov ChainsI’ve misled you.There are M.C.s for which X∞ does not exist.

If this M.C. starts in state 1, then

X0 =[

1 0]

X1 =[

0 1]

X2 =[

1 0]

Xeven =[

1 0]

Xodd =[

0 1].

Page 1198: Finite Math Lecture Slides

Similarly with this M.C.

The problem is:not enough connections between the states.

Page 1199: Finite Math Lecture Slides

Similarly with this M.C.

The problem is:not enough connections between the states.

Page 1200: Finite Math Lecture Slides

The most connections possible: every state is connected to everyother state.

You can recognize that immediately when you look at thetransition matrix.

P =

.2 .3 .5.1 .2 .7.4 .5 .1

P =

.2 0 .8.1 .9 0.4 .5 .1

On the left: fully connected.On the right: some connections missing.

Page 1201: Finite Math Lecture Slides

The most connections possible: every state is connected to everyother state.

You can recognize that immediately when you look at thetransition matrix.

P =

.2 .3 .5.1 .2 .7.4 .5 .1

P =

.2 0 .8.1 .9 0.4 .5 .1

On the left: fully connected.On the right: some connections missing.

Page 1202: Finite Math Lecture Slides

The most connections possible: every state is connected to everyother state.

You can recognize that immediately when you look at thetransition matrix.

P =

.2 .3 .5.1 .2 .7.4 .5 .1

P =

.2 0 .8.1 .9 0.4 .5 .1

On the left: fully connected.On the right: some connections missing.

Page 1203: Finite Math Lecture Slides

The most connections possible: every state is connected to everyother state.

You can recognize that immediately when you look at thetransition matrix.

P =

.2 .3 .5.1 .2 .7.4 .5 .1

P =

.2 0 .8.1 .9 0.4 .5 .1

On the left: fully connected.On the right: some connections missing.

Page 1204: Finite Math Lecture Slides

Do we need every connection to be presentin order for X∞ to exist?

No, but they must eventually be present.

In other words,if we wait long enough all transitions become possible.

A regular Markov chain is for which there is some number k suchthat P(k) has all positive entries.

Page 1205: Finite Math Lecture Slides

Do we need every connection to be presentin order for X∞ to exist?

No, but they must eventually be present.

In other words,if we wait long enough all transitions become possible.

A regular Markov chain is for which there is some number k suchthat P(k) has all positive entries.

Page 1206: Finite Math Lecture Slides

Do we need every connection to be presentin order for X∞ to exist?

No, but they must eventually be present.

In other words,if we wait long enough all transitions become possible.

A regular Markov chain is for which there is some number k suchthat P(k) has all positive entries.

Page 1207: Finite Math Lecture Slides

Example: The M.C. with transition matrix

P =

.2 0 .8.1 .9 0.4 .5 .1

has some connections missing, but what if we are patient?

P(2) = P2 =

.36 .40 .24.11 .81 .08.17 .50 .33

All two step transitions are possible.This M.C. is regular.

Page 1208: Finite Math Lecture Slides

Example: The M.C. with transition matrix

P =

.2 0 .8.1 .9 0.4 .5 .1

has some connections missing, but what if we are patient?

P(2) = P2 =

.36 .40 .24.11 .81 .08.17 .50 .33

All two step transitions are possible.This M.C. is regular.

Page 1209: Finite Math Lecture Slides

Example: The M.C. with transition matrix

P =

.2 0 .8.1 .9 0.4 .5 .1

has some connections missing, but what if we are patient?

P(2) = P2 =

.36 .40 .24.11 .81 .08.17 .50 .33

All two step transitions are possible.This M.C. is regular.

Page 1210: Finite Math Lecture Slides

Determining whether a M.C. is regular

Suppose a Markov chain has transition matrix P.

1. Raise P to higher and higher powers.

2. If the zeros eventually disappear then the M.C. is regular.

3. If notice a repeating cycle containing zeros then it is notregular.

Two things make this easy :

1. don’t do any arithmetic: just note 0s and +s;

2. use repeated squarings to look farther faster.

Page 1211: Finite Math Lecture Slides

Determining whether a M.C. is regular

Suppose a Markov chain has transition matrix P.

1. Raise P to higher and higher powers.

2. If the zeros eventually disappear then the M.C. is regular.

3. If notice a repeating cycle containing zeros then it is notregular.

Two things make this easy :

1. don’t do any arithmetic: just note 0s and +s;

2. use repeated squarings to look farther faster.

Page 1212: Finite Math Lecture Slides

Example: Is the M.C. with transition matrix

P =

.2 0 .80 .9 0.4 0 .1

regular or not?

Solution: First: we’ll ignore the numbers and just keep track of 0sand +s.

P =

+ 0 +0 + 0+ 0 +

Page 1213: Finite Math Lecture Slides

Example: Is the M.C. with transition matrix

P =

.2 0 .80 .9 0.4 0 .1

regular or not?

Solution: First: we’ll ignore the numbers and just keep track of 0sand +s.

P =

+ 0 +0 + 0+ 0 +

Page 1214: Finite Math Lecture Slides

P =

0 + 0+ 0 +0 + 0

Now we’ll compute P2 = P(2). Keep in mind the following simple

rules.+ + = + 0 + 0 = 00 (∗) = 0 + + (∗) = +

P2 =

+ 0 +0 + 0+ 0 +

That’s an improvement: 4 zeros instead of 5.

What’s next?

P4 = (P2)2 =

+ 0 +0 + 0+ 0 +

Page 1215: Finite Math Lecture Slides

P =

0 + 0+ 0 +0 + 0

Now we’ll compute P2 = P(2). Keep in mind the following simple

rules.+ + = + 0 + 0 = 00 (∗) = 0 + + (∗) = +

P2 =

+ 0 +0 + 0+ 0 +

That’s an improvement: 4 zeros instead of 5.

What’s next?

P4 = (P2)2 =

+ 0 +0 + 0+ 0 +

Page 1216: Finite Math Lecture Slides

P =

0 + 0+ 0 +0 + 0

Now we’ll compute P2 = P(2). Keep in mind the following simple

rules.+ + = + 0 + 0 = 00 (∗) = 0 + + (∗) = +

P2 =

+ 0 +0 + 0+ 0 +

That’s an improvement: 4 zeros instead of 5.

What’s next?

P

4 = (P2)2 =

+ 0 +0 + 0+ 0 +

Page 1217: Finite Math Lecture Slides

P =

0 + 0+ 0 +0 + 0

Now we’ll compute P2 = P(2). Keep in mind the following simple

rules.+ + = + 0 + 0 = 00 (∗) = 0 + + (∗) = +

P2 =

+ 0 +0 + 0+ 0 +

That’s an improvement: 4 zeros instead of 5.

What’s next?

P4 = (P2)2 =

+ 0 +0 + 0+ 0 +

Page 1218: Finite Math Lecture Slides

P

0 + 0+ 0 +0 + 0

P2 =

+ 0 +0 + 0+ 0 +

P4 =

+ 0 +0 + 0+ 0 +

Now we have stopped making progress:

we’ve entered a cycle.

Since there are still zeros, we’ll never get rid of them.

The M.C. is not regular.

Page 1219: Finite Math Lecture Slides

P

0 + 0+ 0 +0 + 0

P2 =

+ 0 +0 + 0+ 0 +

P4 =

+ 0 +0 + 0+ 0 +

Now we have stopped making progress:

we’ve entered a cycle.

Since there are still zeros, we’ll never get rid of them.

The M.C. is not regular.

Page 1220: Finite Math Lecture Slides

Example: Is the M.C. with transition matrix

P =

1 0 01 0 00 .2 .8

regular or not?

Solution: First: we’ll ignore the numbers and just keep track of 0sand +s.

P =

+ 0 0+ 0 00 + +

Page 1221: Finite Math Lecture Slides

Example: Is the M.C. with transition matrix

P =

1 0 01 0 00 .2 .8

regular or not?

Solution: First: we’ll ignore the numbers and just keep track of 0sand +s.

P =

+ 0 0+ 0 00 + +

Page 1222: Finite Math Lecture Slides

P =

+ 0 0+ 0 00 + +

Now start repeatedly squaring.

P2 =

+ 0 0+ 0 0+ + +

P4 =

+ 0 0+ 0 0+ + +

Not regular.

Page 1223: Finite Math Lecture Slides

P =

+ 0 0+ 0 00 + +

Now start repeatedly squaring.

P2 =

+ 0 0+ 0 0+ + +

P4 =

+ 0 0+ 0 0+ + +

Not regular.

Page 1224: Finite Math Lecture Slides

P =

+ 0 0+ 0 00 + +

Now start repeatedly squaring.

P2 =

+ 0 0+ 0 0+ + +

P4 =

+ 0 0+ 0 0+ + +

Not regular.

Page 1225: Finite Math Lecture Slides

Example: Is the M.C. with transition matrix

P =

0 0 10 0 1.8 .2 0

regular or not?

Solution: First: we’ll ignore the numbers and just keep track of 0sand +s.

P =

0 0 +0 0 ++ + 0

Page 1226: Finite Math Lecture Slides

Example: Is the M.C. with transition matrix

P =

0 0 10 0 1.8 .2 0

regular or not?

Solution: First: we’ll ignore the numbers and just keep track of 0sand +s.

P =

0 0 +0 0 ++ + 0

Page 1227: Finite Math Lecture Slides

P =

0 0 +0 0 ++ + 0

Now start repeatedly squaring.

P2 =

+ + 0+ + 00 0 +

P4 =

+ + 0+ + 00 0 +

Not regular.

Page 1228: Finite Math Lecture Slides

P =

0 0 +0 0 ++ + 0

Now start repeatedly squaring.

P2 =

+ + 0+ + 00 0 +

P4 =

+ + 0+ + 00 0 +

Not regular.

Page 1229: Finite Math Lecture Slides

P =

0 0 +0 0 ++ + 0

Now start repeatedly squaring.

P2 =

+ + 0+ + 00 0 +

P4 =

+ + 0+ + 00 0 +

Not regular.

Page 1230: Finite Math Lecture Slides

Example: Is the M.C. with transition matrix

P =

0 1 00 0 1.8 .2 0

regular or not?

Solution: First: we’ll ignore the numbers and just keep track of 0sand +s.

P =

0 + 00 0 ++ + 0

Page 1231: Finite Math Lecture Slides

Example: Is the M.C. with transition matrix

P =

0 1 00 0 1.8 .2 0

regular or not?

Solution: First: we’ll ignore the numbers and just keep track of 0sand +s.

P =

0 + 00 0 ++ + 0

Page 1232: Finite Math Lecture Slides

P =

0 + 00 0 ++ + 0

Now start repeatedly squaring.

P2 =

0 0 ++ + 00 + +

P4 =

0 + ++ + ++ + +

P8 =

+ + ++ + ++ + +

Regular.

Page 1233: Finite Math Lecture Slides

P =

0 + 00 0 ++ + 0

Now start repeatedly squaring.

P2 =

0 0 ++ + 00 + +

P4 =

0 + ++ + ++ + +

P8 =

+ + ++ + ++ + +

Regular.

Page 1234: Finite Math Lecture Slides

P =

0 + 00 0 ++ + 0

Now start repeatedly squaring.

P2 =

0 0 ++ + 00 + +

P4 =

0 + ++ + ++ + +

P8 =

+ + ++ + ++ + +

Regular.

Page 1235: Finite Math Lecture Slides

P =

0 + 00 0 ++ + 0

Now start repeatedly squaring.

P2 =

0 0 ++ + 00 + +

P4 =

0 + ++ + ++ + +

P8 =

+ + ++ + ++ + +

Regular.